add 4,2,3

अब Quizwiz के साथ अपने होमवर्क और परीक्षाओं को एस करें!

The client says, "I hate this stupid COPD." What is the best response by the nurse?

"What is bothering you?" Encourage the client, and the family, to express their feelings about limitations on their lifestyle and about disease progression. This is not the time to lecture the client regarding his smoking habits; the client is expressing a need for support. "Why" questions can seem accusatory and may make a client less likely to talk about what he or she is feeling. The client's feelings should never be minimized.

33. A patient with COPD has meal-related dyspnea. To address this issue, which drug does the nurse offer the patient 30 minutes before the meal? A. Albuterol (Ventolin) B. Guaifenesin (Organidin) C. Fluticasone (Flovent) D. Pantoprazole sodium (Protonix)

A

A client is having surgery. He asks his nurse, "When they put that tube in my throat, where does it really go?" What is the name of the opening of the vocal cords? A. Arytenoid cartilage B. Epiglottis C. Glottis D. Palatine tonsils

C. Glottis Correct: The glottis is the opening of the vocal cords into which the endotracheal tube is passed during intubation for surgery.

11. An older adult patient experiences an asthma attack that is severe enough to warrant the use of a rescue drug. Which medication is best to use for the acute symptoms? A. Omalizumab (Xolair) B. Fluticasone (Flovent) C. Salmeterol (Severent) D. Albuterol (Proventil)

D

A client has been diagnosed with chronic bronchitis and started on a mucolytic. What is the rationale for ordering a mucolytic for this client?

Mucolytics thin secretions, making them easier to expectorate. The term mucolytic means "breaking down mucus." Mucolytics cause secretions to thin, making them easier to expectorate; this is important for a client with chronic bronchitis. Mucolytics do not decrease secretion production. Mucolytics may increase gas exchange, but this is an indirect property and is not the main function. Mucolytics do not have any bronchodilation properties.

A client is admitted with asthma. How is this disease differentiated from other chronic lung disorders?

The client is symptom-free between exacerbations. The client may be completely symptom-free between exacerbations. Asthma affects people of all ages. Dyspnea is a common symptom of many chronic lung diseases. Coughing occurs in many acute and chronic lung diseases.

A pt has been transferred from the ICU to the med-surg unit after a laryngectomy. What does the nurse suggest to encourage the pt to participate in self-care? a. changing the trach collar b. suctioning the mouth with an oral suction device c. checking the stoma with a flashlight d. observing the color of the reconstructive flap

b

59. The nurse is caring for a patient with a chest tube. What is the correct nursing intervention for this patient? A. The patient is encouraged to cough and do deep-breathing exercises frequently. B. "Stripping" of the chest tubes is done routinely to prevent obstruction by blood clots. C. Water level in the suction chamber need not be monitored, just the collection chamber. D. Drainage containers are positioned upright or on the bed next to the patient.

A

What does the nurse include in the teaching session for a pt who is scheduled to have a partial laryngectomy? a. supraglottic mehod of swallowing b. presence of a trach tube and NG tube for feeding due to post-op swelling c. not being able to eat solid foods d. permanence of the tracheostomy, referred to as a laryngectomy stoma

a

Question 21 of 29 The nurse's friend fears that his mother is getting old, saying that she is becoming extremely forgetful and disoriented and is beginning to wander. What is the nurse's best response? a. "Have you taken her for a check-up?" b. "She has Alzheimer's disease." c. "That is a normal part of aging." d. "You should look into respite care."

a "Have you taken her for a check-up?" The mother's symptoms indicate possible Alzheimer's disease or some other physiologic imbalance, and she should be assessed further by a health care provider. The nurse cannot diagnose Alzheimer's disease. The mother's behavior is not normal age-related behavior. Respite care is for caregivers, not for clients.

Which instructions must the nurse give to a pt after rhinoplasty to prevent bleeding? SELECT ALL THAT APPLY a. limit/avoid straining during bowel movements (eg Valsalva maneuver) b. do not sniff upwards/blow your nose c. sneeze with your mouth closed for a few days after packing is removed d. forceful coughing should be done to keep airways clear e. avoid aspirin-containing products/NSAIDs f. use a humidifier to prevent mucosal drying

a b e f

15. A nurse assesses a client with bladder cancer who is recovering from a complete cystectomy with ileal conduit. Which assessment finding should alert the nurse to urgently contact the health care provider? a. The ileostomy is draining blood-tinged urine. b. There is serous sanguineous drainage present on the surgical dressing. c. The ileostomy stoma is pale and cyanotic in appearance. d. Oxygen saturations are 92% on room air.

ANS: C A pale or cyanotic stoma indicates impaired circulation to the stoma and must be treated to prevent necrosis. Blood-tinged urine and serous sanguineous drainage are expected after this type of surgery. Oxygen saturation of 92% on room air is at the low limit of normal. DIF: Applying/Application REF: 1367 KEY: Urothelial cancer| postoperative nursing MSC: Integrated Process: Nursing Process: Implementation NOT: Client Needs Category: Safe and Effective Care Environment: Management of Care

A nurse assesses a client who is being treated for hyperglycemic-hyperosmolar state (HHS). Which clinical manifestation indicates to the nurse that the therapy needs to be adjusted? a. Serum potassium level has increased. b. Blood osmolarity has decreased. c. Glasgow Coma Scale score is unchanged. d. Urine remains negative for ketone bodies.

ANS: C A slow but steady improvement in central nervous system functioning is the best indicator of therapy effectiveness for HHS. Lack of improvement in the level of consciousness may indicate inadequate rates of fluid replacement. The Glasgow Coma Scale assesses the client's state of consciousness against criteria of a scale including best eye, verbal, and motor responses. An increase in serum potassium, decreased blood osmolality, and urine negative for ketone bodies do not indicate adequacy of treatment.

A nurse cares for a client who is recovering from a closed percutaneous kidney biopsy. The client states, "My pain has suddenly increased from a 3 to a 10 on a scale of 0 to 10." Which action should the nurse take first? a. Reposition the client on the operative side. b. Administer the prescribed opioid analgesic. c. Assess the pulse rate and blood pressure. d. Examine the color of the client's urine.

ANS: C An increase in the intensity of pain after a percutaneous kidney biopsy is a symptom of internal hemorrhage. A change in vital signs can indicate that hemorrhage is occurring. Before other actions, the nurse must assess the client's hemodynamic status.

9. A confused client with pneumonia is admitted with an indwelling catheter in place. During interdisciplinary rounds the following day, which question should the nurse ask the primary health care provider? a. Do you want daily weights on this client? b. Will the client be able to return home? c. Can we discontinue the indwelling catheter? d. Should we get another chest x-ray today?

ANS: C An indwelling catheter dramatically increases the risks of urinary tract infection and urosepsis. Nursing staff should ensure that catheters are left in place only as long as they are medically needed. The nurse should inquire about removing the catheter. All other questions might be appropriate, but because of client safety, this question takes priority. DIF: Applying/Application REF: 1345 KEY: Infection control MSC: Integrated Process: Nursing Process: Assessment NOT: Client Needs Category: Physiological Integrity: Reduction of Risk Potential

A client has a metastatic bone tumor. What action by the nurse takes priority? a. Administer pain medication as prescribed. b. Elevate the extremity and apply moist heat. c. Handle the affected extremity with caution. d. Place the client on protective precautions.

ANS: C Bones invaded by tumors are very fragile and fracture easily. For client safety, the nurse handles the affected extremity with great care. Pain medication should be given to control pain. Elevation and heat may or may not be helpful. Protective precautions are not needed for this client.

14. A nurse cares for a client who has pyelonephritis. The client states, "I am embarrassed to talk about my symptoms." How should the nurse respond? a. "I am a professional. Your symptoms will be kept in confidence." b. "I understand. Elimination is a private topic and shouldn't be discussed." c. "Take your time. It is okay to use words that are familiar to you." d. "You seem anxious. Would you like a nurse of the same gender to care for you?"

ANS: C Clients may be uncomfortable discussing issues related to elimination and the genitourinary area. The nurse should encourage the client to use language that is familiar to the client. The nurse should not make promises that cannot be kept, like keeping the client's symptoms confidential. The nurse must assess the client and cannot take the time to stop the discussion or find another nurse to complete the assessment.

A nurse assesses a client with a rotator cuff injury. Which finding should the nurse expect to assess? a. Inability to maintain adduction of the affected arm for more than 30 seconds b. Shoulder pain that is relieved with overhead stretches and at night c. Inability to initiate or maintain abduction of the affected arm at the shoulder d. Referred pain to the shoulder and arm opposite the affected shoulder

ANS: C Clients with a rotator cuff tear are unable to initiate or maintain abduction of the affected arm at the shoulder. This is known as the drop arm test. The client should not have difficulty with adduction of the arm, nor experience referred pain to the opposite shoulder. Pain is usually more intense at night and with overhead activities.

4. A nurse cares for a middle-aged female client with diabetes mellitus who is being treated for the third episode of acute pyelonephritis in the past year. The client asks, "What can I do to help prevent these infections?" How should the nurse respond? a. "Test your urine daily for the presence of ketone bodies and proteins." b. "Use tampons rather than sanitary napkins during your menstrual period." c. "Drink more water and empty your bladder more frequently during the day." d. "Keep your hemoglobin A1c under 9% by keeping your blood sugar controlled."

ANS: C Clients with long-standing diabetes mellitus are at risk for pyelonephritis for many reasons. Chronically elevated blood glucose levels spill glucose into the urine, changing the pH and providing a favorable climate for bacterial growth. The neuropathy associated with diabetes reduces bladder tone and reduces the client's sensation of bladder fullness. Thus, even with large amounts of urine, the client voids less frequently, allowing stasis and overgrowth of microorganisms. Increasing fluid intake (specifically water) and voiding frequently prevent stasis and bacterial overgrowth. Testing urine and using tampons will not help prevent pyelonephritis. A hemoglobin A1c of 9% is too high.

A nurse cares for a client with chronic obstructive pulmonary disease (COPD) who appears thin and disheveled. Which question should the nurse ask first? a. "Do you have a strong support system?" b. "What do you understand about your disease?" c. "Do you experience shortness of breath with basic activities?" d. "What medications are you prescribed to take each day?"

ANS: C Clients with severe COPD may not be able to perform daily activities, including bathing and eating, because of excessive shortness of breath. The nurse should ask the client if shortness of breath is interfering with basic activities. Although the nurse should know about the client's support systems, current knowledge, and medications, these questions do not address the client's appearance.

A nurse assesses a client who has a 15-year history of diabetes and notes decreased tactile sensation in both feet. Which action should the nurse take first? a. Document the finding in the client's chart. b. Assess tactile sensation in the client's hands. c. Examine the client's feet for signs of injury. d. Notify the health care provider.

ANS: C Diabetic neuropathy is common when the disease is of long duration. The client is at great risk for injury in any area with decreased sensation because he or she is less able to feel injurious events. Feet are common locations for neuropathy and injury, so the nurse should inspect them for any signs of injury. After assessment, the nurse should document findings in the client's chart. Testing sensory perception in the hands may or may not be needed. The health care provider can be notified after assessment and documentation have been completed.

A nurse assesses a client with renal insufficiency and a low red blood cell count. The client asks, "Is my anemia related to the renal insufficiency?" How should the nurse respond? a. "Red blood cells produce erythropoietin, which increases blood flow to the kidneys." b. "Your anemia and renal insufficiency are related to inadequate vitamin D and a loss of bone density." c. "Erythropoietin is usually released from the kidneys and stimulates red blood cell production in the bone marrow." d. "Kidney insufficiency inhibits active transportation of red blood cells throughout the blood."

ANS: C Erythropoietin is produced in the kidney and is released in response to decreased oxygen tension in the renal blood supply. Erythropoietin stimulates red blood cell production in the bone marrow. Anemia and renal insufficiency are not manifestations of vitamin D deficiency. The kidneys do not play a role in the transportation of red blood cells or any other cells in the blood.

A nurse obtains a sterile urine specimen from a client's Foley catheter. After applying a clamp to the drainage tubing distal to the injection port, which action should the nurse take next? a. Clamp another section of the tube to create a fixed sample section for retrieval. b. Insert a syringe into the injection port and aspirate the quantity of urine required. c. Clean the injection port cap of the drainage tubing with povidone-iodine solution. d. Withdraw 10 mL of urine and discard it, then withdraw a fresh sample of urine.

ANS: C It is important to clean the injection port cap of the catheter drainage tubing with an appropriate antiseptic, such as povidone-iodine solution or alcohol. This will help prevent surface contamination before injection of the syringe. The urine sample should be collected directly from the catheter, therefore, a second clamp to create a sample section would not be appropriate. Every sample from the catheter is usable, there is the need to discard the first sample.

A nurse is caring for several older clients in the hospital that the nurse identifies as being at high risk for healthcare-associated pneumonia. To reduce this risk, what activity should the nurse delegate to the unlicensed assistive personnel (UAP)? a. Encourage between-meal snacks. b. Monitor temperature every 4 hours. c. Provide oral care every 4 hours. d. Report any new onset of cough.

ANS: C Oral colonization by gram-negative bacteria is a risk factor for healthcare-associated pneumonia. Good, frequent oral care can help prevent this from developing and is a task that can be delegated to the UAP. Encouraging good nutrition is important, but this will not prevent pneumonia. Monitoring temperature and reporting new cough in clients is important to detect the onset of possible pneumonia but do not prevent it.

4. After teaching a client with bacterial cystitis who is prescribed phenazopyridine (Pyridium), the nurse assesses the clients understanding. Which statement made by the client indicates a correct understanding of the teaching? a. I will not take this drug with food or milk. b. If I think I am pregnant, I will stop the drug. c. An orange color in my urine should not alarm me. d. I will drink two glasses of cranberry juice daily.

ANS: C Phenazopyridine discolors urine, most commonly to a deep reddish orange. Many clients think they have blood in their urine when they see this. In addition, the urine can permanently stain clothing. Phenazopyridine is safe to take if the client is pregnant. There are no dietary restrictions or needs while taking this medication. DIF: Applying/Application REF: 1349 KEY: Cystitis| medication safety MSC: Integrated Process: Teaching/Learning NOT: Client Needs Category: Physiological Integrity: Pharmacological and Parenteral Therapies

26. A nurse provides phone triage to a pregnant client. The client states, I am experiencing a burning pain when I urinate. How should the nurse respond? a. This means labor will start soon. Prepare to go to the hospital. b. You probably have a urinary tract infection. Drink more cranberry juice. c. Make an appointment with your provider to have your infection treated. d. Your pelvic wall is weakening. Pelvic muscle exercises should help.

ANS: C Pregnant clients with a urinary tract infection require prompt and aggressive treatment because cystitis can lead to acute pyelonephritis during pregnancy. The nurse should encourage the client to make an appointment and have the infection treated. Burning pain when urinating does not indicate the start of labor or weakening of pelvic muscles. DIF: Applying/Application REF: 1346 KEY: Cystitis MSC: Integrated Process: Teaching/Learning NOT: Client Needs Category: Physiological Integrity: Reduction of Risk Potential Test Bank - Medical-Surgical Nursing: Concepts for Interprofessional Collaborative Care 9e 564

After teaching a client who is prescribed salmeterol (Serevent), the nurse assesses the client's understanding. Which statement by the client indicates a need for additional teaching? a. "I will be certain to shake the inhaler well before I use it." b. "It may take a while before I notice a change in my asthma." c. "I will use the drug when I have an asthma attack." d. "I will be careful not to let the drug escape out of my nose and mouth."

ANS: C Salmeterol is designed to prevent an asthma attack; it does not relieve or reverse symptoms. Salmeterol has a slow onset of action; therefore, it should not be used as a rescue drug. The drug must be shaken well because it has a tendency to separate easily. Poor technique on the client's part allows the drug to escape through the nose and mouth.

A nurse obtains the health history of a client who is recently diagnosed with lung cancer and identifies that the client has a 60-pack-year smoking history. Which action is most important for the nurse to take when interviewing this client? a. Tell the client that he needs to quit smoking to stop further cancer development. b. Encourage the client to be completely honest about both tobacco and marijuana use. c. Maintain a nonjudgmental attitude to avoid causing the client to feel guilty. d. Avoid giving the client false hope regarding cancer treatment and prognosis.

ANS: C Smoking history includes the use of cigarettes, cigars, pipe tobacco, marijuana, and other controlled substances. Because the client may have guilt or denial about this habit, assume a nonjudgmental attitude during the interview. This will encourage the client to be honest about the exposure. Ask the client whether any of these substances are used now or were used in the past. Assess whether the client has passive exposure to smoke in the home or workplace. If the client smokes, ask for how long, how many packs per day, and whether he or she has quit smoking (and how long ago). Document the smoking history in pack-years (number of packs smoked daily multiplied by the number of years the client has smoked). Quitting smoking may not stop further cancer development. This statement would be giving the client false hope, which should be avoided, but is not as important as maintaining a nonjudgmental attitude.

A nurse cares for a client recovering from an above-the-knee amputation of the right leg. The client reports pain in the right foot. Which prescribed medication should the nurse administer first? a. Intravenous morphine b. Oral acetaminophen c. Intravenous calcitonin d. Oral ibuprofen

ANS: C The client is experiencing phantom limb pain, which usually manifests as intense burning, crushing, or cramping. IV infusions of calcitonin during the week after amputation can reduce phantom limb pain. Opioid analgesics such as morphine are not as effective for phantom limb pain as they are for residual limb pain. Oral acetaminophen and ibuprofen are not used in treating phantom limb pain.

After teaching a client who is newly diagnosed with type 2 diabetes mellitus, the nurse assesses the client's understanding. Which statement made by the client indicates a need for additional teaching? a. "I should increase my intake of vegetables with higher amounts of dietary fiber." b. "My intake of saturated fats should be no more than 10% of my total calorie intake." c. "I should decrease my intake of protein and eliminate carbohydrates from my diet." d. "My intake of water is not restricted by my treatment plan or medication regimen."

ANS: C The client should not completely eliminate carbohydrates from the diet, and should reduce protein if microalbuminuria is present. The client should increase dietary intake of complex carbohydrates, including vegetables, and decrease intake of fat. Water does not need to be restricted unless kidney failure is present.

At 4:45 p.m., a nurse assesses a client with diabetes mellitus who is recovering from an abdominal hysterectomy 2 days ago. The nurse notes that the client is confused and diaphoretic. The nurse reviews the assessment data provided in the chart below: Capillary Blood Glucose Testing (AC/HS) Dietary Intake At 0630: 95 At 1130: 70 At 1630: 47 Breakfast: 10% eaten - client states she is not hungry Lunch: 5% eaten - client is nauseous, vomits once After reviewing the client's assessment data, which action is appropriate at this time? a. Assess the client's oxygen saturation level and administer oxygen. b. Reorient the client and apply a cool washcloth to the client's forehead. c. Administer dextrose 50% intravenously and reassess the client. d. Provide a glass of orange juice and encourage the client to eat dinner.

ANS: C The client's symptoms are related to hypoglycemia. Since the client has not been tolerating food, the nurse should administer dextrose intravenously. The client's oxygen level could be checked, but based on the information provided, this is not the priority. The client will not be reoriented until the glucose level rises.

A nurse cares for a client who tests positive for alpha1-antitrypsin (AAT) deficiency. The client asks, "What does this mean?" How should the nurse respond? a. "Your children will be at high risk for the development of chronic obstructive pulmonary disease." b. "I will contact a genetic counselor to discuss your condition." c. "Your risk for chronic obstructive pulmonary disease is higher, especially if you smoke." d. "This is a recessive gene and should have no impact on your health."

ANS: C The gene for AAT is a recessive gene. Clients with only one allele produce enough AAT to prevent chronic obstructive pulmonary disease (COPD) unless the client smokes. A client with two alleles is at high risk for COPD even if not exposed to smoke or other irritants. The client is a carrier, and children may or may not be at high risk depending on the partner's AAT levels. Contacting a genetic counselor may be helpful but does not address the client's current question.

A client is scheduled for a bone biopsy. What action by the nurse takes priority? a. Administering the preoperative medications b. Answering any questions about the procedure c. Ensuring that informed consent is on the chart d. Showing the client's family where to wait

ANS: C The priority is to ensure that informed consent is on the chart. The preoperative medications should not be administered until the nurse is confident the procedure will occur and the client has already signed the consent, if the medications include anxiolytics or sedatives or opioids. The provider should answer questions about the procedure. The nurse does show the family where to wait, but this is not the priority and could be delegated.

A nurse is providing education to a community women's group about lifestyle changes helpful in preventing osteoporosis. What topics does the nurse cover? (Select all that apply.) a. Cut down on tobacco product use. b. Limit alcohol to two drinks a day. c. Strengthening exercises are important. d. Take recommended calcium and vitamin D. e. Walk 30 minutes at least 3 times a week.

ANS: C, D, E Lifestyle changes can be made to decrease the occurrence of osteoporosis and include strengthening and weight-bearing exercises and getting the recommended amounts of both calcium and vitamin D. Tobacco should be totally avoided. Women should not have more than one drink per day.

A client is admitted to the surgical floor with chest pain, shortness of breath, and hypoxemia after having a knee replacement. What primary assessment will the nurse make while preparing the client for a computed tomography (CT) scan? "Do you have any metal anywhere in your body?" "Do you have diabetes?" "Are you allergic to iodine or shellfish?" "Do you drink alcohol regularly?"

"Are you allergic to iodine or shellfish?" While preparing the client for a CT scan, the nurse's primary assessment would be to determine whether the client has any sensitivity to the contrast material by asking if the client has a known allergy to contrast, iodine or shellfish. CT scans, especially spiral or helical CT scans, with injected contrast can detect pulmonary emboli.Assessing for any metal in the body is done when clients undergo MRI. Diabetes is not a contraindication for CT with contrast. However, if the client receives metformin, the drug is stopped at least 24 hours before contrast dye is used and withheld until adequate kidney function is confirmed. Assessing regular alcohol intake is important, but is not the primary assessment.

A client, who is a mother of two, has autosomal dominant polycystic kidney disease (ADPKD). Which statement by the client indicates a need for further education about her disease? "By maintaining a low-salt diet in our house, I can prevent ADPKD in my children." "Even though my children don't have symptoms at the same age I did, they can still have ADPKD." "If my children have the ADPKD gene, they will have cysts by the age of 30." "My children have a 50% chance of inheriting the ADPKD gene that causes the disease."

"By maintaining a low-salt diet in our house, I can prevent ADPKD in my children." There is no way to prevent ADPKD, although early detection and management of hypertension may slow the progression of kidney damage. Limiting salt intake can help control blood pressure. Presentation of ADPKD can vary by age of onset, manifestations, and illness severity, even in one family. Almost 100% of those who inherit a polycystic kidney disease (PKD) gene will develop kidney cysts by age 30. Children of parents who have the autosomal dominant form of PKD have a 50% chance of inheriting the gene that causes the disease.

A client is scheduled for an electroencephalogram (EEG) in the morning. Which instruction does the nurse give the client? "Do not take any sedatives 12-24 hours before the test." "Please do not have anything to eat or drink after midnight." "You may bring some music to listen to for distraction." "You will need to have someone to drive you home."

"Do not take any sedatives 12-24 hours before the test." Before an EEG, the client needs to be instructed not to use sedatives or stimulants for 12-24 hours prior to the test.A client would not fast prior to an EEG as hypoglycemia may alter results. Testing takes place in a quiet room, so music for distraction is not appropriate. Unless the EEG is for sleep disorder diagnosis, the client will not need to be driven home.

A 70-year-old client has a complicated medical history, including chronic obstructive pulmonary disease. Which client statement indicates the need for further teaching about the disease?

"I am here to receive the yearly pneumonia shot again." Clients 65 years and older, as well as those who have chronic health problems, should be encouraged to receive the pneumonia vaccine, which is not given annually but only once. Older clients are encouraged to receive a flu shot annually because the vaccine changes, depending on anticipated strains for the upcoming year. It is a good idea to avoid large gatherings during cold and flu season. New recommendations from the Centers for Disease Control and Prevention for controlling the spread of flu include coughing or sneezing into the upper sleeve rather than into the hand.

The school nurse is counseling a teenage student about how to prevent kidney trauma. Which statement by the student indicates a need for further teaching? "I can't play any type of contact sports because my brother had kidney cancer." "I avoid riding motorcycles." "I always wear pads when playing football." "I always wear a seat belt in the car."

"I can't play any type of contact sports because my brother had kidney cancer." Contact sports and high-risk activities should be avoided if a person has only one kidney. A family history of kidney cancer does not prohibit this type of activity. To prevent kidney and genitourinary trauma, caution should be taken when riding bicycles and motorcycles. People should wear appropriate protective clothing when participating in contact sports. Anyone riding in a car should wear a seat belt.

Which statement by a client with diabetic nephropathy indicates a need for further education about the disease? "Diabetes is the leading cause of kidney failure." "I need less insulin, so I am getting better." "My blood sugar may drop really low at times." "I must call my provider if the urine dipstick shows protein."

"I need less insulin, so I am getting better." When kidney function is reduced, insulin is available for a longer time and thus less of it is needed. Unfortunately, many clients believe this means that their diabetes is improving. It is true that diabetes mellitus is the leading cause of end-stage kidney disease among Caucasians in the United States. Clients with worsening kidney function may begin to have frequent hypoglycemic episodes. Proteinuria, which may be mild, moderate, or severe, indicates a need for follow-up.

An older client presents to the clinic after a ground level fall at home. What statement by the client indicates the need for more injury prevention education? "I always take my medicine as directed." "I only eat little snacks so I don't gain weight." "I will make sure I drink enough water." "I make sure to get as much sleep as I used to."

"I only eat little snacks so I don't gain weight." More fall injury prevention education is needed when the client says that he/she will only eat little snacks to prevent weight gain. The brain is sensitive to decreased glucose levels which can lead to falls. This is especially noted in older clients.Taking medication as directed, ensuring adequate hydration, nutrition, and sleep help promote nervous system health and decrease the risk for falls in the elderly.

The nurse is performing discharge teaching for a client after a nephrectomy for renal cell carcinoma. Which statement by the client indicates that teaching has been effective? "Because renal cell carcinoma usually affects both kidneys, I'll need to be watched closely." "I'll eventually require some type of renal replacement therapy." "I'll need to decrease my fluid intake to prevent stress to my remaining kidney." "My remaining kidney will provide me with normal kidney function now."

"My remaining kidney will provide me with normal kidney function now." After a nephrectomy, the second kidney is expected to eventually provide adequate kidney function, but this may take days or weeks. Renal cell carcinoma typically only affects one kidney. Renal replacement therapy is not the typical treatment for renal cell carcinoma. Fluids should be maintained to flush the remaining kidney.

A client is hesitant to talk to the nurse about genitourinary dysfunction symptoms. What is the nurse's best response? "Don't worry, no one else will know." "Take your time. What is bothering you the most?" "Why are you hesitant?" "You need to tell me so we can determine what is wrong."

"Take your time. What is bothering you the most?" Asking the client what is bothering him or her expresses patience and understanding when trying to identify the client's problem. Telling the client that others will not know is untrue because the client's symptoms will be in the medical record for other health care personnel to see. Asking why the client is hesitant can seem accusatory and threatening to the client. Admonishing the client to disclose his or her symptoms is too demanding; the nurse must be more understanding of the client's embarrassment.

A client is being discharged home with active tuberculosis. Which information does the nurse include in the discharge teaching plan?

"You will not be contagious to the people you have been living with." The people the client has been living with have already been exposed and need to be tested. They cannot be re-exposed simply because the diagnosis has now been confirmed. The client with active tuberculosis is contagious, even while taking medication. The length of time for treatment is 6 months. Fluid from the pulmonary capillaries and red blood cells moving into the alveoli is a result of the inflammatory process. Rust-colored sputum is an indication that the tuberculosis is getting worse.

People involved in which occupations or activities are encouraged to wear masks and to have adequate ventilation? (Select all that apply.) -Plumbers -Bakers -Coal Miners -Electrician -Furniture Refinishers -Potters

-Bakers -Coal Miners -Furniture Refinishers -Potters Being exposed to flour as a baker for prolonged periods of time may cause a condition called occupational asthma. Coal miners are at risk to develop pneumoconiosis as the result of inhalation of coal dust. Owing to the chemicals used to refinish furniture (paint strippers, solvents), masks and adequate ventilation are essential for furniture refinishers. One of the main solvents involved will metabolize in the body to carbon monoxide and will impair the ability of the tissue to extract oxygen. Silicosis or inhalation of silica dust is a hazard for professional and recreational potters. Except in unique situations, electricians and plumbers do not need to wear masks or utilize special ventilation for their jobs.

A client with tuberculosis (TB) who is homeless and has been living in shelters for the past 6 months asks the nurse why he must take so many medications. What information will the nurse provide in answering this question? (Select all that apply.)

-Combination drug therapy is effective in preventing transmission. -Combination drug therapy is the most effective method of treating TB. -Multiple drug regimens destroy organisms as quickly as possible. -The use of multiple drugs reduces the emergence of drug-resistant organisms. Combination drug therapy is the most effective method for treating TB and preventing transmission. Multiple drug regimens are able to destroy organisms as quickly as possible and reduce the emergence of drug-resistant organisms. Although combination drug therapy will decrease the required length of time for treatment, the length of treatment is decreased to 6 months from 6 to 12 months.

A client has recently been released from prison and has just tested positive for tuberculosis (TB). What teaching points does the community health nurse want to stress for this client regarding medications? (Select all that apply.)

-Not taking the medication could lead to an infection that is difficult to treat or to total drug resistance. -The medications may cause nausea. The client should take them at bedtime. Not taking the medication as prescribed could lead to an infection that is difficult to treat or to total drug resistance. The medications may cause nausea and are best taken at bedtime to prevent this. The client is generally not contagious after 2 to 3 weeks of consecutive treatment and improvement in the condition has been observed. The combination regimen for treatment of TB has decreased treatment time from 6 to 12 months to 6 months. TB medications may cause liver failure, not kidney failure.

29. Patients with asthma are taught self-care activities and treatment modalities according to the "step method." Which symptoms and medication routines relate to step 3? A. Symptoms occur daily; daily use of inhaled corticosteroid, add a long-acting beta agonist. B. Symptoms occur more than once per week; daily use of antiiflammatory inhaler. C. Symptoms occur less than once per week; use of rescue inhalers once per week. D. Frequent exacerbations occur with limited physical activity; increased use of rescue inhalers.

A

45. A patient has returned several times to the clinic for treatment of respiratory problems. Which action does the nurse perform first? A. Obtain a history of the patient's previous problems and response to therapy. B. Ask the patient to describe his compliance with the prescribed therapies. C. Obtain a request for diagnostic testing, including a tuberculosis and human immunodeficiency virus (HIV) evaluation. D. Listen to the patient's lungs, obtain a pulse oximetry reading, and count the respiratory rate.

A

47. The patient with COPD is undergoing pulmonary rehabilitation by walking. What does the nurse teach this patient about when to increase his or her walking time? A. "You should increase your walking time when your rest periods decrease." B. "You should increase your walking time when your heart rate remains less than 80/minute." C. "You should increase your walking time when you are no longer short of breath." D. "You should increase your walking time when you do not need to use an inhaler."

A

54. The nurse has completed a community presentation about lung cancer. Which statement from a participant demonstrates an understanding of the information presented? A. "The primary prevention for reducing the risk of lung cancer is to stop smoking and avoid secondhand smoke." B. "The overall 5-year survival rate for all patients with lung cancer is 85%." C. "The death rate for lung cancer is less than prostate, breast, and colon cancer combined." D. "Cures are most likely for patients who undergo treatment for stage III disease."

A

72. A patient with a history of asthma enters the emergency department with severe dyspnea, accessory muscle involvement, neck vein distention, and severe inspiratory/expiratory wheezing. The nurse is prepared to assist the provider with which emergency procedure if the patient does not respond to initial interventions? A. Intubation B. Needle thoracentesis C. Chest tube insertion D. Pleurodesis

A

74. The nurse is instructing a patient to use a flutter-valve mucus clearance device. What should the patient be taught to do? A. Inhale deeply and exhale forcefully through the device. B. Use an inhalation technique that is similar to the handheld inhaler. C. Use pursed-lip breathing before and after usage. D. Exhale slowly through the nose, and then inhale by sniffing.

A

A nurse assesses a client with asthma and notes bilateral wheezing, decreased pulse oxygen saturation, and suprasternal retraction on inhalation. Which actions should the nurse take? (Select all that apply.) a. Administer prescribed salmeterol (Serevent) inhaler. b. Assess the client for a tracheal deviation. c. Administer oxygen to keep saturations greater than 94%. d. Perform peak expiratory flow readings. e. Administer prescribed albuterol (Proventil) inhaler.

ANS: C, E Suprasternal retraction caused by inhalation usually indicates that the client is using accessory muscles and is having difficulty moving air into the respiratory passages because of airway narrowing. Wheezing indicates a narrowed airway; a decreased pulse oxygen saturation also supports this finding. The asthma is not responding to the medication, and intervention is needed. Administration of a rescue inhaler is indicated, probably along with administration of oxygen. The nurse would not do a peak flow reading at this time, nor would a code be called. Midline trachea is a normal and expected finding.

The community health nurse is planning tuberculosis treatment for a client who is homeless and heroin-addicted. Which action will be most effective in ensuring that the client completes treatment? A) Arrange for a health care worker to watch the client take the medication. B) Give the client written instructions about how to take prescribed medications. C) Have the client repeat medication names and side effects. D) Instruct the client about the possible consequences of nonadherence. (Chp. 31; elsevier resources)

A) Arrange for a health care worker to watch the client take the medication. (Chp. 31; elsevier resources)

A client with tuberculosis (TB) who is homeless and has been living in shelters for the past 6 months asks the nurse why he must take so many medications. What information will the nurse provide in answering this question? Select all that apply. A) Combination drug therapy is effective in preventing transmission. B) Combination drug therapy is the most effective method of treating TB. C) Combination drug therapy will decrease the length of required treatment to 2 months. D) Multiple drug regimens destroy organisms as quickly as possible. E) The use of multiple drugs reduces the emergence of drug-resistant organisms. (Chp. 31; elsevier resources)

A) Combination drug therapy is effective in preventing transmission. B) Combination drug therapy is the most effective method of treating TB. D) Multiple drug regimens destroy organisms as quickly as possible. E) The use of multiple drugs reduces the emergence of drug-resistant organisms. (Chp. 31; elsevier resources)

A client is taking isoniazid, rifampin, pyrazinamide, and ethambutol for tuberculosis. The client calls to report visual changes, including blurred vision and reduced visual fields. Which medication may be causing these changes? A) Ethambutol B) Isoniazid C) Pyrazinamide D) Rifampin (Chp. 31; elsevier resources)

A) Ethambutol (Chp. 31; elsevier resources)

A client who has been homeless and has spent the past 6 months living in shelters has been diagnosed with confirmed tuberculosis (TB). Which medications does the nurse expect to be ordered for the client? A) Isoniazid (INH), rifampin (Rifadin), pyrazinamide (Zinamide), ethambutol (Myambutol) B) Metronidazole (Flagyl), acyclovir (Zovirax), flunisolide (AeroBid), rifampin (Rifadin) C) Prednisone (Prednisone), guaifenesin (Organidin), ketorolac (Toradol), pyrazinamide (Zinamide) D) Salmeterol (Serevent), cromolyn sodium (Intal), dexamethasone (Decadron), isoniazid (INH) (Chp. 31; elsevier resources)

A) Isoniazid (INH), rifampin (Rifadin), pyrazinamide (Zinamide), ethambutol (Myambutol) (Chp. 31; elsevier resources)

A client has recently been released from prison and has just tested positive for tuberculosis (TB). What teaching points does the community health nurse want to stress for this client regarding medications? Select all that apply. A) Not taking the medication could lead to an infection that is difficult to treat or to total drug resistance. B) The medications may cause nausea. The client should take them at bedtime. C) The client is generally not contagious after 2 to 3 consecutive weeks of treatment. D) These medications must be taken for 2 years. E) These medications may cause kidney failure. (Chp. 31; elsevier resources)

A) Not taking the medication could lead to an infection that is difficult to treat or to total drug resistance. B) The medications may cause nausea. The client should take them at bedtime. (Chp. 31; elsevier resources)

A client has been admitted to the hospital with suspected TB. What drugs should the nurse plan to teach the client about before discharge? Select all that apply. A) Rifampin (Rifadin); contact lenses can become stained orange B) Isoniazid (INH); report yellowing of the skin or darkened urine C) Pyrazinamide (PZA); maintain a fluid restriction of 1200 mL/day D) Ethambutol (Myambutol); report any changes in vision E) Amoxicillin (Amoxil); take this drug with food or milk (Chp. 31; p. 598)

A) Rifampin (Rifadin); contact lenses can become stained orange B) Isoniazid (INH); report yellowing of the skin or darkened urine D) Ethambutol (Myambutol); report any changes in vision (Chp. 31; p. 598)

A nurse is caring for an 89-year-old client admitted with pneumonia. He has an IV of normal saline running at 100 mL/hr and antibiotics that were initiated in the emergency department 3 hours ago. He has oxygen at 2 liters/nasal cannula. What assessment finding by the nurse indicates that goals for a priority diagnosis have been met for this client? A) The client is alert and oriented to person, place, and time. B) Blood pressure is within normal limits and client's baseline. C) Skin behind the ears demonstrates no redness or irritation. D) Urine output has been >30 mL/hr per Foley catheter. (Chp. 31; p. 593)

A) The client is alert and oriented to person, place, and time. (Chp. 31; p. 593)

7. 7. People involved in which occupations or activities are encouraged to wear masks and to have adequate ventilations? Select all that apply. A. Bakers B. Coal miners C. Electricians D. Furniture refinishers E. Plumbers F. Potters

A, B, D, F

17. What are the goals of drug therapy in the treatment of asthma? (Select all that apply.) A. Drugs are used to reduce the asthma response. B. Weekly drugs are used to reduce the asthma response. C. Combination drugs are avoided in the treatment of asthma. D. Some patients only require drug therapy during an asthma episode. E. Drugs are used to change airway responsiveness.

A, D, E

The spouse of the client with Alzheimer's disease (AD) is listening to the hospice nurse explaining the client's drug regimen. Which statement by the spouse indicates an understanding of the nurse's instruction? A. "Donepezil (Aricept) will treat the symptoms of Alzheimer's disease." B. "Memantine (Namenda)is indicated for treatment of early symptoms of Alzheimer's disease. C. "Rivastigmine (Excelon) is used to treat depression." D. "Sertraline (Zoloft) will treat the symptoms of Alzheimer's disease."

A. "Donepezil (Aricept) will treat the symptoms of Alzheimer's disease." Rationale A. Cholinesterase inhibitors (e.g., donepezil) are approved for the symptomatic treatment of Alzheimer's disease. B. Memantine (Namenda) is indicated for advanced Alzheimer's disease. C. Rivastigmine (Excelon) is a cholinesterase inhibitor that is used to treat Alzheimer's symptoms. Selective serotonin reuptake inhibitors (SSRIs) are antidepressants and may be used in Alzheimer's clients who develop depression. D. Some clients with Alzheimer's disease experience depression and may be treated with antidepressants such as sertraline.

The nurse's friend fears that his mother is getting old, saying that she is becoming extremely forgetful and disoriented and is beginning to wander. What is the nurse's best response? A. "Have you taken her for a checkup?" B. "She has Alzheimer's disease." C. "That is a normal part of aging." D. "You should look into respite care."

A. "Have you taken her for a checkup?" Rationale A. The mother's symptoms indicate possible Alzheimer's disease or some other physiologic imbalance, and she should be assessed further by a physician. B. The nurse cannot make this diagnosis. The mother should be formally assessed by a physician. C. The mother's behavior is not normal age-related behavior. D. Respite care is for caregivers, not for clients.

The client has been diagnosed with Huntington disease. The nurse is teaching the client and her parents about the genetic aspects of the disease. Which statement made by the parents demonstrates a good understanding of the nurse's teaching? A. "If she has children, she'll pass the gene on to her kids." B. "She could only have gotten the disease from both of us." C. "Because she got the gene from her father, she'll live longer than other people with the disease." D. "More testing should definitely be done to see if she's really got the gene."

A. "If she has children, she'll pass the gene on to her kids." Rationale A. An autosomal dominant trait with high penetrance, such as Huntington disease, means that a person who inherits just one mutated allele has an almost 100% chance of developing the disease. B. Only one defective gene is needed to inherit Huntington disease. The client could have inherited it from her father or mother. C. If the client inherited the gene from her mother, she would live a longer life than other people with the disease. If she inherited the gene from her father, her life would be shorter. D. Additional testing is not necessary. If the client has Huntington disease, then the client has the gene.

The parents of a young child report that their child sometimes stares blankly into space for just a few seconds and then gets very tired. The nurse anticipates that the child will be assessed for which seizure disorder? A. Absence B. Myoclonic C. Simple partial D. Tonic

A. Absence Rationale A. Absence seizures are more common in children and consist of brief (often just seconds) periods of loss of consciousness and blank staring, as though he or she is daydreaming. B. Myoclonic seizures are characterized by brief jerking or stiffening of the extremities, which may occur singly or in groups. C. Partial seizures are most often seen in adults. D. Tonic seizures are characterized by an abrupt increase in muscle tone, loss of consciousness, and autonomic changes lasting from 30 seconds to several minutes.

The client newly diagnosed with Parkinson disease is being discharged. Which instruction is best for the nurse to provide to the client's spouse? A. Administer medications promptly on schedule to maintain therapeutic drug levels. B. Complete activities of daily living for the client. C. Speak loudly for better understanding. D. Provide high-calorie, high-carbohydrate foods to maintain the client's weight.

A. Administer medications promptly on schedule to maintain therapeutic drug levels. Rationale A. This is a correct statement. B. The client should be encouraged to do as much as possible on his own. C. Slow speech rather than loud speech is more effective for the client with Parkinson disease. D. Small, frequent meals are more effective for the client with Parkinson disease.

A client with asthma reports shortness of breath. What is the nurse assessing when auscultating this client's chest? A. Adventitious breath sounds B. Fremitus C. Oxygenation status D. Respiratory excursion

A. Adventitious breath sounds Correct: Adventitious sounds are additional breath sounds superimposed on normal sounds. They indicate pathologic changes in the lung.

The client with a migraine is lying in a darkened room with a wet cloth on the head after receiving analgesic drugs. What will the nurse do next? A. Allow the client to remain undisturbed. B. Assess the client's vital signs. C. Remove the cloth because it can harbor microorganisms. D. Turn on the lights for a neurologic assessment.

A. Allow the client to remain undisturbed. Rationale A. At the beginning of a migraine attack, the client may be able to alleviate pain with analgesics and by lying down and darkening the room with a cool cloth on his or her forehead. If the client falls asleep, he or she should remain undisturbed until awakening. B. Assessing the client' vital signs will disturb the client unnecessarily. C. A cool cloth is helpful for the client with a migraine and does not present enough of a risk that it should be removed. D. This is not appropriate because light can cause the migraine to worsen.

The wife of the client with Alzheimer's disease mentions to the home health nurse that although she loves him, she is exhausted caring for her husband. What does the nurse suggest to alleviate caregiver stress? A. Arranges for respite care B. Provides positive reinforcement and support to the wife C. Restrains the client for a short time each day, to allow the wife to rest D. Teaches the client improved self-care

A. Arranges for respite care Rationale A. Respite care can give the wife some time to re-energize and will provide a social outlet for the client. B. Providing positive reinforcement and support is encouraging but does not help the wife's situation. C. Restraints are almost never appropriate and are used only as an absolute last resort. D. The client with Alzheimer's disease typically is unable to learn improved self-care.

The client is admitted with bacterial meningitis. Which nursing intervention is the highest priority for this client? A. Assessing neurologic status at least every 2 to 4 hours B. Decreasing environmental stimuli C. Managing pain through drug and nondrug methods D. Strict monitoring of hourly intake and output

A. Assessing neurologic status at least every 2 to 4 hours Rationale A. The most important nursing intervention for clients with meningitis is the accurate monitoring and recording of their neurologic status, vital signs, and vascular assessment. The client's neurologic status and vital signs should be assessed at least every 4 hours, or more often if clinically indicated. The priority for care is to monitor for early neurologic changes that may indicate increased intracranial pressure (ICP), such as decreased level of consciousness (LOC). B. Decreasing environmental stimuli is helpful for the client with bacterial meningitis but is not the highest priority. C. Clients with bacterial meningitis report severe headaches requiring pain management, but this is the second-highest priority. D. Assessing fluid balance while preventing overload is not the highest priority.

The client is being discharged to home with progressing stage I Alzheimer's disease. The family expresses concern to the nurse about caring for their parent. What is the priority for best continuity of care? A. Assigning a case manager B. Ensuring that all family questions are answered before discharge C. Providing a safe environment D. Referring the family to the Alzheimer's Association

A. Assigning a case manager Rationale A. Whenever possible, the client and family should be assigned a case manager who can assess their needs for health care resources and facilitate appropriate placement throughout the continuum of care. B. This is necessary for family support but is not relevant for continuity of care. C. This is necessary for safety but is not relevant for continuity of care. D. This is necessary for appropriate resource referral but is not relevant for continuity of care.

A client receiving sumatriptan (Imitrex) for migraine headaches is experiencing adverse effects after taking the drug. Which adverse effect is of greatest concern to the nurse? A. Chest tightness B. Skin flushing C. Tingling feelings D. Warm sensation

A. Chest tightness Rationale A. Triptan drugs are contraindicated in clients with coronary artery disease because they can cause arterial narrowing; the nurse should instruct the client to not take the medication until the nurse can talk with the prescribing health care provider. B. Skin flushing is a common adverse effect with triptan medications and is not an indication to avoid using this group of drugs. C. Tingling feelings is a common adverse effect with triptan medications and is not an indication to avoid using this group of drugs. D. A warm sensation is a common adverse effect with triptan medications and is not an indication to avoid using this group of drugs.

Four clients are sent back to the emergency department from triage at the same time. Which client requires the nurse's immediate attention? A. Client with acute allergic reaction B. Client with dyspnea on exertion C. Client with lung cancer with cough D. Client with sinus infection with fever

A. Client with acute allergic reaction Correct: An acute allergic reaction can lead to immediate respiratory distress. This is an emergent situation that requires the immediate attention of the nurse.

The RN has received report about all of these clients. Which client needs the most immediate assessment? A. Client with acute asthma who has an oxygen saturation of 89% by pulse oximetry B. Client admitted 3 hours ago for a scheduled thoracentesis in 30 minutes C. Client with bronchogenic lung cancer who returned from bronchoscopy 3 hours ago D. Client with pleural effusion who has decreased breath sounds at the right base

A. Client with acute asthma who has an oxygen saturation of 89% by pulse oximetry Correct: An oxygen saturation level less than 91% indicates hypoxemia and requires immediate assessment and intervention to improve blood and tissue oxygenation.

Which change in the cerebrospinal fluid (CSF) indicates to the nurse that the client may have bacterial meningitis? A. Cloudy, turbid CSF B. Decreased white blood cells C. Decreased protein D. Increased glucose

A. Cloudy, turbid CSF Rationale A. Cloudy, turbid cerebrospinal fluid is a sign of bacterial meningitis. Clear fluid is a sign of viral meningitis. B. Increased white blood cells is a sign of bacterial meningitis. C. Increased protein is a sign of bacterial meningitis. D. Decreased glucose is a sign of bacterial meningitis.

A client is admitted to the medical floor with a new diagnosis of lung cancer. How can the nurse assist the client initially with the anxiety associated with the new diagnosis? A. Encourage client to ask questions and verbalize concerns. B. Leave client alone to deal with his own feelings. C. Medicate client with diazepam (Valium) for anxiety every 8 hours. D. Provide journals about cancer treatment.

A. Encourage client to ask questions and verbalize concerns. Correct: Anxiety causes increased oxygen consumption. Oxygen availability is limited in lung cancer. The availability of the nurse to answer questions and listen to the client's concerns will decrease anxiety.

The client with Parkinson disease is being discharged home with his wife. To ensure compliance with the management plan, which discharge action is most effective? A. Involving the client and his wife in developing a plan of care B. Setting up visitations by a home health nurse C. Telling his wife what the client needs D. Writing up a detailed plan of care according to standards

A. Involving the client and his wife in developing a plan of care Rationale A. Involving the client and spouse in developing a plan of care is the best way to ensure compliance. B. Home health nurse visitations are generally helpful but may not be needed for this client. C. Instructing the spouse about the client's needs does not reinforce the spouse's involvement and buy-in with the management plan. D. Providing the spouse with a written plan of care does not reinforce the spouse's involvement and buy-in with the management plan.

The client with early-stage Alzheimer's disease is admitted to the surgical unit for a biopsy. Which client problem is the priority? A. Potential for injury related to chronic confusion and physical deficits B. Risk for reduced mobility related to progression of disability C. Potential for skin breakdown related to immobility and/or impaired nutritional status D. Lack of social contact related to personality and behavior changes

A. Potential for injury related to chronic confusion and physical deficits Rationale A. The priority for interdisciplinary care is safety. Chronic confusion and physical deficits place the client with Alzheimer's disease at high risk for injury. B. This is not a priority for a short hospital stay. This problem is usually the result of long-term care. C. This is not a priority for a short hospital stay. This problem is usually the result of long-term care. D. This is not a priority for a short hospital stay. This problem is usually the result of long-term care.

The nurse instructs a client on how to correctly use an inhaler with a spacer. In which order should these steps occur? 1. "Press down firmly on the canister to release one dose of medication." 2. "Breathe in slowly and deeply." 3. "Shake the whole unit vigorously three or four times." 4. "Insert the mouthpiece of the inhaler into the nonmouthpiece end of the spacer." 5. "Place the mouthpiece into your mouth, over the tongue, and seal your lips tightly around the mouthpiece." 6. "Remove the mouthpiece from your mouth, keep your lips closed, and hold your breath for at least 10 seconds." a. 2, 3, 4, 5, 6, 1 b. 3, 4, 5, 1, 6, 2 c. 4, 3, 5, 1, 2, 6 d. 5, 3, 6, 1, 2, 4

ANS: C The proper order for correctly using an inhaler with a spacer is as follows. Insert the mouthpiece of the inhaler into the nonmouthpiece end of the spacer. Shake the whole unit vigorously three or four times. Place the mouthpiece into the mouth, over the tongue, and seal the lips tightly around it. Press down firmly on the canister of the inhaler to release one dose of medication into the spacer. Breathe in slowly and deeply. Remove the mouthpiece from the mouth, and, keeping the lips closed, hold the breath for at least 10 seconds. Then breathe out slowly. Wait at least 1 minute between puffs.

Which nursing intervention is appropriate when caring for a client with Alzheimer's disease? A. Provide a large clock and calendar. B. Place the client in a geri-chair to prevent wandering. C. Insert a urinary catheter to prevent incontinence. D. Place the client in the nurse's station.

A. Provide a large clock and calendar. Rationale A. Providing a large clock and calendar may stimulate cognition. The nurse will also be able to use these tools to orient the client to date and time. The clock and calendar should not be abstract, because this may frighten the client. The purpose of cognitive stimulation and memory training is to reinforce or promote desirable cognitive function and facilitate memory. Reference: p. 953, Safe and Effective Care Environment B. Placing the client in a geri-chair is considered a physical restraint and should be avoided. Reference: p. 953, Safe and Effective Care Environment C. The client should be placed on a toileting schedule, but an indwelling catheter is not necessary. Removing the catheter while the balloon remains inflated may increase the client's risk for injury. A foreign object such as a catheter may also increase the client's confusion and agitation. Reference: p. 953, Safe and Effective Care Environment D. The client should not be placed near the nursing station because of the level of noise and activity. In addition to disturbed sleep, other negative effects of high noise levels include decreased nutritional intake, changes in blood pressure and pulse rates, and feelings of increased stress and anxiety. The client with AD is especially susceptible to these changes and must have as much undisturbed sleep at night as possible. Fatigue increases confusion and behavioral manifestations such as agitation and aggressiveness. Reference: p. 953, Safe and Effective Care Environment

A nurse cares for a client with a 40-year smoking history who is experiencing distended neck veins and dependent edema. Which physiologic process should the nurse correlate with this client's history and clinical manifestations? a. Increased pulmonary pressure creating a higher workload on the right side of the heart b. Exposure to irritants resulting in increased inflammation of the bronchi and bronchioles c. Increased number and size of mucus glands producing large amounts of thick mucus d. Left ventricular hypertrophy creating a decrease in cardiac output

ANS: A Smoking increases pulmonary hypertension, resulting in cor pulmonale, or right-sided heart failure. Increased pressures in the lungs make it more difficult for blood to flow through the lungs. Blood backs up into the right side of the heart and then into the peripheral venous system, creating distended neck veins and dependent edema. Inflammation in bronchi and bronchioles creates an airway obstruction which manifests as wheezes. Thick mucus in the lungs has no impact on distended neck veins and edema. Left ventricular hypertrophy is associated with left heart failure and is not caused by a 40-year smoking history.

A nurse auscultates a client's lung fields. Which action should the nurse take based on the lung sounds? (Click the media button to hear the audio clip.) a. Assess for airway obstruction. b. Initiate oxygen therapy. c. Assess vital signs. d. Elevate the client's head.

ANS: A Stridor is the sound heard, and it indicates severe airway constriction. The nurse must administer a bronchodilator to get air into the lungs. Administering oxygen, assessing vital signs, and elevating the client's head will not help until the client's airways are open.

A nurse assesses a client who is prescribed fluticasone (Flovent) and notes oral lesions. Which action should the nurse take? a. Encourage oral rinsing after fluticasone administration. b. Obtain an oral specimen for culture and sensitivity. c. Start the client on a broad-spectrum antibiotic. d. Document the finding as a known side effect.

ANS: A The drug reduces local immunity and increases the risk for local infection, especially Candida albicans. Rinsing the mouth after using the inhaler will decrease the risk for developing this infection. Use of mouthwash and broad-spectrum antibiotics is not warranted in this situation. The nurse should document the finding, but the best action to take is to have the client start rinsing his or her mouth after using fluticasone. An oral specimen for culture and sensitivity will not provide information necessary to care for this client.

The nurse instructs a client on the steps needed to obtain a peak expiratory flow rate. In which order should these steps occur? 1. "Take as deep a breath as possible." 2. "Stand up (unless you have a physical disability)." 3. "Place the meter in your mouth, and close your lips around the mouthpiece." 4. "Make sure the device reads zero or is at base level." 5. "Blow out as hard and as fast as possible for 1 to 2 seconds." 6. "Write down the value obtained." 7. "Repeat the process two additional times, and record the highest number in your chart." a. 4, 2, 1, 3, 5, 6, 7 b. 3, 4, 1, 2, 5, 7, 6 c. 2, 1, 3, 4, 5, 6, 7 d. 1, 3, 2, 5, 6, 7, 4

ANS: A The proper order for obtaining a peak expiratory flow rate is as follows. Make sure the device reads zero or is at base level. The client should stand up (unless he or she has a physical disability). The client should take as deep a breath as possible, place the meter in the mouth, and close the lips around the mouthpiece. The client should blow out as hard and as fast as possible for 1 to 2 seconds. The value obtained should be written down. The process should be repeated two more times, and the highest of the three numbers should be recorded in the client's chart.

A nurse plans care for a client who is experiencing dyspnea and must stop multiple times when climbing a flight of stairs. Which intervention should the nurse include in this client's plan of care? a. Assistance with activities of daily living b. Physical therapy activities every day c. Oxygen therapy at 2 liters per nasal cannula d. Complete bedrest with frequent repositioning

ANS: A A client with dyspnea and difficulty completing activities such as climbing a flight of stairs has class III dyspnea. The nurse should provide assistance with activities of daily living. These clients should be encouraged to participate in activities as tolerated. They should not be on complete bedrest, may not be able to tolerate daily physical therapy, and only need oxygen if hypoxia is present.

An emergency department nurse triages a client with diabetes mellitus who has fractured her arm. Which action should the nurse take first? a. Remove the medical alert bracelet from the fractured arm. b. Immobilize the arm by splinting the fractured site. c. Place the client in a supine position with a warm blanket. d. Cover any open areas with a sterile dressing.

ANS: A A client's medical alert bracelet should be removed from the fractured arm before the affected extremity swells. Immobilization, positioning, and dressing should occur after the bracelet is removed.

A nurse admits a client from the emergency department. Client data are listed below: 70 years of age History of diabetes On insulin twice a day Reports new-onset dyspnea and productive cough Crackles and rhonchi heard throughout the lungs Dullness to percussion LLL Afebrile Oriented to person only WBC: 5,200/mm3 PaO2 on room air 65 mm Hg What action by the nurse is the priority? a. Administer oxygen at 4 liters per nasal cannula. b. Begin broad-spectrum antibiotics. c. Collect a sputum sample for culture. d. Start an IV of normal saline at 50 mL/hr.

ANS: A All actions are appropriate for this client who has manifestations of pneumonia. However, airway and breathing come first, so begin oxygen administration and titrate it to maintain saturations greater than 95%. Start the IV and collect a sputum culture, and then begin antibiotics.

13. A nurse cares for a client admitted from a nursing home after several recent falls. What prescription should the nurse complete first? a. Obtain urine sample for culture and sensitivity. b. Administer intravenous antibiotics. c. Encourage protein intake and additional fluids. d. Consult physical therapy for gait training.

ANS: A Although all interventions are or might be important, obtaining a urine sample for urinalysis takes priority. Often urinary tract infection (UTI) symptoms in older adults are atypical, and a UTI may present with new onset of confusion or falling. The urine sample should be obtained before starting antibiotics. Dietary Test Bank - Medical-Surgical Nursing: Concepts for Interprofessional Collaborative Care 9e 560 requirements and gait training should be implemented after obtaining the urine sample. DIF: Applying/Application REF: 1364 KEY: Cystitis| assessment/diagnostic examination| older adult MSC: Integrated Process: Nursing Process: Implementation NOT: Client Needs Category: Safe and Effective Care Environment: Management of Care

A nurse cares for a client with an increased blood urea nitrogen (BUN)/creatinine ratio. Which action should the nurse take first? a. Assess the client's dietary habits. b. Inquire about the use of nonsteroidal anti-inflammatory drugs (NSAIDs). c. Hold the client's metformin (Glucophage). d. Contact the health care provider immediately.

ANS: A An elevated BUN/creatinine ratio is often indicative of dehydration, urinary obstruction, catabolism, or a high-protein diet. The nurse should inquire about the client's dietary habits. Kidney damage related to NSAID use most likely would manifest with elevations in both BUN and creatinine, but no change in the ratio. The nurse should obtain more assessment data before holding any medications or contacting the provider.

A home health nurse assesses a client with diabetes who has a new cast on the arm. The nurse notes the client's fingers are pale, cool, and slightly swollen. Which action should the nurse take first? a. Raise the arm above the level of the heart. b. Encourage range of motion. c. Apply heat to the affected hand. d. Bivalve the cast to decrease pressure.

ANS: A Arm casts can impair circulation when the arm is in the dependent position. The nurse should immediately elevate the arm above the level of the heart, ensuring that the hand is above the elbow, and should re-assess the extremity in 15 minutes. If the fingers are warmer and less swollen, the cast is not too tight and adjustments do not need to be made, but a sling should be worn when the client is upright. Encouraging range of motion would not assist the client as much as elevating the arm. Heat would cause increased edema and should not be used. If the cast is confirmed to be too tight, it could be bivalved.

A client is distressed at body changes related to kyphosis. What response by the nurse is best? a. Ask the client to explain more about these feelings. b. Explain that these changes are irreversible. c. Offer to help select clothes to hide the deformity. d. Tell the client safety is more important than looks.

ANS: A Assessment is the first step of the nursing process, and the nurse should begin by getting as much information about the client's feelings as possible. Explaining that the changes are irreversible discounts the client's feelings. Depending on the extent of the deformity, clothing will not hide it. While safety is more objectively important than looks, the client is worried about looks and the nurse needs to address this issue.

A nurse is assessing an older client and discovers back pain with tenderness along T2 and T3. What action by the nurse is best? a. Consult with the provider about an x-ray. b. Encourage the client to use ibuprofen (Motrin). c. Have the client perform hip range of motion. d. Place the client in a rigid cervical collar.

ANS: A Back pain with tenderness is indicative of a spinal compression fracture, which is the most common type of osteoporotic fracture. The nurse should consult the provider about an x-ray. Motrin may be indicated but not until there is a diagnosis. Range of motion of the hips is not related, although limited spinal range of motion may be found with a vertebral compression fracture. Since the defect is in the thoracic spine, a cervical collar is not needed.

A nurse in a family practice clinic is preparing discharge instructions for a client reporting facial pain that is worse when bending over, tenderness across the cheeks, and postnasal discharge. What instruction will be most helpful? a. "Ice packs may help with the facial pain." b. "Limit fluids to dry out your sinuses." c. "Try warm, moist heat packs on your face." d. "We will schedule you for a computed tomography scan this week."

ANS: C This client has rhinosinusitis. Comfort measures for this condition include breathing in warm steam, hot packs, nasal saline irrigations, sleeping with the head elevated, increased fluids, and avoiding cigarette smoke. The client does not need a CT scan.

A nurse is caring for four clients. After the hand-off report, which client does the nurse see first? a. Client with osteoporosis and a white blood cell count of 27,000/mm3 b. Client with osteoporosis and a bone fracture who requests pain medication c. Post-microvascular bone transfer client whose distal leg is cool and pale d. Client with suspected bone tumor who just returned from having a spinal CT

ANS: C This client is the priority because the assessment findings indicate a critical lack of perfusion. A high white blood cell count is an expected finding for the client with osteoporosis. The client requesting pain medication should be seen second. The client who just returned from a CT scan is stable and needs no specific postprocedure care.

10. After teaching a client with a history of renal calculi, the nurse assesses the clients understanding. Which statement made by the client indicates a correct understanding of the teaching? a. I should drink at least 3 liters of fluid every day. b. I will eliminate all dairy or sources of calcium from my diet. c. Aspirin and aspirin-containing products can lead to stones. d. The doctor can give me antibiotics at the first sign of a stone.

ANS: A Dehydration contributes to the precipitation of minerals to form a stone. Although increased intake of calcium causes hypercalcemia and leads to excessive calcium filtered into the urine, if the client is well hydrated the calcium will be excreted without issues. Dehydration increases the risk for supersaturation of calcium in the urine, which contributes to stone formation. The nurse should encourage the client to drink more fluids, not decrease calcium intake. Ingestion of aspirin or aspirin-containing products does not cause a stone. Antibiotics neither prevent nor treat a stone. DIF: Applying/Application REF: 1361 KEY: Urolithiasis| hydration MSC: Integrated Process: Nursing Process: Evaluation NOT: Client Needs Category: Health Promotion and Maintenance

5. A nurse evaluates a client with acute glomerulonephritis (GN). Which manifestation should the nurse recognize as a positive response to the prescribed treatment? a. The client has lost 11 pounds in the past 10 days. b. The client's urine specific gravity is 1.048. c. No blood is observed in the client's urine. d. The client's blood pressure is 152/88 mm Hg.

ANS: A Fluid retention is a major feature of acute GN. This weight loss represents fluid loss, indicating that the glomeruli are performing the function of filtration. A urine specific gravity of 1.048 is high. Blood is not usually seen in GN, so this finding would be expected. A blood pressure of 152/88 mm Hg is too high; this may indicate kidney damage or fluid overload.

6. A nurse cares for adult clients who experience urge incontinence. For which client should the nurse plan a habit training program? a. A 78-year-old female who is confused b. A 65-year-old male with diabetes mellitus c. A 52-year-old female with kidney failure d. A 47-year-old male with arthritis

ANS: A For a bladder training program to succeed in a client with urge incontinence, the client must be alert, aware, and able to resist the urge to urinate. Habit training will work best for a confused client. This includes going to the bathroom (or being assisted to the bathroom) at set times. The other clients may benefit from another type of bladder training. DIF: Applying/Application REF: 1357 KEY: Urinary incontinence| health screening MSC: Integrated Process: Nursing Process: Assessment NOT: Client Needs Category: Physiological Integrity: Physiological Adaptation

What information does the nurse teach a women's group about osteoporosis? a. "For 5 years after menopause you lose 2% of bone mass yearly." b. "Men actually have higher rates of the disease but are underdiagnosed." c. "There is no way to prevent or slow osteoporosis after menopause." d. "Women and men have an equal chance of getting osteoporosis."

ANS: A For the first 5 years after menopause, women lose about 2% of their bone mass each year. Men have a slower loss of bone after the age of 75. Many treatments are now available for women to slow osteoporosis after menopause.

The emergency department (ED) manager is reviewing client charts to determine how well the staff performs when treating clients with community-acquired pneumonia. What outcome demonstrates that goals for this client type have been met? a. Antibiotics started before admission b. Blood cultures obtained within 20 minutes c. Chest x-ray obtained within 30 minutes d. Pulse oximetry obtained on all clients

ANS: A Goals for treatment of community-acquired pneumonia include initiating antibiotics prior to inpatient admission or within 6 hours of presentation to the ED. Timely collection of blood cultures, chest x-ray, and pulse oximetry are important as well but do not coincide with established goals.

A nurse teaches a client who is diagnosed with diabetes mellitus. Which statement should the nurse include in this client's plan of care to delay the onset of microvascular and macrovascular complications? a. "Maintain tight glycemic control and prevent hyperglycemia." b. "Restrict your fluid intake to no more than 2 liters a day." c. "Prevent hypoglycemia by eating a bedtime snack." d. "Limit your intake of protein to prevent ketoacidosis."

ANS: A Hyperglycemia is a critical factor in the pathogenesis of long-term diabetic complications. Maintaining tight glycemic control will help delay the onset of complications. Restricting fluid intake is not part of the treatment plan for clients with diabetes. Preventing hypoglycemia and ketosis, although important, are not as important as maintaining daily glycemic control.

After teaching a client who is recovering from pancreas transplantation, the nurse assesses the client's understanding. Which statement made by the client indicates a need for additional education? a. "If I develop an infection, I should stop taking my corticosteroid." b. "If I have pain over the transplant site, I will call the surgeon immediately." c. "I should avoid people who are ill or who have an infection." d. "I should take my cyclosporine exactly the way I was taught."

ANS: A Immunosuppressive agents should not be stopped without the consultation of the transplantation physician, even if an infection is present. Stopping immunosuppressive therapy endangers the transplanted organ. The other statements are correct. Pain over the graft site may indicate rejection. Anti-rejection drugs cause immunosuppression, and the client should avoid crowds and people who are ill. Changing the routine of anti-rejection medications may cause them to not work optimally.

A nurse prepares to administer insulin to a client at 1800. The client's medication administration record contains the following information: • Insulin glargine: 12 units daily at 1800 • Regular insulin: 6 units QID at 0600, 1200, 1800, 2400 Based on the client's medication administration record, which action should the nurse take? a. Draw up and inject the insulin glargine first, and then draw up and inject the regular insulin. b. Draw up and inject the insulin glargine first, wait 20 minutes, and then draw up and inject the regular insulin. c. First draw up the dose of regular insulin, then draw up the dose of insulin glargine in the same syringe, mix, and inject the two insulins together. d. First draw up the dose of insulin glargine, then draw up the dose of regular insulin in the same syringe, mix, and inject the two insulins together.

ANS: A Insulin glargine must not be diluted or mixed with any other insulin or solution. Mixing results in an unpredictable alteration in the onset of action and time to peak action. The correct instruction is to draw up and inject first the glargine and then the regular insulin right afterward.

An emergency department nurse assesses a client with ketoacidosis. Which clinical manifestation should the nurse correlate with this condition? a. Increased rate and depth of respiration b. Extremity tremors followed by seizure activity c. Oral temperature of 102° F (38.9° C) d. Severe orthostatic hypotension

ANS: A Ketoacidosis decreases the pH of the blood, stimulating the respiratory control areas of the brain to buffer the effects of increasing acidosis. The rate and depth of respiration are increased (Kussmaul respirations) in an attempt to excrete more acids by exhalation. Tremors, elevated temperature, and orthostatic hypotension are not associated with ketoacidosis.

5. After teaching a client who has stress incontinence, the nurse assesses the clients understanding. Which statement made by the client indicates a need for additional teaching? a. I will limit my total intake of fluids. b. I must avoid drinking alcoholic beverages. c. I must avoid drinking caffeinated beverages. d. I shall try to lose about 10% of my body weight.

ANS: A Limiting fluids concentrates urine and can irritate tissues, leading to increased incontinence. Many people try to manage incontinence by limiting fluids. Alcoholic and caffeinated beverages are bladder stimulants. Obesity increases intra-abdominal pressure, causing incontinence. DIF: Applying/Application REF: 1357 KEY: Cystitis| hydration MSC: Integrated Process: Teaching/Learning NOT: Client Needs Category: Physiological Integrity: Basic Care and Comfort

A nurse reviews prescriptions for an 82-year-old client with a fractured left hip. Which prescription should alert the nurse to contact the provider and express concerns for client safety? a. Meperidine (Demerol) 50 mg IV every 4 hours b. Patient-controlled analgesia (PCA) with morphine sulfate c. Percocet 2 tablets orally every 6 hours PRN for pain d. Ibuprofen elixir every 8 hours for first 2 days

ANS: A Meperidine (Demerol) should not be used for older adults because it has toxic metabolites that can cause seizures. The nurse should question this prescription. The other prescriptions are appropriate for this client's pain management.

A nurse cares for a client with diabetes mellitus who is prescribed metformin (Glucophage) and is scheduled for an intravenous urography. Which action should the nurse take first? a. Contact the provider and recommend discontinuing the metformin. b. Keep the client NPO for at least 6 hours prior to the examination. c. Check the client's capillary artery blood glucose and administer prescribed insulin. d. Administer intravenous fluids to dilute and increase the excretion of dye.

ANS: A Metformin can cause lactic acidosis and renal impairment as the result of an interaction with the dye. This drug must be discontinued for 48 hours before the procedure and not started again after the procedure until urine output is well established. The client's health care provider needs to provide alternative therapy for the client until the metformin can be resumed. Keeping the client NPO, checking the client's blood glucose, and administering intravenous fluids should be part of the client's plan of care, but are not the priority, as the examination should not occur while the client is still taking metformin.

A nurse cares for a client who has a family history of diabetes mellitus. The client states, "My father has type 1 diabetes mellitus. Will I develop this disease as well?" How should the nurse respond? a. "Your risk of diabetes is higher than the general population, but it may not occur." b. "No genetic risk is associated with the development of type 1 diabetes mellitus." c. "The risk for becoming a diabetic is 50% because of how it is inherited." d. "Female children do not inherit diabetes mellitus, but male children will."

ANS: A Risk for type 1 diabetes is determined by inheritance of genes coding for HLA-DR and HLA-DQ tissue types. Clients who have one parent with type 1 diabetes are at increased risk for its development. Diabetes (type 1) seems to require interaction between inherited risk and environmental factors, so not everyone with these genes develops diabetes. The other statements are not accurate.

A nurse cares for a client placed in skeletal traction. The client asks, "What is the primary purpose of this type of traction?" How should the nurse respond? a. "Skeletal traction will assist in realigning your fractured bone." b. "This treatment will prevent future complications and back pain." c. "Traction decreases muscle spasms that occur with a fracture." d. "This type of traction minimizes damage as a result of fracture treatment."

ANS: A Skeletal traction pins or screws are surgically inserted into the bone to aid in bone alignment. As a last resort, traction can be used to relieve pain, decrease muscle spasm, and prevent or correct deformity and tissue damage. These are not primary purposes of skeletal traction.

A nurse cares for a client with a 40-year smoking history who is experiencing distended neck veins and dependent edema. Which physiologic process should the nurse correlate with this client's history and clinical manifestations? a. Increased pulmonary pressure creating a higher workload on the right side of the heart b. Exposure to irritants resulting in increased inflammation of the bronchi and bronchioles c. Increased number and size of mucus glands producing large amounts of thick mucus d. Left ventricular hypertrophy creating a decrease in cardiac output

ANS: A Smoking increases pulmonary hypertension, resulting in cor pulmonale, or right-sided heart failure. Increased pressures in the lungs make it more difficult for blood to flow through the lungs. Blood backs up into the right side of the heart and then into the peripheral venous system, creating distended neck veins and dependent edema. Inflammation in bronchi and bronchioles creates an airway obstruction which manifests as wheezes. Thick mucus in the lungs has no impact on distended neck veins and edema. Left ventricular hypertrophy is associated with left heart failure and is not caused by a 40-year smoking history.

16. A nurse obtains the health history of a client with a suspected diagnosis of bladder cancer. Which question should the nurse ask when determining this clients risk factors? a. Do you smoke cigarettes? b. Do you use any alcohol? c. Do you use recreational drugs? d. Do you take any prescription drugs?

ANS: A Smoking is known to be a factor that greatly increases the risk of bladder cancer. Alcohol use, recreational drug use, and prescription drug use (except medications that contain phenacetin) are not known to increase the risk of developing bladder cancer. DIF: Applying/Application REF: 1365 KEY: Urothelial cancer| health screening MSC: Integrated Process: Nursing Process: Assessment NOT: Client Needs Category: Safe and Effective Care Environment: Management of Care

A client admitted for pneumonia has been tachypneic for several days. When the nurse starts an IV to give fluids, the client questions this action, saying "I have been drinking tons of water. How am I dehydrated?" What response by the nurse is best? a. "Breathing so quickly can be dehydrating." b. "Everyone with pneumonia is dehydrated." c. "This is really just to administer your antibiotics." d. "Why do you think you are so dehydrated?"

ANS: A Tachypnea and mouth breathing, both seen in pneumonia, increase insensible water loss and can lead to a degree of dehydration. The other options do not give the client useful information.

After delegating to an unlicensed assistive personnel (UAP) the task of completing a bladder scan examination for a client, the nurse evaluates the UAP's performance. Which action by the UAP indicates the nurse must provide additional instructions when delegating this task? a. Selecting the female icon for all female clients and male icon for all male clients b. Telling the client, "This test measures the amount of urine in your bladder." c. Applying ultrasound gel to the scanning head and removing it when finished d. Taking at least two readings using the aiming icon to place the scanning head

ANS: A The UAP should use the female icon for women who have not had a hysterectomy. This allows the scanner to subtract the volume of the uterus from readings. If a woman has had a hysterectomy, the UAP should choose the male icon. The UAP should explain the procedure to the client, apply gel to the scanning head and clean it after use, and take at least two readings.

After teaching a client with diabetes mellitus to inject insulin, the nurse assesses the client's understanding. Which statement made by the client indicates a need for additional teaching? a. "The lower abdomen is the best location because it is closest to the pancreas." b. "I can reach my thigh the best, so I will use the different areas of my thighs." c. "By rotating the sites in one area, my chance of having a reaction is decreased." d. "Changing injection sites from the thigh to the arm will change absorption rates."

ANS: A The abdominal site has the fastest rate of absorption because of blood vessels in the area, not because of its proximity to the pancreas. The other statements are accurate assessments of insulin administration.

A nurse assesses an older adult client who was admitted 2 days ago with a fractured hip. The nurse notes that the client is confused and restless. The client's vital signs are heart rate 98 beats/min, respiratory rate 32 breaths/min, blood pressure 132/78 mm Hg, and SpO2 88%. Which action should the nurse take first? a. Administer oxygen via nasal cannula. b. Re-position to a high-Fowler's position. c. Increase the intravenous flow rate. d. Assess response to pain medications.

ANS: A The client is at high risk for a fat embolism and has some of the clinical manifestations of altered mental status and dyspnea. Although this is a life-threatening emergency, the nurse should take the time to administer oxygen first and then notify the health care provider. Oxygen administration can reduce the risk for cerebral damage from hypoxia. The nurse would not restrain a client who is confused without further assessment and orders. Sitting the client in a high-Fowler's position will not decrease hypoxia related to a fat embolism. The IV rate is not related. Pain medication most likely would not cause the client to be restless.

18. A nurse teaches a young female client who is prescribed amoxicillin (Amoxil) for a urinary tract infection. Which statement should the nurse include in this clients teaching? a. Use a second form of birth control while on this medication. b. You will experience increased menstrual bleeding while on this drug. c. You may experience an irregular heartbeat while on this drug. d. Watch for blood in your urine while taking this medication.

ANS: A The client should use a second form of birth control because penicillin seems to reduce the effectiveness of estrogen-containing contraceptives. She should not experience increased menstrual bleeding, an irregular heartbeat, or blood in her urine while taking the medication. DIF: Understanding/Comprehension REF: 1348 KEY: Cystitis| medication safety MSC: Integrated Process: Teaching/Learning NOT: Client Needs Category: Physiological Integrity: Pharmacological and Parenteral Therapies

A nurse assesses a client with diabetes mellitus and notes the client only responds to a sternal rub by moaning, has capillary blood glucose of 33 g/dL, and has an intravenous line that is infiltrated with 0.45% normal saline. Which action should the nurse take first? a. Administer 1 mg of intramuscular glucagon. b. Encourage the client to drink orange juice. c. Insert a new intravenous access line. d. Administer 25 mL dextrose 50% (D50) IV push.

ANS: A The client's blood glucose level is dangerously low. The nurse needs to administer glucagon IM immediately to increase the client's blood glucose level. The nurse should insert a new IV after administering the glucagon and can use the new IV site for future doses of D50 if the client's blood glucose level does not rise. Once the client is awake, orange juice may be administered orally along with a form of protein such as a peanut butter.

A nurse assesses a client who is prescribed fluticasone (Flovent) and notes oral lesions. Which action should the nurse take? a. Encourage oral rinsing after fluticasone administration. b. Obtain an oral specimen for culture and sensitivity. c. Start the client on a broad-spectrum antibiotic. d. Document the finding as a known side effect.

ANS: A The drug reduces local immunity and increases the risk for local infection, especially Candida albicans. Rinsing the mouth after using the inhaler will decrease the risk for developing this infection. Use of mouthwash and broad-spectrum antibiotics is not warranted in this situation. The nurse should document the finding, but the best action to take is to have the client start rinsing his or her mouth after using fluticasone. An oral specimen for culture and sensitivity will not provide information necessary to care for this client.

A client with bone cancer is hospitalized for a limb salvage procedure. How can the nurse best address the client's psychosocial needs? a. Assess the client's coping skills and support systems. b. Explain that the surgery leads to a longer life expectancy. c. Refer the client to the social worker or hospital chaplain. d. Reinforce physical therapy to aid with ambulating normally.

ANS: A The first step in the nursing process is assessment. The nurse should assess coping skills and possible support systems that will be helpful in this client's treatment. Explaining that a limb salvage procedure will extend life does not address the client's psychosocial needs. Referrals may be necessary, but the nurse should assess first. Reinforcing physical therapy is also helpful but again does not address the psychosocial needs of the client.

A client had an arthroscopy 1 hour ago on the left knee. The nurse finds the left lower leg to be pale and cool, with 1+/4+ pedal pulses. What action by the nurse is best? a. Assess the neurovascular status of the right leg. b. Document the findings in the client's chart. c. Elevate the left leg on at least two pillows. d. Notify the provider of the findings immediately.

ANS: A The nurse should compare findings of the two legs as these findings may be normal for the client. If a difference is observed, the nurse notifies the provider. Documentation should occur after the nurse has all the data. Elevating the left leg will not improve perfusion if there is a problem.

23. A nurse assesses a client who presents with renal calculi. Which question should the nurse ask? a. Do any of your family members have this problem? b. Do you drink any cranberry juice? c. Do you urinate after sexual intercourse? d. Do you experience burning with urination?

ANS: A There is a strong association between family history and stone formation and recurrence. Nephrolithiasis is associated with many genetic variations; therefore, the nurse should ask whether other family members have also had renal stones. The other questions do not refer to renal calculi but instead are questions that should be Test Bank - Medical-Surgical Nursing: Concepts for Interprofessional Collaborative Care 9e 563 asked of a client with a urinary tract infection. DIF: Applying/Application REF: 1361 KEY: Urolithiasis| health screening MSC: Integrated Process: Nursing Process: Assessment NOT: Client Needs Category: Safe and Effective Care Environment: Management of Care

An emergency department nurse cares for a client who sustained a crush injury to the right lower leg. The client reports numbness and tingling in the affected leg. Which action should the nurse take first? a. Assess the pedal pulses. b. Apply oxygen by nasal cannula. c. Increase the IV flow rate. d. Loosen the traction.

ANS: A These symptoms represent early warning signs of acute compartment syndrome. In acute compartment syndrome, sensory deficits such as paresthesias precede changes in vascular or motor signs. If the nurse finds a decrease in pedal pulses, the health care provider should be notified as soon as possible. Vital signs need to be obtained to determine if oxygen and intravenous fluids are necessary. Traction, if implemented, should never be loosened without a provider's prescription.

A client is in the internal medicine clinic reporting bone pain. The client's alkaline phosphatase level is 180 units/L. What action by the nurse is most appropriate? a. Assess the client for leg bowing. b. Facilitate an oncology workup. c. Instruct the client on fluid restrictions. d. Teach the client about ibuprofen (Motrin).

ANS: A This client has manifestations of Paget's disease. The nurse should assess for other manifestations such as bowing of the legs. Other care measures can be instituted once the client has a confirmed diagnosis.

A client with osteoporosis is going home, where the client lives alone. What action by the nurse is best? a. Arrange a home safety evaluation. b. Ensure the client has a walker at home. c. Help the client look into assisted living. d. Refer the client to Meals on Wheels.

ANS: A This client has several risk factors that place him or her at a high risk for falling. The nurse should consult social work or home health care to conduct a home safety evaluation. The other options may or may not be needed based upon the client's condition at discharge.

A nurse assesses a client who has diabetes mellitus and notes the client is awake and alert, but shaky, diaphoretic, and weak. Five minutes after administering a half-cup of orange juice, the client's clinical manifestations have not changed. Which action should the nurse take next? a. Administer another half-cup of orange juice. b. Administer a half-ampule of dextrose 50% intravenously. c. Administer 10 units of regular insulin subcutaneously. d. Administer 1 mg of glucagon intramuscularly.

ANS: A This client is experiencing mild hypoglycemia. For mild hypoglycemic manifestations, the nurse should administer oral glucose in the form of orange juice. If the symptoms do not resolve immediately, the treatment should be repeated. The client does not need intravenous dextrose, insulin, or glucagon.

A hospitalized client's strength of the upper extremities is rated at 3. What does the nurse understand about this client's ability to perform activities of daily living (ADLs)? a. The client is able to perform ADLs but not lift some items. b. No difficulties are expected with ADLs. c. The client is unable to perform ADLs alone. d. The client would need near-total assistance with ADLs.

ANS: A This rating indicates fair muscle strength with full range of motion against gravity but not resistance. The client could complete ADLs independently unless they required lifting objects.

A school nurse is conducting scoliosis screening. In screening the client, what technique is most appropriate? a. Bending forward from the hips b. Sitting upright with arms outstretched c. Walking across the room and back d. Walking with both eyes closed

ANS: A To assess for scoliosis, a spinal deformity, the student should bend forward at the hips. Standing behind the student, the nurse looks for a lateral curve in the spine. The other actions are not correct.

A nurse is providing community education about preventing traumatic musculoskeletal injuries related to car crashes. Which group does the nurse target as the priority for this education? a. High school football team b. High school homeroom class c. Middle-aged men d. Older adult women

ANS: A Young men are at highest risk for musculoskeletal injury due to trauma, especially due to motor vehicle crashes. The high school football team, with its roster of young males, is the priority group.

A nurse prepares a client for lumbar puncture (LP). Which assessment finding should alert the nurse to contact the health care provider? a. Shingles on the clients back b. Client is claustrophobic c. Absence of intravenous access d. Paroxysmal nocturnal dyspnea

ANS: A An LP should not be performed if the client has a skin infection at or near the puncture site because of the risk of infection. A nurse would want to notify the health care provider if shingles were identified on the clients back. If a client has shortness of breath when lying flat, the LP can be adapted to meet the clients needs. Claustrophobia, absence of IV access, and paroxysmal nocturnal dyspnea have no impact on whether an LP can be performed.

A client is having a bone marrow biopsy and is extremely anxious. What action by the nurse is best? a. Assess client fears and coping mechanisms. b. Reassure the client this is a common test. c. Sedate the client prior to the procedure. d. Tell the client he or she will be asleep.

ANS: A Assessing the clients specific fears and coping mechanisms helps guide the nurse in providing holistic care that best meets the clients needs. Reassurance will be helpful but is not the best option. Sedation is usually used. The client may or may not be totally asleep during the procedure.

A nurse teaches a client who is scheduled for a positron emission tomography scan of the brain. Which statement should the nurse include in this clients teaching? a. Avoid caffeine-containing substances for 12 hours before the test. b. Drink at least 3 liters of fluid during the first 24 hours after the test. c. Do not take your cardiac medication the morning of the test. d. Remove your dentures and any metal before the test begins.

ANS: A Caffeine-containing liquids and foods are central nervous system stimulants and may alter the test results. No contrast is used; therefore, the client does not need to increase fluid intake. The client should take cardiac medications as prescribed. Metal does not have to be removed; this is done for magnetic resonance imaging.

A nurse assesses a client recovering from a cerebral angiography via the clients right femoral artery. Which assessment should the nurse complete? a. Palpate bilateral lower extremity pulses. b. Obtain orthostatic blood pressure readings. c. Perform a funduscopic examination. d. Assess the gag reflex prior to eating.

ANS: A Cerebral angiography is performed by threading a catheter through the femoral or brachial artery. The extremity is kept immobilized after the procedure. The nurse checks the extremity for adequate circulation by noting skin color and temperature, presence and quality of pulses distal to the injection site, and capillary refill. Clients usually are on bedrest; therefore, orthostatic blood pressure readings cannot be performed. The funduscopic examination would not be affected by cerebral angiography. The client is given analgesics but not conscious sedation; therefore, the clients gag reflex would not be compromised.

A nurse delegates care to the unlicensed assistive personnel (UAP). Which statement should the nurse include when delegating care for a client with cranial nerve II impairment? a. Tell the client where food items are on the breakfast tray. b. Place the client in a high-Fowlers position for all meals. c. Make sure the clients food is visually appetizing. d. Assist the client by placing the fork in the left hand.

ANS: A Cranial nerve II, the optic nerve, provides central and peripheral vision. A client who has cranial nerve II impairment will not be able to see, so the UAP should tell the client where different food items are on the meal tray. The other options are not appropriate for a client with cranial nerve II impairment.

A nurse performs an assessment of pain discrimination on an older adult client. The client correctly identifies, with eyes closed, a sharp sensation on the right hand when touched with a pin. Which action should the nurse take next? a. Touch the pin on the same area of the left hand. b. Contact the provider with the assessment results. c. Ask the client about current medications. d. Continue the assessment on the clients feet.

ANS: A If testing is begun on the right hand and the client correctly identifies the pain stimulus, the nurse should continue the assessment on the left hand. This is a normal finding and does not need to be reported to the provider, but instead documented in the clients chart. Medications do not need to be assessed in response to this finding. The nurse should assess the left hand prior to assessing the feet.

A client is receiving rivaroxaban (Xarelto) and asks the nurse to explain how it works. What response by the nurse is best? a. It inhibits thrombin. b. It inhibits fibrinogen. c. It thins your blood. d. It works against vitamin K.

ANS: A Rivaroxaban is a direct thrombin inhibitor. It does not work on fibrinogen or vitamin K. It is not a blood thinner, although many clients call anticoagulants by this name.

A nurse cares for a client who is recovering from a single-photon emission computed tomography (SPECT) with a radiopharmaceutical agent. Which statement should the nurse include when discussing the plan of care with this client? a. You may return to your previous activity level immediately. b. You are radioactive and must use a private bathroom. c. Frequent assessments of the injection site will be completed. d. We will be monitoring your renal functions closely.

ANS: A The client may return to his or her previous activity level immediately. Radioisotopes will be eliminated in the urine after SPECT, but no monitoring or special precautions are required. The injection site will not need to be assessed after the procedure is complete.

A nurse assesses a client who demonstrates a positive Rombergs sign with eyes closed but not with eyes open. Which condition does the nurse associate with this finding? a. Difficulty with proprioception b. Peripheral motor disorder c. Impaired cerebellar function d. Positive pronator drift

ANS: A The client who sways with eyes closed (positive Rombergs sign) but not with eyes open most likely has a disorder of proprioception and uses vision to compensate for it. The other options do not describe a positive Rombergs sign.

A nurse is assessing a dark-skinned client for pallor. What action is best? a. Assess the conjunctiva of the eye. b. Have the client open the hand widely. c. Look at the roof of the clients mouth. d. Palpate for areas of mild swelling.

ANS: A To assess pallor in dark-skinned people, assess the conjunctiva of the eye or the mucous membranes. Looking at the roof of the mouth can reveal jaundice. Opening the hand widely is not related to pallor, nor is palpating for mild swelling.

A nurse teaches a client who has chronic obstructive pulmonary disease. Which statements related to nutrition should the nurse include in this client's teaching? (Select all that apply.) a. "Avoid drinking fluids just before and during meals." b. "Rest before meals if you have dyspnea." c. "Have about six small meals a day." d. "Eat high-fiber foods to promote gastric emptying." e. "Increase carbohydrate intake for energy."

ANS: A, B, C Fluids can make a client feel bloated and should be avoided with meals. Resting before the meal will help a client with dyspnea. Six small meals a day also will help to decrease bloating. Fibrous foods can produce gas, which can cause abdominal bloating and can increase shortness of breath. The client should increase calorie and protein intake to prevent malnourishment. The client should not increase carbohydrate intake as this will increase carbon dioxide production and increase the client's risk of for acidosis.

A nurse teaches a client who has chronic obstructive pulmonary disease. Which statements related to nutrition should the nurse include in this client's teaching? (Select all that apply.) a. "Avoid drinking fluids just before and during meals." b. "Rest before meals if you have dyspnea." c. "Have about six small meals a day." d. "Eat high-fiber foods to promote gastric emptying." e. "Increase carbohydrate intake for energy."

ANS: A, B, C Fluids can make a client feel bloated and should be avoided with meals. Resting before the meal will help a client with dyspnea. Six small meals a day also will help to decrease bloating. Fibrous foods can produce gas, which can cause abdominal bloating and can increase shortness of breath. The client should increase calorie and protein intake to prevent malnourishment. The client should not increase carbohydrate intake as this will increase carbon dioxide production and increase the client's risk of for acidosis.

7. A nurse teaches a client about self-care after experiencing a urinary calculus treated by lithotripsy. Which statements should the nurse include in this clients discharge teaching? (Select all that apply.) a. Finish the prescribed antibiotic even if you are feeling better. b. Drink at least 3 liters of fluid each day. c. The bruising on your back may take several weeks to resolve. d. Report any blood present in your urine. e. It is normal to experience pain and difficulty urinating.

ANS: A, B, C The client should be taught to finish the prescribed antibiotic to ensure that he or she does not get a urinary tract infection. The client should drink at least 3 liters of fluid daily to dilute potential stone-forming crystals, prevent dehydration, and promote urine flow. After lithotripsy, the client should expect bruising that may take several weeks to resolve. The client should also experience blood in the urine for several days. The client should report any pain, fever, chills, or difficulty with urination to the provider as these may signal the beginning of an infection or the formation of another stone. DIF: Applying/Application REF: 1365 KEY: Urolithiasis| patient education MSC: Integrated Process: Teaching/Learning NOT: Client Needs Category: Health Promotion and Maintenance

A nurse is caring for a client who is prescribed a computed tomography (CT) scan with iodine-based contrast. Which actions should the nurse take to prepare the client for this procedure? (Select all that apply.) a. Ensure that an informed consent is present. b. Ask the client about any allergies. c. Evaluate the clients renal function. d. Auscultate bilateral breath sounds. e. Assess hematocrit and hemoglobin levels.

ANS: A, B, C A client who is scheduled to receive iodine-based contrast should be asked about allergies, especially allergies to iodine or shellfish. The clients kidney function should also be evaluated to determine if it is safe to administer contrast during the procedure. Finally, the nurse should ensure that an informed consent is present because all clients receiving iodine-based contrast must give consent. The CT will have no impact on the clients breath sounds or hematocrit and hemoglobin levels. Findings from these assessments will not influence the clients safety during the procedure.

A nurse coordinates care for a client with a wet plaster cast. Which statement should the nurse include when delegating care for this client to an unlicensed assistive personnel (UAP)? a. "Assess distal pulses for potential compartment syndrome." b. "Turn the client every 3 to 4 hours to promote cast drying." c. "Use a cloth-covered pillow to elevate the client's leg." d. "Handle the cast with your fingertips to prevent indentations."

ANS: C When delegating care to a UAP for a client with a wet plaster cast, the UAP should be directed to ensure that the extremity is elevated on a cloth pillow instead of a plastic pillow to promote drying. The client should be assessed for impaired arterial circulation, a complication of compartment syndrome; however, the nurse should not delegate assessments to a UAP. The client should be turned every 1 to 2 hours to allow air to circulate and dry all parts of the cast. Providers should handle the cast with the palms of the hands to prevent indentations.

A nurse plans care for an 83-year-old client who is experiencing age-related sensory perception changes. Which intervention should the nurse include in this clients plan of care? a. Provide a call button that requires only minimal pressure to activate. b. Write the date on the clients white board to promote orientation. c. Ensure that the path to the bathroom is free from equipment. d. Encourage the client to season food to stimulate nutritional intake.

ANS: C Dementia and confusion are not common phenomena in older adults. However, physical impairment related to illness can be expected. Providing opportunities for hazard-free ambulation will maintain strength and mobility (and ensure safety). Providing a call button, providing the date, and seasoning food do not address the clients impaired sensory perception.

A nurse asks a client to take deep breaths during an electroencephalography. The client asks, Why are you asking me to do this? How should the nurse respond? a. Hyperventilation causes vascular dilation of cerebral arteries, which decreases electoral activity in the brain. b. Deep breathing helps you to relax and allows the electroencephalograph to obtain a better waveform. c. Hyperventilation causes cerebral vasoconstriction and increases the likelihood of seizure activity. d. Deep breathing will help you to blow off carbon dioxide and decreases intracranial pressures.

ANS: C Hyperventilation produces cerebral vasoconstriction and alkalosis, which increases the likelihood of seizure activity. The client is asked to breathe deeply 20 to 30 times for 3 minutes. The other responses are not accurate.

A nurse is caring for a client with a history of renal insufficiency who is scheduled for a computed tomography scan of the head with contrast medium. Which priority intervention should the nurse implement? a. Educate the client about strict bedrest after the procedure. b. Place an indwelling urinary catheter to closely monitor output. c. Obtain a prescription for intravenous fluids. d. Contact the provider to cancel the procedure.

ANS: C If a contrast medium is used, intravenous fluid may be given to promote excretion of the contrast medium. Contrast medium also may act as a diuretic, resulting in the need for fluid replacement. The client will not require bedrest. Although urinary output should be monitored closely, there is no need for an indwelling urinary catheter. There is no need to cancel the procedure as long as actions are taken to protect the kidneys.

After teaching a client who is scheduled for magnetic resonance imaging (MRI), the nurse assesses the clients understanding. Which client statement indicates a correct understanding of the teaching? a. I must increase my fluids because of the dye used for the MRI. b. My urine will be radioactive so I should not share a bathroom. c. I can return to my usual activities immediately after the MRI. d. My gag reflex will be tested before I can eat or drink anything.

ANS: C No postprocedure restrictions are imposed after MRI. The client can return to normal activities after the test is complete. There are no dyes or radioactive materials used for the MRI; therefore, increased fluids are not needed and the clients urine would not be radioactive. The procedure does not impact the clients gag reflex.

A nurse teaches an 80-year-old client with diminished touch sensation. Which statement should the nurse include in this clients teaching? a. Place soft rugs in your bathroom to decrease pain in your feet. b. Bathe in warm water to increase your circulation. c. Look at the placement of your feet when walking. d. Walk barefoot to decrease pressure ulcers from your shoes.

ANS: C Older clients with decreased sensation are at risk of injury from the inability to sense changes in terrain when walking. To compensate for this loss, the client is instructed to look at the placement of her or his feet when walking. Throw rugs can slip and increase fall risk. Bath water that is too warm places the client at risk for thermal injury. The client should wear sturdy shoes for ambulation.

A nurse plans care for a client who has chronic obstructive pulmonary disease and thick, tenacious secretions. Which interventions should the nurse include in this client's plan of care? (Select all that apply.) a. Ask the client to drink 2 liters of fluids daily. b. Add humidity to the prescribed oxygen. c. Suction the client every 2 to 3 hours. d. Use a vibrating positive expiratory pressure device. e. Encourage diaphragmatic breathing.

ANS: A, B, D Interventions to decrease thick tenacious secretions include maintaining adequate hydration and providing humidified oxygen. These actions will help to thin secretions, making them easier to remove by coughing. The use of a vibrating positive expiratory pressure device can also help clients remove thick secretions. Although suctioning may assist with the removal of secretions, frequent suctioning can cause airway trauma and does not support the client's ability to successfully remove secretions through normal coughing. Diaphragmatic breathing is not used to improve the removal of thick secretions.

5. A nurse teaches a client with polycystic kidney disease (PKD). Which statements should the nurse include in this client's discharge teaching? (Select all that apply.) a. "Take your blood pressure every morning." b. "Weigh yourself at the same time each day." c. "Adjust your diet to prevent diarrhea." d. "Contact your provider if you have visual disturbances." e. "Assess your urine for renal stones."

ANS: A, B, D A client who has PKD should measure and record his or her blood pressure and weight daily, limit salt intake, and adjust dietary selections to prevent constipation. The client should notify the provider if urine smells foul or has blood in it, as these are signs of a urinary tract infection or glomerular injury. The client should also notify the provider if visual disturbances are experienced, as this is a sign of a possible berry aneurysm, which is a complication of PKD. Diarrhea and renal stones are not manifestations or complications of PKD; therefore, teaching related to these concepts would be inappropriate.

1. A nurse assesses a client who has had two episodes of bacterial cystitis in the last 6 months. Which questions should the nurse ask? (Select all that apply.) a. How much water do you drink every day? b. Do you take estrogen replacement therapy? c. Does anyone in your family have a history of cystitis? d. Are you on steroids or other immune-suppressing drugs? e. Do you drink grapefruit juice or orange juice daily?

ANS: A, B, D Fluid intake, estrogen levels, and immune suppression all can increase the chance of recurrent cystitis. Family history is usually insignificant, and cranberry juice, not grapefruit or orange juice, has been found to increase the acidic pH and reduce the risk for bacterial cystitis. DIF: Applying/Application REF: 1344 KEY: Cystitis| assessment/diagnostic examination MSC: Integrated Process: Nursing Process: Assessment NOT: Client Needs Category: Physiological Integrity: Physiological Adaptation

A nurse plans care for a client who has chronic obstructive pulmonary disease and thick, tenacious secretions. Which interventions should the nurse include in this client's plan of care? (Select all that apply.) a. Ask the client to drink 2 liters of fluids daily. b. Add humidity to the prescribed oxygen. c. Suction the client every 2 to 3 hours. d. Use a vibrating positive expiratory pressure device. e. Encourage diaphragmatic breathing.

ANS: A, B, D Interventions to decrease thick tenacious secretions include maintaining adequate hydration and providing humidified oxygen. These actions will help to thin secretions, making them easier to remove by coughing. The use of a vibrating positive expiratory pressure device can also help clients remove thick secretions. Although suctioning may assist with the removal of secretions, frequent suctioning can cause airway trauma and does not support the client's ability to successfully remove secretions through normal coughing. Diaphragmatic breathing is not used to improve the removal of thick secretions.

2. A nurse assesses a client with nephrotic syndrome. For which clinical manifestations should the nurse assess? (Select all that apply.) a. Proteinuria b. Hypoalbuminemia c. Dehydration d. Lipiduria e. Dysuria f. Costovertebral angle (CVA) tenderness

ANS: A, B, D Nephrotic syndrome is caused by glomerular damage and is characterized by proteinuria (protein level higher than 3.5 g/24 hr), hypoalbuminemia, edema, and lipiduria. Fluid overload leading to edema and hypertension is common with nephrotic syndrome; dehydration does not occur. Dysuria is present with cystitis. CVA tenderness is present with inflammatory changes in the kidney.

A nurse plans care for a client who is recovering from open reduction and internal fixation (ORIF) surgery for a right hip fracture. Which interventions should the nurse include in this client's plan of care? (Select all that apply.) a. Elevate heels off the bed with a pillow. b. Ambulate the client on the first postoperative day. c. Push the client's patient-controlled analgesia button. d. Re-position the client every 2 hours. e. Use pillows to encourage subluxation of the hip.

ANS: A, B, D Postoperative care for a client who has ORIF of the hip includes elevating the client's heels off the bed and re-positioning every 2 hours to prevent pressure and skin breakdown. It also includes ambulating the client on the first postoperative day, and using pillows or an abduction pillow to prevent subluxation of the hip. The nurse should teach the client to use the patient-controlled analgesia pump, but the nurse should never push the button for the client.

5. A nurse cares for clients with urinary incontinence. Which types of incontinence are correctly paired with their clinical manifestation? (Select all that apply.) a. Stress incontinence Urine loss with physical exertion b. Urge incontinence Large amount of urine with each occurrence c. Functional incontinence Urine loss results from abnormal detrusor contractions d. Overflow incontinence Constant dribbling of urine e. Reflex incontinence Leakage of urine without lower urinary tract disorder

ANS: A, B, D Stress incontinence is a loss of urine with physical exertion, coughing, sneezing, or exercising. Urge incontinence presents with an abrupt and strong urge to void and usually has a large amount of urine released with each occurrence. Overflow incontinence occurs with bladder distention and results in a constant dribbling of urine. Functional incontinence is the leakage of urine caused by factors other than a disorder of the lower urinary tract. Reflex incontinence results from abnormal detrusor contractions from a neurologic abnormality. DIF: Remembering/Knowledge REF: 1351 KEY: Urinary incontinence MSC: Integrated Process: Nursing Process: Analysis NOT: Client Needs Category: Physiological Integrity: Physiological Adaptation

A client is admitted with a bone tumor. The nurse finds the client weak and lethargic with decreased deep tendon reflexes. What actions by the nurse are best? (Select all that apply.) a. Assess the daily serum calcium level. b. Consult the provider about a loop diuretic. c. Institute seizure precautions for the client. d. Instruct the client to call for help out of bed. e. Place the client on a 1500-mL fluid restriction.

ANS: A, B, D The client is exhibiting manifestations of possible hypercalcemia. This disorder is treated with increased fluids and loop diuretics. The nurse should assess the calcium level, consult with the provider, and instruct the client to call for help getting out of bed due to possible fractures and weakness. The client does not need seizure precautions or fluid restrictions.

A nurse reviews a client's laboratory results. Which results from the client's urinalysis should the nurse identify as normal? (Select all that apply.) a. pH: 6 b. Specific gravity: 1.015 c. Protein: 1.2 mg/dL d. Glucose: negative e. Nitrate: small f. Leukocyte esterase: positive

ANS: A, B, D The pH, specific gravity, and glucose are all within normal ranges. The other values are abnormal.

A nurse delegates care for an older adult client to the unlicensed assistive personnel (UAP). Which statements should the nurse include when delegating this clients care? (Select all that apply.) a. Plan to bathe the client in the evening when the client is most alert. b. Encourage the client to use a cane when ambulating. c. Assess the client for symptoms related to pain and discomfort. d. Remind the client to look at foot placement when walking. e. Schedule additional time for teaching about prescribed therapies.

ANS: A, B, D The nurse should tell the UAP to schedule activities when the client is normally awake, encourage the client to use a cane when ambulating, and remind the client to look where feet are placed when walking. The nurse should assess the client for symptoms of pain and should provide sufficient time for older adults to process information, including new teaching. These are not items the nurse can delegate.

A nurse is assessing a community group for dietary factors that contribute to osteoporosis. In addition to inquiring about calcium, the nurse also assesses for which other dietary components? (Select all that apply.) a. Alcohol b. Caffeine c. Fat d. Carbonated beverages e. Vitamin D

ANS: A, B, D, E Dietary components that affect the development of osteoporosis include alcohol, caffeine, high phosphorus intake, carbonated beverages, and vitamin D. Tobacco is also a contributing lifestyle factor. Fat intake does not contribute to osteoporosis.

When assessing gait, what features does the nurse inspect? (Select all that apply.) a. Balance b. Ease of stride c. Goniometer readings d. Length of stride e. Steadiness

ANS: A, B, D, E To assess gait, look at balance, ease and length of stride, and steadiness. Goniometer readings assess flexion and extension or joint range of motion.

6. A nurse teaches a client with a history of calcium phosphate urinary stones. Which statements should the nurse include in this clients dietary teaching? (Select all that apply.) a. Limit your intake of food high in animal protein. b. Read food labels to help minimize your sodium intake. c. Avoid spinach, black tea, and rhubarb. d. Drink white wine or beer instead of red wine. e. Reduce your intake of milk and other dairy products.

ANS: A, B, E Clients with calcium phosphate urinary stones should be taught to limit the intake of foods high in animal protein, sodium, and calcium. Clients with calcium oxalate stones should avoid spinach, black tea, and rhubarb. Clients with uric acid stones should avoid red wine. DIF: Applying/Application REF: 1365 KEY: Urolithiasis| nutritional requirements MSC: Integrated Process: Teaching/Learning NOT: Client Needs Category: Physiological Integrity: Reduction of Risk Potential

A nurse teaches a client with a fractured tibia about external fixation. Which advantages of external fixation for the immobilization of fractures should the nurse share with the client? (Select all that apply.) a. It leads to minimal blood loss. b. It allows for early ambulation. c. It decreases the risk of infection. d. It increases blood supply to tissues. e. It promotes healing.

ANS: A, B, E External fixation is a system in which pins or wires are inserted through the skin and bone and then connected to a ridged external frame. With external fixation, blood loss is less than with internal fixation, but the risk for infection is much higher. The device allows early ambulation and exercise, maintains alignment, stabilizes the fracture site, and promotes healing. The device does not increase blood supply to the tissues. The nurse should assess for distal circulation, movement, and sensation, which can be disturbed by fracture injuries and treatments.

A nurse teaches a client who is at risk for carpal tunnel syndrome. Which health promotion activities should the nurse include in this client's teaching? (Select all that apply.) a. Frequently assess the ergonomics of the equipment being used. b. Take breaks to stretch fingers and wrists during working hours. c. Do not participate in activities that require repetitive actions. d. Take ibuprofen (Motrin) to decrease pain and swelling in wrists. e. Adjust chair height to allow for good posture.

ANS: A, B, E Health promotion activities to prevent carpal tunnel syndrome include assessing the ergonomics of the equipment being used, taking breaks to stretch fingers and wrists during working hours, and adjusting chair height to allow for good posture. The client should be allowed to participate in activities that require repetitive actions as long as precautions are taken to promote health. Pain medications are not part of health promotion activities.

A nurse prepares a client for a percutaneous kidney biopsy. Which actions should the nurse take prior to this procedure? (Select all that apply.) a. Keep the client NPO for 4 to 6 hours. b. Obtain coagulation study results. c. Maintain strict bedrest in a supine position. d. Assess for blood in the client's urine. e. Administer antihypertensive medications.

ANS: A, B, E Prior to a percutaneous kidney biopsy, the client should be NPO for 4 to 6 hours. Coagulation studies should be completed to prevent bleeding after the biopsy. Blood pressure medications should be administered to prevent hypertension before and after the procedure. There is no need to keep the client on bedrest or assess for blood in the client's urine prior to the procedure, these interventions should be implemented after a percutaneous kidney biopsy.

8. A nurse teaches a female client who has stress incontinence. Which statements should the nurse include about pelvic muscle exercises? (Select all that apply.) a. When you start and stop your urine stream, you are using your pelvic muscles. b. Tighten your pelvic muscles for a slow count of 10 and then relax for a slow count of 10. c. Pelvic muscle exercises should only be performed sitting upright with your feet on the floor. d. After you have been doing these exercises for a couple days, your control of urine will improve. e. Like any other muscle in your body, you can make your pelvic muscles stronger by contracting them.

ANS: A, B, E The client should be taught that the muscles used to start and stop urination are pelvic muscles, and that pelvic muscles can be strengthened by contracting and relaxing them. The client should tighten pelvic muscles for a slow count of 10 and then relax the muscles for a slow count of 10, and perform this exercise 15 times while in lying-down, sitting-up, and standing positions. The client should begin to notice improvement in control of urine after several weeks of exercising the pelvic muscles. DIF: Understanding/Comprehension REF: 1354 KEY: Urinary incontinence| patient education MSC: Integrated Process: Teaching/Learning NOT: Client Needs Category: Health Promotion and Maintenance

A nurse assesses a client with a brain tumor. Which newly identified assessment findings should alert the nurse to urgently communicate with the health care provider? (Select all that apply.) a. Glasgow Coma Scale score of 8 b. Decerebrate posturing c. Reactive pupils d. Uninhibited speech e. Diminished cognition

ANS: A, B, E The nurse should urgently communicate changes in a clients neurologic status, including a decrease in the Glasgow Coma Scale score, abnormal flexion or extension, changes in cognition or speech, and pinpointed, dilated, and nonreactive pupils.

A nurse plans care for an older adult client. Which interventions should the nurse include in this client's plan of care to promote kidney health? (Select all that apply.) a. Ensure adequate fluid intake. b. Leave the bathroom light on at night. c. Encourage use of the toilet every 6 hours. d. Delegate bladder training instructions to the unlicensed assistive personnel (UAP). e. Provide thorough perineal care after each voiding. f. Assess for urinary retention and urinary tract infection.

ANS: A, B, E, F The nurse should ensure that the client receives adequate fluid intake and has adequate lighting to ambulate safely to the bathroom at night, encourage the client to use the toilet every 2 hours, provide thorough perineal care after each voiding, and assess for urinary retention and urinary tract infections. The nurse should not delegate any teaching to the UAP, including bladder training instructions. The UAP may participate in bladder training activities, including encouraging and assisting the client to the bathroom at specific times.

An older client asks the nurse why people my age have weaker immune systems than younger people. What responses by the nurse are best? (Select all that apply.) a. Bone marrow produces fewer blood cells. b. You may have decreased levels of circulating platelets. c. You have lower levels of plasma proteins in the blood. d. Lymphocytes become more reactive to antigens. e. Spleen function declines after age 60.

ANS: A, C The aging adult has bone marrow that produces fewer cells and decreased blood volume with fewer plasma proteins. Platelet numbers remain unchanged, lymphocytes become less reactive, and spleen function stays the same.

A nurse assesses a client with a cast for potential compartment syndrome. Which clinical manifestations are correctly paired with the physiologic changes of compartment syndrome? (Select all that apply.) a. Edema - Increased capillary permeability b. Pallor - Increased blood blow to the area c. Unequal pulses - Increased production of lactic acid d. Cyanosis - Anaerobic metabolism e. Tingling - A release of histamine

ANS: A, C, D Clinical manifestations of compartment syndrome are caused by several physiologic changes. Edema is caused by increased capillary permeability, release of histamine, decreased tissue perfusion, and vasodilation. Unequal pulses are caused by an increased production of lactic acid. Cyanosis is caused by anaerobic metabolism. Pallor is caused by decreased oxygen to tissues, and tingling is caused by increased tissue pressure.

A nurse plans care for a client who is at high risk for a pulmonary infection. Which interventions should the nurse include in this client's plan of care? (Select all that apply.) a. Encourage deep breathing and coughing. b. Implement an air mattress overlay. c. Ambulate the client three times each day. d. Provide a diet high in protein and vitamins. e. Administer acetaminophen (Tylenol) twice daily.

ANS: A, C, D Regular pulmonary hygiene and activities to maintain health and fitness help to maximize functioning of the respiratory system and prevent infection. A client at high risk for a pulmonary infection may need a specialty bed to help with postural drainage or percussion; this would not include an air mattress overlay, which is used to prevent pressure ulcers. Tylenol would not decrease the risk of a pulmonary infection.

An emergency department nurse assesses a client who was struck in the temporal lobe with a baseball. For which clinical manifestations that are related to a temporal lobe injury should the nurse assess? (Select all that apply.) a. Memory loss b. Personality changes c. Difficulty with sound interpretation d. Speech difficulties e. Impaired taste

ANS: A, C, D Wernickes area (language area) is located in the temporal lobe and enables the processing of words into coherent thought as well as the understanding of written or spoken words. The temporal lobe also is responsible for the auditory centers interpretation of sound and complicated memory patterns. Personality changes are related to frontal lobe injury. Impaired taste is associated with injury to the parietal lobe.

A nurse is providing pneumonia vaccinations in a community setting. Due to limited finances, the event organizers must limit giving the vaccination to priority groups. What clients would be considered a priority when administering the pneumonia vaccination? (Select all that apply.) a. 22-year-old client with asthma b. Client who had a cholecystectomy last year c. Client with well-controlled diabetes d. Healthy 72-year-old client e. Client who is taking medication for hypertension

ANS: A, C, D, E Clients over 65 years of age and any client (no matter what age) with a chronic health condition would be considered a priority for a pneumonia vaccination. Having a cholecystectomy a year ago does not qualify as a chronic health condition.

A nurse cares for a client who is prescribed an intravenous prostacyclin agent. Which actions should the nurse take to ensure the client's safety while on this medication? (Select all that apply.) a. Keep an intravenous line dedicated strictly to the infusion. b. Teach the client that this medication increases pulmonary pressures. c. Ensure that there is always a backup drug cassette available. d. Start a large-bore peripheral intravenous line. e. Use strict aseptic technique when using the drug delivery system.

ANS: A, C, E Intravenous prostacyclin agents should be administered in a central venous catheter with a dedicated intravenous line for this medication. Death has been reported when the drug delivery system is interrupted; therefore, a backup drug cassette should also be available. The nurse should use strict aseptic technique when using the drug delivery system. The nurse should teach the client that this medication decreases pulmonary pressures and increases lung blood flow.

A student nurse learns about changes that occur to the musculoskeletal system due to aging. Which changes does this include? (Select all that apply.) a. Bone changes lead to potential safety risks. b. Increased bone density leads to stiffness. c. Osteoarthritis occurs due to cartilage degeneration. d. Osteoporosis is a universal occurrence. e. Some muscle tissue atrophy occurs with aging.

ANS: A, C, E Many age-related changes occur in the musculoskeletal system, including decreased bone density, degeneration of cartilage, and some degree of muscle tissue atrophy. Osteoporosis, while common, is not universal. Bone density decreases with age, not increases.

A client with chronic osteomyelitis is being discharged from the hospital. What information is important for the nurse to teach this client and family? (Select all that apply.) a. Adherence to the antibiotic regimen b. Correct intramuscular injection technique c. Eating high-protein and high-carbohydrate foods d. Keeping daily follow-up appointments e. Proper use of the intravenous equipment

ANS: A, C, E The client going home with chronic osteomyelitis will need long-term antibiotic therapy—first intravenous, then oral. The client needs education on how to properly administer IV antibiotics, care for the IV line, adhere to the regimen, and eat a healthy diet to encourage wound healing. The antibiotics are not given by IM injection. The client does not need daily follow-up.

2. A nurse teaches a client about self-catheterization in the home setting. Which statements should the nurse include in this clients teaching? (Select all that apply.) a. Wash your hands before and after self-catheterization. b. Use a large-lumen catheter for each catheterization. c. Use lubricant on the tip of the catheter before insertion. d. Self-catheterize at least twice a day or every 12 hours. e. Use sterile gloves and sterile technique for the procedure. f. Maintain a specific schedule for catheterization.

ANS: A, C, F The key points in self-catheterization include washing hands, using lubricants, and maintaining a regular schedule to avoid distention and retention of urine that leads to bacterial growth. A smaller rather than a larger lumen catheter is preferred. The client needs to catheterize more often than every 12 hours. Self-catheterization in the home is a clean procedure. DIF: Applying/Application REF: 1359 KEY: Urinary incontinence| patient education MSC: Integrated Process: Teaching/Learning NOT: Client Needs Category: Safe and Effective Care Environment: Safety and Infection Control

A nurse assesses a client recovering from a cystoscopy. Which assessment findings should alert the nurse to urgently contact the health care provider? (Select all that apply.) a. Decrease in urine output b. Tolerating oral fluids c. Prescription for metformin d. Blood clots present in the urine e. Burning sensation when urinating

ANS: A, D The nurse should monitor urine output and contact the provider if urine output decreases or becomes absent. The nurse should also assess for blood in the client's urine. The urine may be pink-tinged, but gross bleeding or blood clots should not be present. If bleeding is present, the nurse should urgently contact the provider. Tolerating oral fluids is a positive outcome and does not need intervention. Metformin would be a concern if the client received dye, no dye is used in a cystoscopy procedure. The client may experience a burning sensation when urinating after this procedure. this would not require a call to the provider.

A nurse assesses a client who is prescribed varenicline (Chantix) for smoking cessation. Which manifestations should the nurse identify as adverse effects of this medication? (Select all that apply.) a. Visual hallucinations b. Tachycardia c. Decreased cravings d. Impaired judgment e. Increased thirst

ANS: A, D Varenicline (Chantix) has a black box warning stating that the drug can cause manic behavior and hallucinations. The nurse should assess for changes in behavior and thought processes, including impaired judgment and visual hallucinations. Tachycardia and increased thirst are not adverse effects of this medication. Decreased cravings is a therapeutic response to this medication.

A nurse teaches a client about prosthesis care after amputation. Which statements should the nurse include in this client's teaching? (Select all that apply.) a. "The device has been custom made specifically for you." b. "Your prosthetic is good for work but not for exercising." c. "A prosthetist will clean your inserts for you each month." d. "Make sure that you wear the correct liners with your prosthetic." e. "I have scheduled a follow-up appointment for you."

ANS: A, D, E A client with a new prosthetic should be taught that the prosthetic device is custom made for the client, taking into account the client's level of amputation, lifestyle (including exercise preferences), and occupation. In collaboration with a prosthetist, the client should be taught proper techniques for cleansing the sockets and inserts, wearing the correct liners, and assessing shoe wear. Follow-up care and appointments are important for ongoing assessment.

While obtaining a client's health history, the client states, "I am allergic to avocados." Which responses by the nurse are best? (Select all that apply.) a. "What response do you have when you eat avocados?" b. "I will remove any avocados that are on your lunch tray." c. "When was the last time you ate foods containing avocados?" d. "I will document this in your record so all of your providers will know." e. "Have you ever been treated for this allergic reaction?"

ANS: A, D, E Nurses should assess clients who have allergies for the specific cause, treatment, and response to treatment. The nurse should also document the allergies in a prominent place in the client's medical record. The nurse should collaborate with food services to ensure no avocados are placed on the client's meal trays. Asking about the last time the client ate avocados does not provide any pertinent information for the client's plan of care.

A nurse teaches a client who is interested in smoking cessation. Which statements should the nurse include in this client's teaching? (Select all that apply.) a. "Find an activity that you enjoy and will keep your hands busy." b. "Keep snacks like potato chips on hand to nibble on." c. "Identify a punishment for yourself in case you backslide." d. "Drink at least eight glasses of water each day." e. "Make a list of reasons you want to stop smoking."

ANS: A, D, E The nurse should teach a client who is interested in smoking cessation to find an activity that keeps the hands busy, to keep healthy snacks on hand to nibble on, to drink at least 8 glasses of water each day, and to make a list of reasons for quitting smoking. The nurse should also encourage the client not to be upset if he or she backslides and has a cigarette.

A nurse collaborates with a respiratory therapist to complete pulmonary function tests (PFTs) for a client. Which statements should the nurse include in communications with the respiratory therapist prior to the tests? (Select all that apply.) a. "I held the client's morning bronchodilator medication." b. "The client is ready to go down to radiology for this examination." c. "Physical therapy states the client can run on a treadmill." d. "I advised the client not to smoke for 6 hours prior to the test." e. "The client is alert and can follow your commands."

ANS: A, D, E To ensure the PFTs are accurate, the therapist needs to know that no bronchodilators have been administered in the past 4 to 6 hours, the client did not smoke within 6 to 8 hours prior to the test, and the client can follow basic commands, including different breathing maneuvers. The respiratory therapist can perform PFTs at the bedside. A treadmill is not used for this test.

A nurse assesses clients at a health fair. Which clients should the nurse counsel to be tested for diabetes? (Select all that apply.) a. 56-year-old African-American male b. Female with a 30-pound weight gain during pregnancy c. Male with a history of pancreatic trauma d. 48-year-old woman with a sedentary lifestyle e. Male with a body mass index greater than 25 kg/m2 f. 28-year-old female who gave birth to a baby weighing 9.2 pounds

ANS: A, D, E, F Risk factors for type 2 diabetes include certain ethnic/racial groups (African Americans, American Indians, Hispanics), obesity and physical inactivity, and giving birth to large babies. Pancreatic trauma and a 30-pound gestational weight gain are not risk factors.

A nurse cares for a client with arthritis who reports frequent asthma attacks. Which action should the nurse take first? a. Review the client's pulmonary function test results. b. Ask about medications the client is currently taking. c. Assess how frequently the client uses a bronchodilator. d. Consult the provider and request arterial blood gases.

ANS: B Aspirin and other nonsteroidal anti-inflammatory drugs (NSAIDs) can trigger asthma in some people. This results from increased production of leukotriene when aspirin or NSAIDs suppress other inflammatory pathways and is a high priority given the client's history. Reviewing pulmonary function test results will not address the immediate problem of frequent asthma attacks. This is a good intervention for reviewing response to bronchodilators. Questioning the client about the use of bronchodilators will address interventions for the attacks but not their cause. Reviewing arterial blood gas results would not be of use in a client between attacks because many clients are asymptomatic when not having attacks

A nurse administers medications to a client who has asthma. Which medication classification is paired correctly with its physiologic response to the medication? a. Bronchodilator - Stabilizes the membranes of mast cells and prevents the release of inflammatory mediators b. Cholinergic antagonist - Causes bronchodilation by inhibiting the parasympathetic nervous system c. Corticosteroid - Relaxes bronchiolar smooth muscles by binding to and activating pulmonary beta2 receptors d. Cromone - Disrupts the production of pathways of inflammatory mediators

ANS: B Cholinergic antagonist drugs cause bronchodilation by inhibiting the parasympathetic nervous system. This allows the sympathetic nervous system to dominate and release norepinephrine that actives beta2 receptors. Bronchodilators relax bronchiolar smooth muscles by binding to and activating pulmonary beta2 receptors. Corticosteroids disrupt the production of pathways of inflammatory mediators. Cromones stabilize the membranes of mast cells and prevent the release of inflammatory mediators.

While assessing a client who is 12 hours postoperative after a thoracotomy for lung cancer, a nurse notices that the lower chest tube is dislodged. Which action should the nurse take first? a. Assess for drainage from the site. b. Cover the insertion site with sterile gauze. c. Contact the provider and obtain a suture kit. d. Reinsert the tube using sterile technique.

ANS: B Immediately covering the insertion site helps prevent air from entering the pleural space and causing a pneumothorax. The area will not reseal quickly enough to prevent air from entering the chest. The nurse should not leave the client to obtain a suture kit. An occlusive dressing may cause a tension pneumothorax. The site should only be assessed after the insertion site is covered. The provider should be called to reinsert the chest tube or prescribe other treatment options.

A pulmonary nurse cares for clients who have chronic obstructive pulmonary disease (COPD). Which client should the nurse assess first? a. A 46-year-old with a 30-pack-year history of smoking b. A 52-year-old in a tripod position using accessory muscles to breathe c. A 68-year-old who has dependent edema and clubbed fingers d. A 74-year-old with a chronic cough and thick, tenacious secretions

ANS: B The client who is in a tripod position and using accessory muscles is working to breathe. This client must be assessed first to establish how well the client is breathing and provide interventions to minimize respiratory failure. The other clients are not in acute distress.

After teaching a client how to perform diaphragmatic breathing, the nurse assesses the client's understanding. Which action demonstrates that the client correctly understands the teaching? a. The client lays on his or her side with his or her knees bent. b. The client places his or her hands on his or her abdomen. c. The client lays in a prone position with his or her legs straight. d. The client places his or her hands above his or her head.

ANS: B To perform diaphragmatic breathing correctly, the client should place his or her hands on his or her abdomen to create resistance. This type of breathing cannot be performed effectively while lying on the side or with hands over the head. This type of breathing would not be as effective lying prone.

A client has a bone density score of -2.8. What action by the nurse is best? a. Asking the client to complete a food diary b. Planning to teach about bisphosphonates c. Scheduling another scan in 2 years d. Scheduling another scan in 6 months

ANS: B A T-score from a bone density scan at or lower than -2.5 indicates osteoporosis. The nurse should plan to teach about medications used to treat this disease. One class of such medications is bisphosphonates. A food diary is helpful to determine if the client gets adequate calcium and vitamin D, but at this point, dietary changes will not prevent the disease. Simply scheduling another scan will not help treat the disease either.

A nurse prepares to administer prescribed regular and NPH insulin. Place the nurse's actions in the correct order to administer these medications. 1. Inspect bottles for expiration dates. 2. Gently roll the bottle of NPH between the hands. 3. Wash your hands. 4. Inject air into the regular insulin. 5. Withdraw the NPH insulin. 6. Withdraw the regular insulin. 7. Inject air into the NPH bottle. 8. Clean rubber stoppers with an alcohol swab. a. 1, 3, 8, 2, 4, 6, 7, 5 b. 3, 1, 2, 8, 7, 4, 6, 5 c. 8, 1, 3, 2, 4, 6, 7, 5 d. 2, 3, 1, 8, 7, 5, 4, 6

ANS: B After washing hands, it is important to inspect the bottles and then to roll the NPH to mix the insulin. Rubber stoppers should be cleaned with alcohol after rolling the NPH and before sticking a needle into either bottle. It is important to inject air into the NPH bottle before placing the needle in a regular insulin bottle to avoid mixing of regular and NPH insulin. The shorter-acting insulin is always drawn up first.

12. A nurse provides health screening for a community health center with a large population of African-American clients. Which priority assessment should the nurse include when working with this population? a. Measure height and weight. b. Assess blood pressure. c. Observe for any signs of abuse. d. Ask about medications.

ANS: B All interventions are important for the visiting nurse to accomplish. However, African Americans have a high rate of hypertension leading to end-stage renal disease. Each encounter that the nurse has with an African-American client provides a chance to detect hypertension and treat it. If the client is already on antihypertensive medication, assessing blood pressure monitors therapy.

A nurse cares for a client who is diagnosed with acute rejection 2 months after receiving a simultaneous pancreas-kidney transplant. The client states, "I was doing so well with my new organs, and the thought of having to go back to living on hemodialysis and taking insulin is so depressing." How should the nurse respond? a. "Following the drug regimen more closely would have prevented this." b. "One acute rejection episode does not mean that you will lose the new organs." c. "Dialysis is a viable treatment option for you and may save your life." d. "Since you are on the national registry, you can receive a second transplantation."

ANS: B An episode of acute rejection does not automatically mean that the client will lose the transplant. Pharmacologic manipulation of host immune responses at this time can limit damage to the organ and allow the graft to be maintained. The other statements either belittle the client or downplay his or her concerns. The client may not be a candidate for additional organ transplantation.

2. A nurse reviews the laboratory findings of a client with a urinary tract infection. The laboratory report notes a shift to the left in a clients white blood cell count. Which action should the nurse take? a. Request that the laboratory perform a differential analysis on the white blood cells. b. Notify the provider and start an intravenous line for parenteral antibiotics. c. Collaborate with the unlicensed assistive personnel (UAP) to strain the clients urine for renal calculi. d. Assess the client for a potential allergic reaction and anaphylactic shock.

ANS: B An increase in band cells creates a shift to the left. A left shift most commonly occurs with urosepsis and is seen rarely with uncomplicated urinary tract infections. The nurse will be administering antibiotics, most likely via IV, so he or she should notify the provider and prepare to give the antibiotics. The shift to the left is part of a differential white blood cell count. The nurse would not need to strain urine for stones. Allergic reactions are associated with elevated eosinophil cells, not band cells. DIF: Applying/Application REF: 1347 KEY: Cystitis| assessment/diagnostic examination MSC: Integrated Process: Nursing Process: Implementation NOT: Client Needs Category: Physiological Integrity: Physiological Adaptation

Which teaching point is most important for the client with bacterial pharyngitis? a. Gargle with warm salt water. b. Take all antibiotics as directed. c. Use a humidifier in the bedroom. d. Wash hands frequently.

ANS: B Any client on antibiotics must be instructed to complete the entire course of antibiotics. Not completing them can lead to complications or drug-resistant strains of bacteria. The other instructions are appropriate, just not the most important

A nurse cares for a client with arthritis who reports frequent asthma attacks. Which action should the nurse take first? a. Review the client's pulmonary function test results. b. Ask about medications the client is currently taking. c. Assess how frequently the client uses a bronchodilator. d. Consult the provider and request arterial blood gases.

ANS: B Aspirin and other nonsteroidal anti-inflammatory drugs (NSAIDs) can trigger asthma in some people. This results from increased production of leukotriene when aspirin or NSAIDs suppress other inflammatory pathways and is a high priority given the client's history. Reviewing pulmonary function test results will not address the immediate problem of frequent asthma attacks. This is a good intervention for reviewing response to bronchodilators. Questioning the client about the use of bronchodilators will address interventions for the attacks but not their cause. Reviewing arterial blood gas results would not be of use in a client between attacks because many clients are asymptomatic when not having attacks.

A nurse is caring for an older adult client who has a pulmonary infection. Which action should the nurse take first? a. Encourage the client to increase fluid intake. b. Assess the client's level of consciousness. c. Raise the head of the bed to at least 45 degrees. d. Provide the client with humidified oxygen.

ANS: B Assessing the client's level of consciousness will be most important because it will show how the client is responding to the presence of the infection. Although it will be important for the nurse to encourage the client to turn, cough, and frequently breathe deeply; raise the head of the bed; increase oral fluid intake; and humidify the oxygen administered, none of these actions will be as important as assessing the level of consciousness. Also, the client who has a pulmonary infection may not be able to cough effectively if an area of abscess is present.

A client is undergoing computed tomography (CT) of a joint. What action by the nurse is most important before the test? a. Administer sedation as prescribed. b. Assess for seafood or iodine allergy. c. Ensure that the client has no metal on the body. d. Provide preprocedure pain medication.

ANS: B Because CT uses iodine-based contrast material, the nurse assesses the client for allergies to iodine or seafood (which often contains iodine). The other actions are not needed.

A nurse is teaching a client with diabetes mellitus who asks, "Why is it necessary to maintain my blood glucose levels no lower than about 60 mg/dL?" How should the nurse respond? a. "Glucose is the only fuel used by the body to produce the energy that it needs." b. "Your brain needs a constant supply of glucose because it cannot store it." c. "Without a minimum level of glucose, your body does not make red blood cells." d. "Glucose in the blood prevents the formation of lactic acid and prevents acidosis."

ANS: B Because the brain cannot synthesize or store significant amounts of glucose, a continuous supply from the body's circulation is needed to meet the fuel demands of the central nervous system. The nurse would want to educate the client to prevent hypoglycemia. The body can use other sources of fuel, including fat and protein, and glucose is not involved in the production of red blood cells. Glucose in the blood will encourage glucose metabolism but is not directly responsible for lactic acid formation.

A nurse administers medications to a client who has asthma. Which medication classification is paired correctly with its physiologic response to the medication? a. Bronchodilator - Stabilizes the membranes of mast cells and prevents the release of inflammatory mediators b. Cholinergic antagonist - Causes bronchodilation by inhibiting the parasympathetic nervous system c. Corticosteroid - Relaxes bronchiolar smooth muscles by binding to and activating pulmonary beta2 receptors d. Cromone - Disrupts the production of pathways of inflammatory mediators

ANS: B Cholinergic antagonist drugs cause bronchodilation by inhibiting the parasympathetic nervous system. This allows the sympathetic nervous system to dominate and release norepinephrine that actives beta2 receptors. Bronchodilators relax bronchiolar smooth muscles by binding to and activating pulmonary beta2 receptors. Corticosteroids disrupt the production of pathways of inflammatory mediators. Cromones stabilize the membranes of mast cells and prevent the release of inflammatory mediators.

3. After teaching a client with early polycystic kidney disease (PKD) about nutritional therapy, the nurse assesses the client's understanding. Which statement made by the client indicates a correct understanding of the teaching? a. "I will take a laxative every night before going to bed." b. "I must increase my intake of dietary fiber and fluids." c. "I shall only use salt when I am cooking my own food." d. "I'll eat white bread to minimize gastrointestinal gas."

ANS: B Clients with PKD often have constipation, which can be managed with increased fiber, exercise, and drinking plenty of water. Laxatives should be used cautiously. Clients with PKD should be on a restricted salt diet, which includes not cooking with salt. White bread has a low fiber count and would not be included in a high-fiber diet.

A nurse plans care for a client who is recovering from a below-the-knee amputation of the left leg. Which intervention should the nurse include in this client's plan of care? a. Place pillows between the client's knees. b. Encourage range-of-motion exercises. c. Administer prophylactic antibiotics. d. Implement strict bedrest in a supine position.

ANS: B Clients with a below-the-knee amputation should complete range-of-motion exercises to prevent flexion contractions and prepare for a prosthesis. A pillow may be used under the limb as support. Clients recovering from this type of amputation are at low risk for infection and should not be prescribed prophylactic antibiotics. The client should be encouraged to re-position, move, and exercise frequently, and therefore should not be restricted to bedrest.

8. An emergency department nurse assesses a client with kidney trauma and notes that the client's abdomen is tender and distended and blood is visible at the urinary meatus. Which prescription should the nurse consult the provider about before implementation? a. Assessing vital signs every 15 minutes b. Inserting an indwelling urinary catheter c. Administering intravenous fluids at 125 mL/hr d. Typing and crossmatching for blood products

ANS: B Clients with blood at the urinary meatus should not have a urinary catheter inserted via the urethra before additional diagnostic studies are done. The urethra could be torn. The nurse should question the provider about the need for a catheter; if one is needed, the provider can insert a suprapubic catheter. The nurse should monitor the client's vital signs closely, send blood for type and crossmatch in case the client needs blood products, and administer intravenous fluids.

After teaching a client with type 2 diabetes mellitus, the nurse assesses the client's understanding. Which statement made by the client indicates a need for additional teaching? a. "I need to have an annual appointment even if my glucose levels are in good control." b. "Since my diabetes is controlled with diet and exercise, I must be seen only if I am sick." c. "I can still develop complications even though I do not have to take insulin at this time." d. "If I have surgery or get very ill, I may have to receive insulin injections for a short time."

ANS: B Clients with diabetes need to be seen at least annually to monitor for long-term complications, including visual changes, microalbuminuria, and lipid analysis. The client may develop complications and may need insulin in the future.

A nurse is caring for a client who received benzocaine spray prior to a recent bronchoscopy. The client presents with continuous cyanosis even with oxygen therapy. Which action should the nurse take next? a. Administer an albuterol treatment. b. Notify the Rapid Response Team. c. Assess the client's peripheral pulses. d. Obtain blood and sputum cultures.

ANS: B Cyanosis unresponsive to oxygen therapy is a manifestation of methemoglobinemia, which is an adverse effect of benzocaine spray. Death can occur if the level of methemoglobin rises and cyanosis occurs. The nurse should notify the Rapid Response Team to provide advanced nursing care. An albuterol treatment would not address the client's oxygenation problem. Assessment of pulses and cultures will not provide data necessary to treat this client.

A nurse reviews the medication list of a client with a 20-year history of diabetes mellitus. The client holds up the bottle of prescribed duloxetine (Cymbalta) and states, "My cousin has depression and is taking this drug. Do you think I'm depressed?" How should the nurse respond? a. "Many people with long-term diabetes become depressed after a while." b. "It's for peripheral neuropathy. Do you have burning pain in your feet or hands?" c. "This antidepressant also has anti-inflammatory properties for diabetic pain." d. "No. Many medications can be used for several different disorders."

ANS: B Damage along nerves causes peripheral neuropathy and leads to burning pain along the nerves. Many drugs, including duloxetine (Cymbalta), can be used to treat peripheral neuropathy. The nurse should assess the client for this condition and then should provide an explanation of why this drug is being used. This medication, although it is used for depression, is not being used for that reason in this case. Duloxetine does not have anti-inflammatory properties. Telling the client that many medications are used for different disorders does not provide the client with enough information to be useful.

A nurse cares for a client with diabetes mellitus who asks, "Why do I need to administer more than one injection of insulin each day?" How should the nurse respond? a. "You need to start with multiple injections until you become more proficient at self-injection." b. "A single dose of insulin each day would not match your blood insulin levels and your food intake patterns." c. "A regimen of a single dose of insulin injected each day would require that you eat fewer carbohydrates." d. "A single dose of insulin would be too large to be absorbed, predictably putting you at risk for insulin shock."

ANS: B Even when a single injection of insulin contains a combined dose of different-acting insulin types, the timing of the actions and the timing of food intake may not match well enough to prevent wide variations in blood glucose levels. One dose of insulin would not be appropriate even if the client decreased carbohydrate intake. Additional injections are not required to allow the client practice with injections, nor will one dose increase the client's risk of insulin shock.

22. A nurse plans care for clients with urinary incontinence. Which client is correctly paired with the appropriate intervention? a. A 29-year-old client after a difficult vaginal delivery Habit training b. A 58-year-old postmenopausal client who is not taking estrogen therapy Electrical stimulation c. A 64-year-old female with Alzheimers-type senile dementia Bladder training d. A 77-year-old female who has difficulty ambulating Exercise therapy

ANS: B Exercise therapy and electrical stimulation are used for clients with stress incontinence related to childbirth or low levels of estrogen after menopause. Exercise therapy increases pelvic wall strength; it does not improve ambulation. Physical therapy and a bedside commode would be appropriate interventions for the client who has difficulty ambulating. Habit training is the type of bladder training that will be most effective with cognitively impaired clients. Bladder training can be used only with a client who is alert, aware, and able to resist the urge to urinate. DIF: Applying/Application REF: 1350 KEY: Urinary incontinence MSC: Integrated Process: Nursing Process: Assessment NOT: Client Needs Category: Physiological Integrity: Basic Care and Comfort

7. After delegating care to an unlicensed assistive personnel (UAP) for a client who is prescribed habit training to manage incontinence, a nurse evaluates the UAPs understanding. Which action indicates the UAP needs additional teaching? a. Toileting the client after breakfast b. Changing the clients incontinence brief when wet c. Encouraging the client to drink fluids d. Recording the clients incontinence episodes

ANS: B Habit training is undermined by the use of absorbent incontinence briefs or pads. The nurse should re-educate the UAP on the technique of habit training. The UAP should continue to toilet the client after meals, encourage the client to drink fluids, and record incontinent episodes. DIF: Applying/Application REF: 1358 KEY: Urinary incontinence| delegation| supervision| unlicensed assistive personnel (UAP) MSC: Integrated Process: Teaching/Learning NOT: Client Needs Category: Safe and Effective Care Environment: Management of Care

6. After teaching a client with nephrotic syndrome and a normal glomerular filtration, the nurse assesses the client's understanding. Which statement made by the client indicates a correct understanding of the nutritional therapy for this condition? a. "I must decrease my intake of fat." b. "I will increase my intake of protein." c. "A decreased intake of carbohydrates will be required." d. "An increased intake of vitamin C is necessary."

ANS: B In nephrotic syndrome, the renal loss of protein is significant, leading to hypoalbuminemia and edema formation. If glomerular filtration is normal or near normal, increased protein loss should be matched by increased intake of protein. The client would not need to adjust fat, carbohydrates, or vitamins based on this disorder.

3. A nurse cares for a postmenopausal client who has had two episodes of bacterial urethritis in the last 6 months. The client asks, I never have urinary tract infections. Why is this happening now? How should the nurse respond? a. Your immune system becomes less effective as you age. b. Low estrogen levels can make the tissue more susceptible to infection. c. You should be more careful with your personal hygiene in this area. d. It is likely that you have an untreated sexually transmitted disease.

ANS: B Low estrogen levels decrease moisture and secretions in the perineal area and cause other tissue changes, predisposing it to the development of infection. Urethritis is most common in postmenopausal women for this reason. Although immune function does decrease with aging and sexually transmitted diseases are a known cause of urethritis, the most likely reason in this client is low estrogen levels. Personal hygiene usually does not contribute to this disease process. DIF: Applying/Application REF: 1344 Test Bank - Medical-Surgical Nursing: Concepts for Interprofessional Collaborative Care 9e 557 KEY: Cystitis| patient education MSC: Integrated Process: Teaching/Learning NOT: Client Needs Category: Health Promotion and Maintenance

A nurse teaches a client with type 2 diabetes mellitus who is prescribed glipizide (Glucotrol). Which statement should the nurse include in this client's teaching? a. "Change positions slowly when you get out of bed." b. "Avoid taking nonsteroidal anti-inflammatory drugs (NSAIDs)." c. "If you miss a dose of this drug, you can double the next dose." d. "Discontinue the medication if you develop a urinary infection."

ANS: B NSAIDs potentiate the hypoglycemic effects of sulfonylurea agents. Glipizide is a sulfonylurea. The other statements are not applicable to glipizide.

A nurse cares for a client who has diabetes mellitus. The nurse administers 6 units of regular insulin and 10 units of NPH insulin at 0700. At which time should the nurse assess the client for potential problems related to the NPH insulin? a. 0800 b. 1600 c. 2000 d. 2300

ANS: B Neutral protamine Hagedorn (NPH) is an intermediate-acting insulin with an onset of 1.5 hours, peak of 4 to 12 hours, and duration of action of 22 hours. Checking the client at 0800 would be too soon. Checking the client at 2000 and 2300 would be too late. The nurse should check the client at 1600.

A client is in the family medicine clinic reporting a dry, sore throat. The provider asks the nurse to assess for odynophagia. What assessment technique is most appropriate? a. Ask the client what foods cause trouble swallowing. b. Assess the client for pain when swallowing. c. Determine if the client can swallow saliva. d. Palpate the client's jaw while swallowing.

ANS: B Odynophagia is painful swallowing. The nurse should assess the client for this either by asking or by having the client attempt to drink water. It is not related to specific foods and is not assessed by palpating the jaw. Being unable to swallow saliva is not odynophagia, but it would be a serious situation.

1. A nurse assesses a client with polycystic kidney disease (PKD). Which assessment finding should alert the nurse to immediately contact the health care provider? a. Flank pain b. Periorbital edema c. Bloody and cloudy urine d. Enlarged abdomen

ANS: B Periorbital edema would not be a finding related to PKD and should be investigated further. Flank pain and a distended or enlarged abdomen occur in PKD because the kidneys enlarge and displace other organs. Urine can be bloody or cloudy as a result of cyst rupture or infection.

The charge nurse on a medical unit is preparing to admit several "clients" who have possible pandemic flu during a preparedness drill. What action by the nurse is best? a. Admit the "clients" on Contact Precautions. b. Cohort the "clients" in the same area of the unit. c. Do not allow pregnant caregivers to care for these "clients." d. Place the "clients" on enhanced Droplet Precautions.

ANS: B Preventing the spread of pandemic flu is equally important as caring for the clients who have it. Clients can be cohorted together in the same set of rooms on one part of the unit to use distancing to help prevent the spread of the disease. The other actions are not appropriate.

A nurse assesses a client with diabetes mellitus. Which clinical manifestation should alert the nurse to decreased kidney function in this client? a. Urine specific gravity of 1.033 b. Presence of protein in the urine c. Elevated capillary blood glucose level d. Presence of ketone bodies in the urine

ANS: B Renal dysfunction often occurs in the client with diabetes. Proteinuria is a result of renal dysfunction. Specific gravity is elevated with dehydration. Elevated capillary blood glucose levels and ketones in the urine are consistent with diabetes mellitus but are not specific to renal function.

A nurse reviews the health history of a client with an oversecretion of renin. Which disorder should the nurse correlate with this assessment finding? a. Alzheimer's disease b. Hypertension c. Diabetes mellitus d. Viral hepatitis

ANS: B Renin is secreted when special cells in the distal convoluted tubule, called the macula densa, sense changes in blood volume and pressure. When the macula densa cells sense that blood volume, blood pressure, or blood sodium levels are low, renin is secreted. Renin then converts angiotensinogen into angiotensin I. This leads to a series of reactions that cause secretion of the hormone aldosterone. This hormone increases kidney reabsorption of sodium and water, increasing blood pressure, blood volume, and blood sodium levels. Inappropriate or excessive renin secretion is a major cause of persistent hypertension. Renin has no impact on Alzheimer's disease, diabetes mellitus, or viral hepatitis.

A nurse develops a dietary plan for a client with diabetes mellitus and new-onset microalbuminuria. Which component of the client's diet should the nurse decrease? a. Carbohydrates b. Proteins c. Fats d. Total calories

ANS: B Restriction of dietary protein to 0.8 g/kg of body weight per day is recommended for clients with microalbuminuria to delay progression to renal failure. The client's diet does not need to be decreased in carbohydrates, fats, or total calories.

13. After teaching a client with renal cancer who is prescribed temsirolimus (Torisel), the nurse assesses the client's understanding. Which statement made by the client indicates a correct understanding of the teaching? a. "I will take this medication with food and plenty of water." b. "I shall keep my appointment at the infusion center each week." c. "I'll limit my intake of green leafy vegetables while on this medication." d. "I must not take this medication if I have an infection or am feeling ill."

ANS: B Temsirolimus is administered as a weekly intravenous infusion. This medication blocks protein that is needed for cell division and therefore inhibits cell cycle progression. This medication is not taken orally, and clients do not need to follow a specific diet.

A nurse reviews the chart and new prescriptions for a client with diabetic ketoacidosis: Vital Signs and Assessment Laboratory Results Medications Blood pressure: 90/62 mm Hg Pulse: 120 beats/min Respiratory rate: 28 breaths/min Urine output: 20 mL/hr via catheter Serum potassium: 2.6 mEq/L Potassium chloride 40 mEq IV bolus STAT Increase IV fluid to 100 mL/hr Which action should the nurse take? a. Administer the potassium and then consult with the provider about the fluid order. b. Increase the intravenous rate and then consult with the provider about the potassium prescription. c. Administer the potassium first before increasing the infusion flow rate. d. Increase the intravenous flow rate before administering the potassium.

ANS: B The client is acutely ill and is severely dehydrated and hypokalemic. The client requires more IV fluids and potassium. However, potassium should not be infused unless the urine output is at least 30 mL/hr. The nurse should first increase the IV rate and then consult with the provider about the potassium.

A nurse reviews laboratory results for a client with diabetes mellitus who is prescribed an intensified insulin regimen: • Fasting blood glucose: 75 mg/dL • Postprandial blood glucose: 200 mg/dL • Hemoglobin A1c level: 5.5% How should the nurse interpret these laboratory findings? a. Increased risk for developing ketoacidosis b. Good control of blood glucose c. Increased risk for developing hyperglycemia d. Signs of insulin resistance

ANS: B The client is maintaining blood glucose levels within the defined ranges for goals in an intensified regimen. Because the client's glycemic control is good, he or she is not at higher risk for ketoacidosis or hyperglycemia and is not showing signs of insulin resistance.

9. After teaching a client with hypertension secondary to renal disease, the nurse assesses the client's understanding. Which statement made by the client indicates a need for additional teaching? a. "I can prevent more damage to my kidneys by managing my blood pressure." b. "If I have increased urination at night, I need to drink less fluid during the day." c. "I need to see the registered dietitian to discuss limiting my protein intake." d. "It is important that I take my antihypertensive medications as directed."

ANS: B The client should not restrict fluids during the day due to increased urination at night. Clients with renal disease may be prescribed fluid restrictions. These clients should be assessed thoroughly for potential dehydration. Increased nocturnal voiding can be decreased by consuming fluids earlier in the day. Blood pressure control is needed to slow the progression of renal dysfunction. When dietary protein is restricted, refer the client to the registered dietitian as needed.

21. A nurse teaches a client who is starting urinary bladder training. Which statement should the nurse include in this clients teaching? a. Use the toilet when you first feel the urge, rather than at specific intervals. b. Try to consciously hold your urine until the scheduled toileting time. c. Initially try to use the toilet at least every half hour for the first 24 hours. d. The toileting interval can be increased once you have been continent for a week.

ANS: B The client should try to hold the urine consciously until the next scheduled toileting time. Toileting should occur at specific intervals during the training. The toileting interval should be no less than every hour. The interval can be increased once the client becomes comfortable with the interval. DIF: Understanding/Comprehension REF: 1357 KEY: Urinary incontinence| patient education MSC: Integrated Process: Teaching/Learning NOT: Client Needs Category: Health Promotion and Maintenance

A nurse cares for a client with a fractured fibula. Which assessment should alert the nurse to take immediate action? a. Pain of 4 on a scale of 0 to 10 b. Numbness in the extremity c. Swollen extremity at the injury site d. Feeling cold while lying in bed

ANS: B The client with numbness and/or tingling of the extremity may be displaying the first signs of acute compartment syndrome. This is an acute problem that requires immediate intervention because of possible decreased circulation. Moderate pain and swelling is an expected assessment after a fracture. These findings can be treated with comfort measures. Being cold can be treated with additional blankets or by increasing the temperature of the room.

A nurse cares for a client who had a long-leg cast applied last week. The client states, "I cannot seem to catch my breath and I feel a bit light-headed." Which action should the nurse take next? a. Auscultate the client's lung fields anteriorly and posteriorly. b. Administer oxygen to keep saturations greater than 92%. c. Check the client's blood glucose level. d. Ask the client to take deep breaths.

ANS: B The client's symptoms are consistent with the development of pulmonary embolism caused by leg immobility in the long cast. The nurse should check the client's pulse oximetry reading and provide oxygen to keep saturations greater than 92%. Auscultating lung fields, checking blood glucose level, or deep breathing will not assist this client.

A nurse assesses a client with a fracture who is being treated with skeletal traction. Which assessment should alert the nurse to urgently contact the health provider? a. Blood pressure increases to 130/86 mm Hg b. Traction weights are resting on the floor c. Oozing of clear fluid is noted at the pin site d. Capillary refill is less than 3 seconds

ANS: B The immediate action of the nurse should be to reapply the weights to give traction to the fracture. The health care provider must be notified that the weights were lying on the floor, and the client should be realigned in bed. The client's blood pressure is slightly elevated; this could be related to pain and muscle spasms resulting from lack of pressure to reduce the fracture. Oozing of clear fluid is normal, as is the capillary refill time.

A nurse observes that a client's anteroposterior (AP) chest diameter is the same as the lateral chest diameter. Which question should the nurse ask the client in response to this finding? a. "Are you taking any medications or herbal supplements?" b. "Do you have any chronic breathing problems?" c. "How often do you perform aerobic exercise?" d. "What is your occupation and what are your hobbies?"

ANS: B The normal chest has a lateral diameter that is twice as large as the AP diameter. When the AP diameter approaches or exceeds the lateral diameter, the client is said to have a barrel chest. Most commonly, barrel chest occurs as a result of a long-term chronic airflow limitation problem, such as chronic obstructive pulmonary disease or severe chronic asthma. It can also be seen in people who have lived at a high altitude for many years. Therefore, an AP chest diameter that is the same as the lateral chest diameter should be rechecked but is not as indicative of underlying disease processes as an AP diameter that exceeds the lateral diameter. Medications, herbal supplements, and aerobic exercise are not associated with a barrel chest. Although occupation and hobbies may expose a client to irritants that can cause chronic lung disorders and barrel chest, asking about chronic breathing problems is more direct and should be asked first.

A nurse cares for an older adult client with multiple fractures. Which action should the nurse take to manage this client's pain? a. Meperidine (Demerol) injections every 4 hours around the clock b. Patient-controlled analgesia (PCA) pump with morphine c. Ibuprofen (Motrin) 600 mg orally every 4 hours PRN for pain d. Morphine 4 mg intravenous push every 2 hours PRN for pain

ANS: B The older adult client should never be treated with meperidine because toxic metabolites can cause seizures. The client should be managed with a PCA pump to control pain best. Motrin most likely would not provide complete pain relief with multiple fractures. IV morphine PRN would not control pain as well as a pump that the client can control.

A client has scoliosis with a 65-degree curve to the spine. What action by the nurse takes priority? a. Allow the client to rest in a position of comfort. b. Assess the client's cardiac and respiratory systems. c. Assist the client with ambulating and position changes. d. Position the client on one side propped with pillows.

ANS: B This degree of curvature of the spine affects cardiac and respiratory function. The nurse's priority is to assess those systems. Positioning is up to the client. The client may or may not need assistance with movement.

A client is having a myelography. What action by the nurse is most important? a. Assess serum aspartate aminotransferase (AST) levels. b. Ensure that informed consent is on the chart. c. Position the client flat after the procedure. d. Reinforce the dressing if it becomes saturated.

ANS: B This diagnostic procedure is invasive and requires informed consent. The AST does not need to be assessed prior to the procedure. The client is positioned with the head of the bed elevated after the test to keep the contrast material out of the brain. The dressing should not become saturated; if it does, the nurse calls the provider.

After teaching a client how to perform diaphragmatic breathing, the nurse assesses the client's understanding. Which action demonstrates that the client correctly understands the teaching? a. The client lays on his or her side with his or her knees bent. b. The client places his or her hands on his or her abdomen. c. The client lays in a prone position with his or her legs straight. d. The client places his or her hands above his or her head.

ANS: B To perform diaphragmatic breathing correctly, the client should place his or her hands on his or her abdomen to create resistance. This type of breathing cannot be performed effectively while lying on the side or with hands over the head. This type of breathing would not be as effective lying prone.

A nurse assesses a client who has diabetes mellitus. Which arterial blood gas values should the nurse identify as potential ketoacidosis in this client? a. pH 7.38, HCO3 - 22 mEq/L, PCO2 38 mm Hg, PO2 98 mm Hg b. pH 7.28, HCO3 - 18 mEq/L, PCO2 28 mm Hg, PO2 98 mm Hg c. pH 7.48, HCO3 - 28 mEq/L, PCO2 38 mm Hg, PO2 98 mm Hg d. pH 7.32, HCO3 - 22 mEq/L, PCO2 58 mm Hg, PO2 88 mm Hg

ANS: B When the lungs can no longer offset acidosis, the pH decreases to below normal. A client who has diabetic ketoacidosis would present with arterial blood gas values that show primary metabolic acidosis with decreased bicarbonate levels and a compensatory respiratory alkalosis with decreased carbon dioxide levels.

A client is having a bone marrow biopsy today. What action by the nurse takes priority? a. Administer pain medication first. b. Ensure valid consent is on the chart. c. Have the client shower in the morning. d. Premedicate the client with sedatives.

ANS: B A bone marrow biopsy is an invasive procedure that requires informed consent. Pain medication and sedation are important components of care for this client but do not take priority. The client may or may not need or be able to shower.

A nurse obtains a focused health history for a client who is scheduled for magnetic resonance angiography. Which priority question should the nurse ask before the test? a. Have you had a recent blood transfusion? b. Do you have allergies to iodine or shellfish? c. Are you taking any cardiac medications? d. Do you currently use oral contraceptives?

ANS: B Allergies to iodine and/or shellfish need to be explored because the client may have a similar reaction to the dye used in the procedure. In some cases, the client may need to be medicated with antihistamines or steroids before the test is given. A recent blood transfusion or current use of cardiac medications or oral contraceptives would not affect the angiography.

A nurse plans care for a client who has a hypoactive response to a test of deep tendon reflexes. Which intervention should the nurse include in this clients plan of care? a. Check bath water temperature with a thermometer. b. Provide the client with assistance when ambulating. c. Place elastic support hose on the clients legs. d. Assess the clients feet for wounds each shift.

ANS: B Hypoactive deep tendon reflexes and loss of vibration sense can impair balance and coordination, predisposing the client to falls. The nurse should plan to provide the client with ambulation assistance to prevent injury. The other interventions do not address the clients problem.

A nurse assesses a client who is recovering from a lumbar puncture (LP). Which complication of this procedure should alert the nurse to urgently contact the health care provider? a. Weak pedal pulses b. Nausea and vomiting c. Increased thirst d. Hives on the chest

ANS: B The nurse should immediately contact the provider if the client experiences a severe headache, nausea, vomiting, photophobia, or a change in level of consciousness after an LP, which are all signs of increased intracranial pressure. Weak pedal pulses, increased thirst, and hives are not complications of an LP.

A nurse assesses a client who has a chest tube. For which manifestations should the nurse immediately intervene? (Select all that apply.) a. Production of pink sputum b. Tracheal deviation c. Sudden onset of shortness of breath d. Pain at insertion site e. Drainage of 75 mL/hr

ANS: B, C Tracheal deviation and sudden onset of shortness of breath are manifestations of a tension pneumothorax. The nurse must intervene immediately for this emergency situation. Pink sputum is associated with pulmonary edema and is not a complication of a chest tube. Pain at the insertion site and drainage of 75 mL/hr are normal findings with a chest tube.

A client with Paget's disease is hospitalized for an unrelated issue. The client reports pain and it is not yet time for more medication. What comfort measures can the nurse delegate to the unlicensed assistive personnel (UAP)? (Select all that apply.) a. Administering ibuprofen (Motrin) b. Applying a heating pad c. Providing a massage d. Referring the client to a support group e. Using a bed cradle to lift sheets off the feet

ANS: B, C Comfort measures for Paget's disease include heat and massage. Administering medications and referrals are done by the nurse. A bed cradle is not necessary.

A nurse assesses a client who has a chest tube. For which manifestations should the nurse immediately intervene? (Select all that apply.) a. Production of pink sputum b. Tracheal deviation c. Sudden onset of shortness of breath d. Pain at insertion site e. Drainage of 75 mL/hr

ANS: B, C Tracheal deviation and sudden onset of shortness of breath are manifestations of a tension pneumothorax. The nurse must intervene immediately for this emergency situation. Pink sputum is associated with pulmonary edema and is not a complication of a chest tube. Pain at the insertion site and drainage of 75 mL/hr are normal findings with a chest tube.

A nurse assesses clients on the medical-surgical unit. Which clients are at risk for kidney problems? (Select all that apply.) a. A 24-year-old pregnant woman prescribed prenatal vitamins b. A 32-year-old bodybuilder taking synthetic creatine supplements c. A 56-year-old who is taking metformin for diabetes mellitus d. A 68-year-old taking high-dose nonsteroidal anti-inflammatory drugs (NSAIDs) for chronic back pain e. A 75-year-old with chronic obstructive pulmonary disease (COPD) who is prescribed an albuterol nebulizer

ANS: B, C, D Many medications can affect kidney function. Clients who take synthetic creatine supplements, metformin, and high-dose or long-term NSAIDs are at risk for kidney dysfunction. Prenatal vitamins and albuterol nebulizers do not place these clients at risk.

1. A nurse assesses a client who has a family history of polycystic kidney disease (PKD). For which clinical manifestations should the nurse assess? (Select all that apply.) a. Nocturia b. Flank pain c. Increased abdominal girth d. Dysuria e. Hematuria f. Diarrhea

ANS: B, C, E Clients with PKD experience abdominal distention that manifests as flank pain and increased abdominal girth. Bloody urine is also present with tissue damage secondary to PKD. Clients with PKD often experience constipation, but would not report nocturia or dysuria.

A nurse assesses a client with chronic obstructive pulmonary disease. Which questions should the nurse ask to determine the client's activity tolerance? (Select all that apply.) a. "What color is your sputum?" b. "Do you have any difficulty sleeping?" c. "How long does it take to perform your morning routine?" d. "Do you walk upstairs every day?" e. "Have you lost any weight lately?"

ANS: B, C, E Difficulty sleeping could indicate worsening breathlessness, as could taking longer to perform activities of daily living. Weight loss could mean increased dyspnea as the client becomes too fatigued to eat. The color of the client's sputum would not assist in determining activity tolerance. Asking whether the client walks upstairs every day is not as pertinent as determining if the client becomes short of breath on walking upstairs, or if the client goes upstairs less often than previously.

A nurse works in a gerontology clinic. What age-related changes cause the nurse to alter standard assessment techniques from those used for younger adults? (Select all that apply.) a. Dentition deteriorates with more cavities. b. Nail beds may be thickened or discolored. c. Progressive loss of hair occurs with age. d. Sclerae begin to turn yellow or pale. e. Skin becomes dry as the client ages.

ANS: B, C, E Common findings in older adults include thickened or discolored nail beds, dry skin, and thinning hair. The nurse adapts to these changes by altering assessment techniques. Having more dental caries and changes in the sclerae are not normal age-related changes.

A nurse assesses a client who has a mediastinal chest tube. Which symptoms require the nurse's immediate intervention? (Select all that apply.) a. Production of pink sputum b. Tracheal deviation c. Pain at insertion site d. Sudden onset of shortness of breath e. Drainage greater than 70 mL/hr f. Disconnection at Y site

ANS: B, D, E, F Immediate intervention is warranted if the client has tracheal deviation because this could indicate a tension pneumothorax. Sudden shortness of breath could indicate dislodgment of the tube, occlusion of the tube, or pneumothorax. Drainage greater than 70 mL/hr could indicate hemorrhage. Disconnection at the Y site could result in air entering the tubing. Production of pink sputum, oxygen saturation less than 95%, and pain at the insertion site are not signs/symptoms that would require immediate intervention.

A nurse assesses a client who has a mediastinal chest tube. Which symptoms require the nurse's immediate intervention? (Select all that apply.) a. Production of pink sputum b. Tracheal deviation c. Pain at insertion site d. Sudden onset of shortness of breath e. Drainage greater than 70 mL/hr f. Disconnection at Y site

ANS: B, D, E, F Immediate intervention is warranted if the client has tracheal deviation because this could indicate a tension pneumothorax. Sudden shortness of breath could indicate dislodgment of the tube, occlusion of the tube, or pneumothorax. Drainage greater than 70 mL/hr could indicate hemorrhage. Disconnection at the Y site could result in air entering the tubing. Production of pink sputum, oxygen saturation less than 95%, and pain at the insertion site are not signs/symptoms that would require immediate intervention.

4. A nurse assesses a client with a fungal urinary tract infection (UTI). Which assessments should the nurse complete? (Select all that apply.) a. Palpate the kidneys and bladder. b. Assess the medical history and current medical problems. c. Perform a bladder scan to assess post-void residual. d. Inquire about recent travel to foreign countries. e. Obtain a current list of medications.

ANS: B, E Clients who are severely immunocompromised or who have diabetes mellitus are more prone to fungal UTIs. The nurse should assess for these factors by asking about medical history, current medical problems, and the current medication list. A physical examination and a post-void residual may be needed, but not until further information is obtained indicating that these examinations are necessary. Travel to foreign countries probably would not be important because, even if exposed, the client needs some degree of compromised immunity to develop a fungal UTI. DIF: Applying/Application REF: 1354 KEY: Cystitis| assessment/diagnostic examination MSC: Integrated Process: Nursing Process: Assessment NOT: Client Needs Category: Physiological Integrity: Reduction of Risk Potential

3. A nurse teaches clients about the difference between urge incontinence and stress incontinence. Which statements should the nurse include in this education? (Select all that apply.) a. Urge incontinence involves a post-void residual volume less than 50 mL. b. Stress incontinence occurs due to weak pelvic floor muscles. c. Stress incontinence usually occurs in people with dementia. d. Urge incontinence can be managed by increasing fluid intake. e. Urge incontinence occurs due to abnormal bladder contractions.

ANS: B, E Clients who suffer from stress incontinence have weak pelvic floor muscles or urethral sphincter and cannot tighten their urethra sufficiently to overcome the increased detrusor pressure. Stress incontinence is common after childbirth, when the pelvic muscles are stretched and weakened from pregnancy and delivery. Urge incontinence occurs in people who cannot suppress the contraction signal from the detrusor muscle. Abnormal detrusor contractions may be a result of neurologic abnormalities including dementia, or may occur with no known abnormality. Post-void residual is associated with reflex incontinence, not with urge incontinence or stress incontinence. Management of urge incontinence includes decreasing fluid intake, especially in the evening hours. DIF: Understanding/Comprehension REF: 1352 Test Bank - Medical-Surgical Nursing: Concepts for Interprofessional Collaborative Care 9e 565 KEY: Urinary incontinence| patient education MSC: Integrated Process: Nursing Process: Assessment NOT: Client Needs Category: Physiological Integrity: Physiological Adaptation

The nurse is caring for a client with lung cancer who states, "I don't want any pain medication because I am afraid I'll become addicted." How should the nurse respond? a. "I will ask the provider to change your medication to a drug that is less potent." b. "Would you like me to use music therapy to distract you from your pain?" c. "It is unlikely you will become addicted when taking medicine for pain." d. "Would you like me to give you acetaminophen (Tylenol) instead?"

ANS: C Clients should be encouraged to take their pain medications; addiction usually is not an issue with a client in pain. The nurse would not request that the pain medication be changed unless it was not effective. Other methods to decrease pain can be used, in addition to pain medication.

A nurse is teaching a client who has cystic fibrosis (CF). Which statement should the nurse include in this client's teaching? a. "Take an antibiotic each day." b. "Contact your provider to obtain genetic screening." c. "Eat a well-balanced, nutritious diet." d. "Plan to exercise for 30 minutes every day."

ANS: C Clients with CF often are malnourished due to vitamin deficiency and pancreatic malfunction. Maintaining nutrition is essential. Daily antibiotics and daily exercise are not essential actions. Genetic screening would not help the client manage CF better.

A nurse cares for a client with chronic obstructive pulmonary disease (COPD) who appears thin and disheveled. Which question should the nurse ask first? a. "Do you have a strong support system?" b. "What do you understand about your disease?" c. "Do you experience shortness of breath with basic activities?" d. "What medications are you prescribed to take each day?"

ANS: C Clients with severe COPD may not be able to perform daily activities, including bathing and eating, because of excessive shortness of breath. The nurse should ask the client if shortness of breath is interfering with basic activities. Although the nurse should know about the client's support systems, current knowledge, and medications, these questions do not address the client's appearance.

A nurse cares for a female client who has a family history of cystic fibrosis. The client asks, "Will my children have cystic fibrosis?" How should the nurse respond? a. "Since many of your family members are carriers, your children will also be carriers of the gene." b. "Cystic fibrosis is an autosomal recessive disorder. If you are a carrier, your children will have the disorder." c. "Since you have a family history of cystic fibrosis, I would encourage you and your partner to be tested." d. "Cystic fibrosis is caused by a protein that controls the movement of chloride. Adjusting your diet will decrease the spread of this disorder."

ANS: C Cystic fibrosis is an autosomal recessive disorder in which both gene alleles must be mutated for the disorder to be expressed. The nurse should encourage both the client and partner to be tested for the abnormal gene. The other statements are not true.

After teaching a client who is prescribed a long-acting beta2 agonist medication, a nurse assesses the client's understanding. Which statement indicates the client comprehends the teaching? a. "I will carry this medication with me at all times in case I need it." b. "I will take this medication when I start to experience an asthma attack." c. "I will take this medication every morning to help prevent an acute attack." d. "I will be weaned off this medication when I no longer need it."

ANS: C Long-acting beta2 agonist medications will help prevent an acute asthma attack because they are long acting. The client will take this medication every day for best effect. The client does not have to always keep this medication with him or her because it is not used as a rescue medication. This is not the medication the client will use during an acute asthma attack because it does not have an immediate onset of action. The client will not be weaned off this medication because this is likely to be one of his or her daily medications.

A nurse cares for a client with chronic obstructive pulmonary disease (COPD). The client states that he no longer enjoys going out with his friends. How should the nurse respond? a. "There are a variety of support groups for people who have COPD." b. "I will ask your provider to prescribe you with an antianxiety agent." c. "Share any thoughts and feelings that cause you to limit social activities." d. "Friends can be a good support system for clients with chronic disorders."

ANS: C Many clients with moderate to severe COPD become socially isolated because they are embarrassed by frequent coughing and mucus production. They also can experience fatigue, which limits their activities. The nurse needs to encourage the client to verbalize thoughts and feelings so that appropriate interventions can be selected. Joining a support group would not decrease feelings of social isolation if the client does not verbalize feelings. Antianxiety agents will not help the client with social isolation. Encouraging a client to participate in activities without verbalizing concerns also would not be an effective strategy for decreasing social isolation.

After teaching a client who is prescribed salmeterol (Serevent), the nurse assesses the client's understanding. Which statement by the client indicates a need for additional teaching? a. "I will be certain to shake the inhaler well before I use it." b. "It may take a while before I notice a change in my asthma." c. "I will use the drug when I have an asthma attack." d. "I will be careful not to let the drug escape out of my nose and mouth."

ANS: C Salmeterol is designed to prevent an asthma attack; it does not relieve or reverse symptoms. Salmeterol has a slow onset of action; therefore, it should not be used as a rescue drug. The drug must be shaken well because it has a tendency to separate easily. Poor technique on the client's part allows the drug to escape through the nose and mouth.

A nurse cares for a client who tests positive for alpha1-antitrypsin (AAT) deficiency. The client asks, "What does this mean?" How should the nurse respond? a. "Your children will be at high risk for the development of chronic obstructive pulmonary disease." b. "I will contact a genetic counselor to discuss your condition." c. "Your risk for chronic obstructive pulmonary disease is higher, especially if you smoke." d. "This is a recessive gene and should have no impact on your health."

ANS: C The gene for AAT is a recessive gene. Clients with only one allele produce enough AAT to prevent chronic obstructive pulmonary disease (COPD) unless the client smokes. A client with two alleles is at high risk for COPD even if not exposed to smoke or other irritants. The client is a carrier, and children may or may not be at high risk depending on the partner's AAT levels. Contacting a genetic counselor may be helpful but does not address the client's current question.

The nurse instructs a client on how to correctly use an inhaler with a spacer. In which order should these steps occur? 1. "Press down firmly on the canister to release one dose of medication." 2. "Breathe in slowly and deeply." 3. "Shake the whole unit vigorously three or four times." 4. "Insert the mouthpiece of the inhaler into the nonmouthpiece end of the spacer." 5. "Place the mouthpiece into your mouth, over the tongue, and seal your lips tightly around the mouthpiece." 6. "Remove the mouthpiece from your mouth, keep your lips closed, and hold your breath for at least 10 seconds." a. 2, 3, 4, 5, 6, 1 b. 3, 4, 5, 1, 6, 2 c. 4, 3, 5, 1, 2, 6 d. 5, 3, 6, 1, 2, 4

ANS: C The proper order for correctly using an inhaler with a spacer is as follows. Insert the mouthpiece of the inhaler into the nonmouthpiece end of the spacer. Shake the whole unit vigorously three or four times. Place the mouthpiece into the mouth, over the tongue, and seal the lips tightly around it. Press down firmly on the canister of the inhaler to release one dose of medication into the spacer. Breathe in slowly and deeply. Remove the mouthpiece from the mouth, and, keeping the lips closed, hold the breath for at least 10 seconds. Then breathe out slowly. Wait at least 1 minute between puffs.

A nurse cares for a client who has a chest tube. When would this client be at highest risk for developing a pneumothorax? a. When the insertion site becomes red and warm to the touch b. When the tube drainage decreases and becomes sanguineous c. When the client experiences pain at the insertion site d. When the tube becomes disconnected from the drainage system

ANS: D Intrathoracic pressures are less than atmospheric pressures; therefore, if the chest tube becomes disconnected from the drainage system, air can be sucked into the pleural space and cause a pneumothorax. A red, warm, and painful insertion site does not increase the client's risk for a pneumothorax. Tube drainage should decrease and become serous as the client heals. Sanguineous drainage is a sign of bleeding but does not increase the client's risk for a pneumothorax.

The nurse is caring for a client who is prescribed a long-acting beta2 agonist. The client states, "The medication is too expensive to use every day. I only use my inhaler when I have an attack." How should the nurse respond? a. "You are using the inhaler incorrectly. This medication should be taken daily." b. "If you decrease environmental stimuli, it will be okay for you to use the inhaler only for asthma attacks." c. "Tell me more about your fears related to feelings of breathlessness." d. "It is important to use this type of inhaler every day. Let's identify potential community services to help you."

ANS: D Long-acting beta2 agonists should be used every day to prevent asthma attacks. This medication should not be taken when an attack starts. Asthma medications can be expensive. Telling the client that he or she is using the inhaler incorrectly does not address the client's financial situation, which is the main issue here. Clients with limited incomes should be provided with community resources. Asking the client about fears related to breathlessness does not address the client's immediate concerns.

A nurse evaluates the following arterial blood gas and vital sign results for a client with chronic obstructive pulmonary disease (COPD): Arterial Blood Gas Results Vital Signs pH = 7.32 PaCO2 = 62 mm Hg PaO2 = 46 mm Hg HCO3- = 28 mEq/L Heart rate = 110 beats/min Respiratory rate = 12 breaths/min Blood pressure = 145/65 mm Hg Oxygen saturation = 76% Which action should the nurse take first? a. Administer a short-acting beta2 agonist inhaler. b. Document the findings as normal for a client with COPD. c. Teach the client diaphragmatic breathing techniques. d. Initiate oxygenation therapy to increase saturation to 92%.

ANS: D Oxygen should be administered to a client who is hypoxic even if the client has COPD and is a carbon dioxide retainer. The other interventions do not address the client's hypoxia, which is the priority.

A nurse cares for a client who has a pleural chest tube. Which action should the nurse take to ensure safe use of this equipment? a. Strip the tubing to minimize clot formation and ensure patency. b. Secure tubing junctions with clamps to prevent accidental disconnections. c. Connect the chest tube to wall suction at the level prescribed by the provider. d. Keep padded clamps at the bedside for use if the drainage system is interrupted.

ANS: D Padded clamps should be kept at the bedside for use if the drainage system becomes dislodged or is interrupted. The nurse should never strip the tubing. Tubing junctions should be taped, not clamped. Wall suction should be set at the level indicated by the device's manufacturer, not the provider.

A nurse assesses several clients who have a history of asthma. Which client should the nurse assess first? a. A 66-year-old client with a barrel chest and clubbed fingernails b. A 48-year-old client with an oxygen saturation level of 92% at rest c. A 35-year-old client who has a longer expiratory phase than inspiratory phase d. A 27-year-old client with a heart rate of 120 beats/min

ANS: D Tachycardia can indicate hypoxemia as the body tries to circulate the oxygen that is available. A barrel chest is not an emergency finding. Likewise, a pulse oximetry level of 92% is not considered an acute finding. The expiratory phase is expected to be longer than the inspiratory phase in someone with airflow limitation.

A nurse cares for a client who had a chest tube placed 6 hours ago and refuses to take deep breaths because of the pain. Which action should the nurse take? a. Ambulate the client in the hallway to promote deep breathing. b. Auscultate the client's anterior and posterior lung fields. c. Encourage the client to take shallow breaths to help with the pain. d. Administer pain medication and encourage the client to take deep breaths.

ANS: D A chest tube is placed in the pleural space and may be uncomfortable for a client. The nurse should provide pain medication to minimize discomfort and encourage the client to take deep breaths. The other responses do not address the client's discomfort and need to take deep breaths to prevent complications.

A nurse assesses a client after a thoracentesis. Which assessment finding warrants immediate action? a. The client rates pain as a 5/10 at the site of the procedure. b. A small amount of drainage from the site is noted. c. Pulse oximetry is 93% on 2 liters of oxygen. d. The trachea is deviated toward the opposite side of the neck.

ANS: D A deviated trachea is a manifestation of a tension pneumothorax, which is a medical emergency. The other findings are normal or near normal.

The nurse is assessing four clients with musculoskeletal disorders. The nurse should assess the client with which laboratory result first? a. Serum alkaline phosphatase (ALP): 108 units/L b. Serum aspartate aminotransferase (AST): 26 units/L c. Serum calcium: 10.2 mg/dL d. Serum phosphorus: 2 mg/dL

ANS: D A normal serum phosphorus level is 3 to 4.5 mg/dL; a level of 2 mg/dL is low, and this client should be assessed first. The values for serum ALP, AST, and calcium are all within normal ranges.

The nurse sees several clients with osteoporosis. For which client would bisphosphonates not be a good option? a. Client with diabetes who has a serum creatinine of 0.8 mg/dL b. Client who recently fell and has vertebral compression fractures c. Hypertensive client who takes calcium channel blockers d. Client with a spinal cord injury who cannot tolerate sitting up

ANS: D Clients on bisphosphonates must be able to sit upright for 30 to 60 minutes after taking them. The client who cannot tolerate sitting up is not a good candidate for this class of drug. Poor renal function also makes clients bad candidates for this drug, but the client with a creatinine of 0.8 mg/dL is within normal range. Diabetes and hypertension are not related unless the client also has renal disease. The client who recently fell and sustained fractures is a good candidate for this drug if the fractures are related to osteoporosis.

A nurse teaches a client with diabetes mellitus who is experiencing numbness and reduced sensation. Which statement should the nurse include in this client's teaching to prevent injury? a. "Examine your feet using a mirror every day." b. "Rotate your insulin injection sites every week." c. "Check your blood glucose level before each meal." d. "Use a bath thermometer to test the water temperature."

ANS: D Clients with diminished sensory perception can easily experience a burn injury when bathwater is too hot. Instead of checking the temperature of the water by feeling it, they should use a thermometer. Examining the feet daily does not prevent injury, although daily foot examinations are important to find problems so they can be addressed. Rotating insulin and checking blood glucose levels will not prevent injury.

A trauma nurse cares for several clients with fractures. Which client should the nurse identify as at highest risk for developing deep vein thrombosis? a. An 18-year-old male athlete with a fractured clavicle b. A 36-year old female with type 2 diabetes and fractured ribs c. A 55-year-old woman prescribed aspirin for rheumatoid arthritis d. A 74-year-old man who smokes and has a fractured pelvis

ANS: D Deep vein thrombosis (DVT) as a complication with bone fractures occurs more often when fractures are sustained in the lower extremities and the client has additional risk factors for thrombus formation. Other risk factors include obesity, smoking, oral contraceptives, previous thrombus events, advanced age, venous stasis, and heart disease. The other clients do not have risk factors for DVT.

When teaching a client recently diagnosed with type 1 diabetes mellitus, the client states, "I will never be able to stick myself with a needle." How should the nurse respond? a. "I can give your injections to you while you are here in the hospital." b. "Everyone gets used to giving themselves injections. It really does not hurt." c. "Your disease will not be managed properly if you refuse to administer the shots." d. "Tell me what it is about the injections that are concerning you."

ANS: D Devote as much teaching time as possible to insulin injection and blood glucose monitoring. Clients with newly diagnosed diabetes are often fearful of giving themselves injections. If the client is worried about giving the injections, it is best to try to find out what specifically is causing the concern, so it can be addressed. Giving the injections for the client does not promote self-care ability. Telling the client that others give themselves injections may cause the client to feel bad. Stating that you don't know another way to manage the disease is dismissive of the client's concerns.

19. A nurse teaches a client with functional urinary incontinence. Which statement should the nurse include in this clients teaching? a. You must clean around your catheter daily with soap and water. b. Wash the vaginal weights with a 10% bleach solution after each use. c. Operations to repair your bladder are available, and you can consider these. d. Buy slacks with elastic waistbands that are easy to pull down.

ANS: D Functional urinary incontinence occurs as the result of problems not related to the clients bladder, such as trouble ambulating or difficulty accessing the toilet. One goal is that the client will be able to manage his or her clothing independently. Elastic waistband slacks that are easy to pull down can help the client get on the toilet in time to void. The other instructions do not relate to functional urinary incontinence. DIF: Applying/Application REF: 1359 KEY: Urinary incontinence MSC: Integrated Process: Teaching/Learning NOT: Client Needs Category: Physiological Integrity: Basic Care and Comfort

A nurse reviews the urinalysis of a client and notes the presence of glucose. Which action should the nurse take? a. Document findings and continue to monitor the client. b. Contact the provider and recommend a 24-hour urine test. c. Review the client's recent dietary selections. d. Perform a capillary artery glucose assessment.

ANS: D Glucose normally is not found in the urine. The normal renal threshold for glucose is about 220 mg/dL, which means that a person whose blood glucose is less than 220 mg/dL will not have glucose in the urine. A positive finding for glucose on urinalysis indicates high blood sugar. The most appropriate action would be to perform a capillary artery glucose assessment. The client needs further evaluation for this abnormal result, therefore, documenting and continuing to monitor is not appropriate. Requesting a 24-hour urine test or reviewing the client's dietary selections will not assist the nurse to make a clinical decision related to this abnormality.

A nurse teaches a client who is prescribed an insulin pump. Which statement should the nurse include in this client's discharge education? a. "Test your urine daily for ketones." b. "Use only buffered insulin in your pump." c. "Store the insulin in the freezer until you need it." d. "Change the needle every 3 days."

ANS: D Having the same needle remain in place through the skin for longer than 3 days drastically increases the risk for infection in or through the delivery system. Having an insulin pump does not require the client to test for ketones in the urine. Insulin should not be frozen. Insulin is not buffered.

A nurse reviews laboratory results for a client with diabetes mellitus who presents with polyuria, lethargy, and a blood glucose of 560 mg/dL. Which laboratory result should the nurse correlate with the client's polyuria? a. Serum sodium: 163 mEq/L b. Serum creatinine: 1.6 mg/dL c. Presence of urine ketone bodies d. Serum osmolarity: 375 mOsm/kg

ANS: D Hyperglycemia causes hyperosmolarity of extracellular fluid. This leads to polyuria from an osmotic diuresis. The client's serum osmolarity is high. The client's sodium would be expected to be high owing to dehydration. Serum creatinine and urine ketone bodies are not related to the polyuria.

8. A nurse plans care for a client with overflow incontinence. Which intervention should the nurse include in this clients plan of care to assist with elimination? a. Stroke the medial aspect of the thigh. b. Use intermittent catheterization. c. Provide digital anal stimulation. d. Use the Valsalva maneuver.

ANS: D In clients with overflow incontinence, the voiding reflex arc is not intact. Mechanical pressure, such as that achieved through the Valsalva maneuver (holding the breath and bearing down as if to defecate), can initiate voiding. Stroking the medial aspect of the thigh or providing digital anal stimulation requires the reflex arc to be intact to initiate elimination. Due to the high risk for infection, intermittent catheterization should only be implemented when other interventions are not successful. DIF: Applying/Application REF: 1358 KEY: Urinary incontinence MSC: Integrated Process: Nursing Process: Implementation NOT: Client Needs Category: Physiological Integrity: Basic Care and Comfort

A nurse reviews the laboratory results of a client who is receiving intravenous insulin. Which should alert the nurse to intervene immediately? a. Serum chloride level of 98 mmol/L b. Serum calcium level of 8.8 mg/dL c. Serum sodium level of 132 mmol/L d. Serum potassium level of 2.5 mmol/L

ANS: D Insulin activates the sodium-potassium ATPase pump, increasing the movement of potassium from the extracellular fluid into the intracellular fluid, resulting in hypokalemia. In hyperglycemia, hypokalemia can also result from excessive urine loss of potassium. The chloride level is normal. The calcium and sodium levels are slightly low, but this would not be related to hyperglycemia and insulin administration.

A nurse cares for a client who has a chest tube. When would this client be at highest risk for developing a pneumothorax? a. When the insertion site becomes red and warm to the touch b. When the tube drainage decreases and becomes sanguineous c. When the client experiences pain at the insertion site d. When the tube becomes disconnected from the drainage system

ANS: D Intrathoracic pressures are less than atmospheric pressures; therefore, if the chest tube becomes disconnected from the drainage system, air can be sucked into the pleural space and cause a pneumothorax. A red, warm, and painful insertion site does not increase the client's risk for a pneumothorax. Tube drainage should decrease and become serous as the client heals. Sanguineous drainage is a sign of bleeding but does not increase the client's risk for a pneumothorax.

The nurse is caring for a client who is prescribed a long-acting beta2 agonist. The client states, "The medication is too expensive to use every day. I only use my inhaler when I have an attack." How should the nurse respond? a. "You are using the inhaler incorrectly. This medication should be taken daily." b. "If you decrease environmental stimuli, it will be okay for you to use the inhaler only for asthma attacks." c. "Tell me more about your fears related to feelings of breathlessness." d. "It is important to use this type of inhaler every day. Let's identify potential community services to help you."

ANS: D Long-acting beta2 agonists should be used every day to prevent asthma attacks. This medication should not be taken when an attack starts. Asthma medications can be expensive. Telling the client that he or she is using the inhaler incorrectly does not address the client's financial situation, which is the main issue here. Clients with limited incomes should be provided with community resources. Asking the client about fears related to breathlessness does not address the client's immediate concerns.

A nurse teaches a client with type 1 diabetes mellitus. Which statement should the nurse include in this client's teaching to decrease the client's insulin needs? a. "Limit your fluid intake to 2 liters a day." b. "Animal organ meat is high in insulin." c. "Limit your carbohydrate intake to 80 grams a day." d. "Walk at a moderate pace for 1 mile daily."

ANS: D Moderate exercise such as walking helps regulate blood glucose levels on a daily basis and results in lowered insulin requirements for clients with type 1 diabetes mellitus. Restricting fluids and eating organ meats will not reduce insulin needs. People with diabetes need at least 130 grams of carbohydrates each day.

After teaching a client with type 2 diabetes mellitus who is prescribed nateglinide (Starlix), the nurse assesses the client's understanding. Which statement made by the client indicates a correct understanding of the prescribed therapy? a. "I'll take this medicine during each of my meals." b. "I must take this medicine in the morning when I wake." c. "I will take this medicine before I go to bed." d. "I will take this medicine immediately before I eat."

ANS: D Nateglinide is an insulin secretagogue that is designed to increase meal-related insulin secretion. It should be taken immediately before each meal. The medication should not be taken without eating as it will decrease the client's blood glucose levels. The medication should be taken before meals instead of during meals.

A nurse cares for a client who had a wrist cast applied 3 days ago. The client states, "The cast is loose enough to slide off." How should the nurse respond? a. "Keep your arm above the level of your heart." b. "As your muscles atrophy, the cast is expected to loosen." c. "I will wrap a bandage around the cast to prevent it from slipping." d. "You need a new cast now that the swelling is decreased."

ANS: D Often the surrounding soft tissues may be swollen considerably when the cast is initially applied. After the swelling has resolved, if the cast is loose enough to permit two or more fingers between the cast and the client's skin, the cast needs to be replaced. Elevating the arm will not solve the problem, and the client's muscles should not atrophy while in a cast for 6 weeks or less. An elastic bandage will not prevent slippage of the cast.

A nurse evaluates the following arterial blood gas and vital sign results for a client with chronic obstructive pulmonary disease (COPD): pH = 7.32 PaCO2 = 62 mm Hg PaO2 = 46 mm Hg HCO3- = 28 mEq/L Heart rate = 110 beats/min Respiratory rate = 12 breaths/min Blood pressure = 145/65 mm Hg Oxygen saturation = 76% Which action should the nurse take first? a. Administer a short-acting beta2 agonist inhaler. b. Document the findings as normal for a client with COPD. c. Teach the client diaphragmatic breathing techniques. d. Initiate oxygenation therapy to increase saturation to 92%.

ANS: D Oxygen should be administered to a client who is hypoxic even if the client has COPD and is a carbon dioxide retainer. The other interventions do not address the client's hypoxia, which is the priority.

A nurse cares for a client who has a pleural chest tube. Which action should the nurse take to ensure safe use of this equipment? a. Strip the tubing to minimize clot formation and ensure patency. b. Secure tubing junctions with clamps to prevent accidental disconnections. c. Connect the chest tube to wall suction at the level prescribed by the provider. d. Keep padded clamps at the bedside for use if the drainage system is interrupted.

ANS: D Padded clamps should be kept at the bedside for use if the drainage system becomes dislodged or is interrupted. The nurse should never strip the tubing. Tubing junctions should be taped, not clamped. Wall suction should be set at the level indicated by the device's manufacturer, not the provider.

A nurse obtains the health history of a client with a fractured femur. Which factor identified in the client's history should the nurse recognize as an aspect that may impede healing of the fracture? a. Sedentary lifestyle b. A 30-pack-year smoking history c. Prescribed oral contraceptives d. Paget's disease

ANS: D Paget's disease and bone cancer can cause pathologic fractures such as a fractured femur that do not achieve total healing. The other factors do not impede healing but may cause other health risks.

A client is in the family practice clinic reporting a severe "cold" that started 4 days ago. On examination, the nurse notes the client also has a severe headache and muscle aches. What action by the nurse is best? a. Educate the client on oseltamivir (Tamiflu). b. Facilitate admission to the hospital. c. Instruct the client to have a flu vaccine. d. Teach the client to sneeze in the upper sleeve.

ANS: D Sneezing and coughing into one's sleeve helps prevent the spread of upper respiratory infections. The client does have manifestations of the flu (influenza), but it is too late to start antiviral medications; to be effective, they must be started within 24 to 48 hours of symptom onset. The client does not need hospital admission. The client should be instructed to have a flu vaccination, but now that he or she has the flu, vaccination will have to wait until next year.

A nurse assesses several clients who have a history of asthma. Which client should the nurse assess first? a. A 66-year-old client with a barrel chest and clubbed fingernails b. A 48-year-old client with an oxygen saturation level of 92% at rest c. A 35-year-old client who has a longer expiratory phase than inspiratory phase d. A 27-year-old client with a heart rate of 120 beats/min

ANS: D Tachycardia can indicate hypoxemia as the body tries to circulate the oxygen that is available. A barrel chest is not an emergency finding. Likewise, a pulse oximetry level of 92% is not considered an acute finding. The expiratory phase is expected to be longer than the inspiratory phase in someone with airflow limitation.

17. A nurse cares for a client who is scheduled for the surgical creation of an ileal conduit. The client states, I am anxious about having an ileal conduit. What is it like to have this drainage tube? How should the nurse respond? a. I will ask the provider to prescribe you an antianxiety medication. b. Would you like to discuss the procedure with your doctor once more? c. I think it would be nice to not have to worry about finding a bathroom. d. Would you like to speak with someone who has an ileal conduit?

ANS: D The goal for the client who is scheduled to undergo a procedure such as an ileal conduit is to have a positive self-image and a positive attitude about his or her body. Discussing the procedure candidly with someone who has undergone the same procedure will foster such feelings, especially when the current client has an opportunity to ask questions and voice concerns to someone with first-hand knowledge. Medications for anxiety will not promote a positive self-image and a positive attitude, nor will discussing the procedure once more with the physician or hearing the nurses opinion. DIF: Applying/Application REF: 1367 KEY: Urothelial cancer| psychosocial response| coping MSC: Integrated Process: Caring NOT: Client Needs Category: Psychosocial Integrity

14. A nurse assesses clients on the medical-surgical unit. Which client is at greatest risk for bladder cancer? a. A 25-year-old female with a history of sexually transmitted diseases b. A 42-year-old male who has worked in a lumber yard for 10 years c. A 55-year-old female who has had numerous episodes of bacterial cystitis d. An 86-year-old male with a 50pack-year cigarette smoking history

ANS: D The greatest risk factor for bladder cancer is a long history of tobacco use. The other factors would not necessarily contribute to the development of this specific type of cancer. DIF: Remembering/Knowledge REF: 1365 KEY: Urothelial cancer| health screening MSC: Integrated Process: Nursing Process: Assessment NOT: Client Needs Category: Safe and Effective Care Environment: Management of Care

A client is admitted with a large draining wound on the leg. What action does the nurse take first? a. Administer ordered antibiotics. b. Insert an intravenous line. c. Give pain medications if needed. d. Obtain cultures of the leg wound.

ANS: D The nurse first obtains wound cultures prior to administering broad-spectrum antibiotics. The nurse would need to start the IV prior to giving the antibiotics as they will most likely be parenteral. Pain should be treated but that is not the priority.

7. A nurse assesses a client who is recovering from a radical nephrectomy for renal cell carcinoma. The nurse notes that the client's blood pressure has decreased from 134/90 to 100/56 mm Hg and urine output is 20 mL for this past hour. Which action should the nurse take? a. Position the client to lay on the surgical incision. b. Measure the specific gravity of the client's urine. c. Administer intravenous pain medications. d. Assess the rate and quality of the client's pulse.

ANS: D The nurse should first fully assess the client for signs of volume depletion and shock, and then notify the provider. The radical nature of the surgery and the proximity of the surgery to the adrenal gland put the client at risk for hemorrhage and adrenal insufficiency. Hypotension is a clinical manifestation associated with both hemorrhage and adrenal insufficiency. Hypotension is particularly dangerous for the remaining kidney, which must receive adequate perfusion to function effectively. Re-positioning the client, measuring specific gravity, and administering pain medication would not provide data necessary to make an appropriate clinical decision, nor are they appropriate interventions at this time.

A nurse cares for a client who is having trouble voiding. The client states, "I cannot urinate in public places." How should the nurse respond? a. "I will turn on the faucet in the bathroom to help stimulate your urination." b. "I can recommend a prescription for a diuretic to improve your urine output." c. "I'll move you to a room with a private bathroom to increase your comfort." d. "I will close the curtain to provide you with as much privacy as possible."

ANS: D The nurse should provide privacy to clients who may be uncomfortable or have issues related to elimination or the urogenital area. Turning on the faucet and administering a diuretic will not address the client's concern. Although moving the client to a private room with a private bathroom would be nice, this is not realistic. The nurse needs to provide as much privacy as possible within the client's current room.

A nurse assesses a client with a pelvic fracture. Which assessment finding should the nurse identify as a complication of this injury? a. Hypertension b. Constipation c. Infection d. Hematuria

ANS: D The pelvis is very vascular and close to major organs. Injury to the pelvis can cause integral damage that may manifest as blood in the urine (hematuria) or stool. The nurse should also assess for signs of hemorrhage and hypovolemic shock, which include hypotension and tachycardia. Constipation and infection are not complications of a pelvic fracture.

A nurse cares for a client experiencing diabetic ketoacidosis who presents with Kussmaul respirations. Which action should the nurse take? a. Administration of oxygen via face mask b. Intravenous administration of 10% glucose c. Implementation of seizure precautions d. Administration of intravenous insulin

ANS: D The rapid, deep respiratory efforts of Kussmaul respirations are the body's attempt to reduce the acids produced by using fat rather than glucose for fuel. Only the administration of insulin will reduce this type of respiration by assisting glucose to move into cells and to be used for fuel instead of fat. The client who is in ketoacidosis may not experience any respiratory impairment and therefore does not need additional oxygen. Giving the client glucose would be contraindicated. The client does not require seizure precautions.

A nurse cares for a client in skeletal traction. The nurse notes that the skin around the client's pin sites is swollen, red, and crusty with dried drainage. Which action should the nurse take next? a. Request a prescription to decrease the traction weight. b. Apply an antibiotic ointment and a clean dressing. c. Cleanse the area, scrubbing off the crusty areas. d. Obtain a prescription to culture the drainage.

ANS: D These clinical manifestations indicate inflammation and possible infection. Infected pin sites can lead to osteomyelitis and should be treated immediately. The nurse should obtain a culture and assess vital signs. The provider should be notified. By decreasing the traction weight, applying a new dressing, or cleansing the area, the infection cannot be significantly treated.

A nurse plans care for a client who is prescribed skeletal traction. Which intervention should the nurse include in this plan of care to decrease the client's risk for infection? a. Wash the traction lines and sockets once a day. b. Release traction tension for 30 minutes twice a day. c. Do not place the traction weights on the floor. d. Schedule for pin care to be provided every shift.

ANS: D To decrease the risk for infection in a client with skeletal traction of external fixation, the nurse should provide routine pin care and assess manifestations of infection at the pin sites every shift. The traction lines and sockets are external and do not come in contact with the client's skin; these do not need to be washed. Although traction weights should not be removed or released for any period of time without a prescription, or placed on the floor, this does not decrease the risk for infection.

A nurse delegates care of a client in traction to an unlicensed assistive personnel (UAP). Which statement should the nurse include when delegating hygiene care for this client? a. "Remove the traction when re-positioning the client." b. "Inspect the client's skin when performing a bed bath." c. "Provide pin care by using alcohol wipes to clean the sites." d. "Ensure that the weights remain freely hanging at all times."

ANS: D Traction weights should be freely hanging at all times. They should not be lifted manually or allowed to rest on the floor. The client should remain in traction during hygiene activities. The nurse should assess the client's skin and provide pin and wound care for a client who is in traction; this should not be delegated to the UAP.

A client has an ingrown toenail. About what self-management measure does the nurse teach the client? a. Long-term antibiotic use b. Shoe padding c. Toenail trimming d. Warm moist soaks

ANS: D Treatment of an ingrown toenail includes a podiatrist clipping away the ingrown part of the nail, warm moist soaks, and antibiotic ointment if needed. Antibiotics are not used long-term. Padding the shoes will not treat or prevent ingrown toenails. Clients should not attempt to trim ingrown nails themselves.

A nurse cares for a client who has type 1 diabetes mellitus. The client asks, "Is it okay for me to have an occasional glass of wine?" How should the nurse respond? a. "Drinking any wine or alcohol will increase your insulin requirements." b. "Because of poor kidney function, people with diabetes should avoid alcohol." c. "You should not drink alcohol because it will make you hungry and overeat." d. "One glass of wine is okay with a meal and is counted as two fat exchanges."

ANS: D Under normal circumstances, blood glucose levels will not be affected by moderate use of alcohol when diabetes is well controlled. Because alcohol can induce hypoglycemia, it should be ingested with or shortly after a meal. One alcoholic beverage is substituted for two fat exchanges when caloric intake is calculated. Kidney function is not impacted by alcohol intake. Alcohol is not associated with increased hunger or overeating.

A nursing student wants to know why clients with chronic obstructive pulmonary disease tend to be polycythemic. What response by the nurse instructor is best? a. It is due to side effects of medications for bronchodilation. b. It is from overactive bone marrow in response to chronic disease. c. It combats the anemia caused by an increased metabolic rate. d. It compensates for tissue hypoxia caused by lung disease.

ANS: D In response to hypoxia, more red blood cells are made so more oxygen can be carried and delivered to tissues. This is a physiologic process in response to the disease; it is not a medication side effect, the result of overactive bone marrow, or a response to anemia.

A nurse cares for a client who is experiencing deteriorating neurologic functions. The client states, I am worried I will not be able to care for my young children. How should the nurse respond? a. Caring for your children is a priority. You may not want to ask for help, but you have to. b. Our community has resources that may help you with some household tasks so you have energy to care for your children. c. You seem distressed. Would you like to talk to a psychologist about adjusting to your changing status? d. Give me more information about what worries you, so we can see if we can do something to make adjustments.

ANS: D Investigate specific concerns about situational or role changes before providing additional information. The nurse should not tell the client what is or is not a priority for him or her. Although community resources may be available, they may not be appropriate for the client. Consulting a psychologist would not be appropriate without obtaining further information from the client related to current concerns.

A nurse obtains a focused health history for a client who is scheduled for magnetic resonance imaging (MRI). Which condition should alert the nurse to contact the provider and cancel the procedure? a. Creatine phosphokinase (CPK) of 100 IU/L b. Atrioventricular graft c. Blood urea nitrogen (BUN) of 50 mg/dL d. Internal insulin pump

ANS: D Metal devices such as internal pumps, pacemakers, and prostheses interfere with the accuracy of the image and can become displaced by the magnetic force generated by an MRI procedure. An atrioventricular graft does not contain any metal. CPK and BUN levels have no impact on an MRI procedure.

A client is having a radioisotopic imaging scan. What action by the nurse is most important? a. Assess the client for shellfish allergies. b. Place the client on radiation precautions. c. Sedate the client before the scan. d. Teach the client about the procedure.

ANS: D The nurse should ensure that teaching is done and the client understands the procedure. Contrast dye is not used, so shellfish/iodine allergies are not related. The client will not be radioactive and does not need radiation precautions. Sedation is not used in this procedure.

A hospitalized client has a platelet count of 58,000/mm3. What action by the nurse is best? a. Encourage high-protein foods. b. Institute neutropenic precautions. c. Limit visitors to healthy adults. d. Place the client on safety precautions.

ANS: D With a platelet count between 40,000 and 80,000/mm3, clients are at risk of prolonged bleeding even after minor trauma. The nurse should place the client on safety precautions. High-protein foods, while healthy, are not the priority. Neutropenic precautions are not needed as the clients white blood cell count is not low. Limiting visitors would also be more likely related to a low white blood cell count.

When caring for a client with nephrotic syndrome, which intervention should be included in the plan of care? Administering angiotensin-converting enzyme (ACE) inhibitors to decrease protein loss Administering heparin to prevent deep vein thrombosis (DVT) Providing antibiotics to decrease infection Providing transfusion of clotting factors

Administering angiotensin-converting enzyme (ACE) inhibitors to decrease protein lossACE inhibitors can decrease protein loss in the urine. Heparin is administered for DVT, but in nephrotic syndrome it may reduce urine protein and kidney insufficiency. Glomerulonephritis may occur secondary to an infection, but it is an inflammatory process; antibiotics are not indicated for nephrotic syndrome. Clotting factors are not indicated unless bleeding and coagulopathy are present.

Which client will the neurologic unit charge nurse assign to a registered nurse who has floated from the labor/delivery unit for the shift? Adult client who has just returned from having a cerebral arteriogram and needs vital sign checks every 15 minutes. Older adult client who was just admitted with a stroke and needs an admission assessment. Young adult client who has had a lumbar puncture and reports, "Light hurts my eyes." Middle-aged client who has a possible brain tumor and has questions about the scheduled magnetic resonance imaging.

Adult client who has just returned from having a cerebral arteriogram and needs vital sign checks every 15 minutes. The charge nurse would assign an RN with experience in labor and delivery to check vital signs and limbs on a client who just returned from a cerebral angiogram. This float nurse would also be able to recognize signs of bleeding.The older adult admitted with a stroke, the young adult post lumbar puncture, and the middle-aged client with a possible brain tumor all require a nurse with more experience with neurologic diagnoses and diagnostic procedures.

A client with asthma has pneumonia, is reporting increased shortness of breath, and has inspiratory and expiratory wheezes. All of these medications are prescribed. Which medication should the nurse administer first?

Albuterol (Proventil) 2 inhalations Albuterol is a beta2 agonist that acts rapidly as a bronchodilator. Fluticasone is a corticosteroid; it is used to prevent asthma attacks and is not used as a rescue medication. Ipratropium is an anticholinergic drug that allows the sympathetic system to dominate and cause bronchodilation; it is not as effective as a beta2 agonist, so it is not a first-line drug. Salmeterol is a long-acting beta2 agonist that must be used regularly over time; this client needs a rescue medication.

The community health nurse is planning tuberculosis treatment for a client who is homeless and heroin-addicted. Which action will be most effective in ensuring that the client completes treatment?

Arrange for a health care worker to watch the client take the medication. Because this client is unlikely to adhere to long-term treatment unless directly supervised while taking medications, the best option is to arrange for directly observed therapy. Giving a client who is homeless and addicted to heroin written instructions on how to take prescribed medications is placing too much responsibility on the client to follow through. Also, the question does not indicate whether the client can read. The fact that the client can state the names and side effects of medications does not mean that the client understands what the medications are and why he or she needs to take them. A client who is homeless may be more concerned with obtaining shelter and food than with properly taking his or her medication.

Which task does the nurse plan to delegate to the unlicensed assistive personnel (UAP) caring for a group of clients in the neurosurgical unit? Assist the health care provider in performing a lumbar puncture on a confused client Attend to the care needs of a client who has had a transcranial Doppler study Educate a client about what to expect during an electroencephalogram (EEG) Prepare a client who is going to radiology for a cerebral arteriogram

Attend to the care needs of a client who has had a transcranial Doppler study The nurse delegates the UAP to care for the client who has had a transcranial Doppler study. Since transcranial Doppler studies are noninvasive and do not require any postprocedure monitoring or care the UAP can safely attend to this client.Assisting the primary care provider in performing a lumbar puncture and preparing a client for a cerebral arteriogram require assessments and interventions that would be done by licensed nursing staff. Client teaching would also be provided by licensed nursing staff.

Because clients with cystic fibrosis (CF) are at increased risk for infection, what will the nurse advise the client with CF who is infected with Burkholderia cepacia to do?

Avoid Cystic Fibrosis Foundation-sponsored events. A serious bacterial infection for clients with CF is Burkholderia cepacia, which is spread by casual contact from one CF client to another. For this reason, the Cystic Fibrosis Foundation bans infected clients (those who have had a positive sputum culture) from participating in any foundation-sponsored events. Avoiding the hospital completely is unrealistic, although special infection control procedures may be implemented, such as scheduling the client's office visits on different days or in different areas of the hospital. Social isolation is not needed for clients with CF and may be detrimental to the psychosocial well-being of the client. Hand hygiene is important, although this is not the best response.

14. A patient who is allergic to dogs experiences a sudden "asthma attack." Which assessment findings does the nurse expect for this patient? A. Slow, deep, pursed-lip respirations B. Breathlessness and difficulty completing sentences C. Clubbing of the fingers and cyanosis of the nailbeds d. Bradycardia and irregular pulse

B

20. The nurse is taking a history of a patient with chronic pulmonary disease. The patient reports often sleeping in a chair that allows his head to be elevated rather than sleeping in a chair that allows his head to be elevated rather than sleeping in a bed. The patient's behavior is a strategy to deal with which condition? A. Paroxysmal nocturnal dyspnea B. Orthopnea C. Tachypnea D. Cheyne-Stokes respirations

B

24. Which statement is true about the relationship of smoking cessation to the pathophysiology of COPD? A. Smoking cessation completely reverses the damage to the lungs. B. Smoking cessation slows the rate of disease progression. C. Smoking cessation is an important therapy for asthma but not for COPD. D. Smoking cessation reverses the effects on the airways but not the lungs.

B

25. A patient has a history of COPD but is admitted for a surgical procedure that is unrelated to the respiratory system. To prevent any complications related to the patient's COPD, what action does the nurse take? A. Assess the patient's respiratory system every 8 hours. B. Monitor for signs and symptoms of pneumonia. C. Give high-flow oxygen to maintain pulse oximetry readings. D. Instruct the patient to use a tissue if coughing or sneezing.

B

28. A patient with respiratory difficulty has completed a pulmonary function test before starting any treatment. The peak expiratory flow (PEF) is 15% to 20% below what is expected for this adult patient's age, gender, and size. The nurse anticipates this patient will need additional information about which topic? A. Further diagnostic tests to confirm pulmonary hypertension B. How to manage asthma medications and identify triggers C. Smoking cessation and its relationship to COPD D. How to manage the acute episode of respiratory infection

B

35. A patient is receiving ipratropium (Atrovent) and reports nausea, blurred vision, headache, and inability to sleep. What action does the nurse take? A. Administer a PRN medication for nausea and a milk PRN sedative. B. Report these symptoms to the provider as signs of overdose. C. Obtain a provider's request for an ipratropium level. D. Tell the patient that these side effects are normal and not to worry.

B

38. The nurse is teaching a patient with chronic airflow limitation about his medications. What is the correct sequence for administering aerosol treatment? A. Bronchodilator should be taken 5 to 10 minutes after the steroid. B. Bronchodilator should be taken at least 5 minutes before other inhaled drugs. C. Bronchodilator should be taken immediately after the steroid. D. Bronchodilator and steroid are two different classes of drugs, so sequence is irrelevant.

B

43. The nurse is caring for a patient with chronic bronchitis, and notes the following clinical findings: fatigue, dependent edema, distended neck veins, and cyanotic lips. What condition is the patient exhibiting? A. COPD B. Cor pulmonale C. Asthma D. Lung cancer

B

5. The nurse is presenting a community education lecture about respiratory disorders. Which statement by a participant indicates a correct understanding of the information? A. "Bronchitis is a genetic disease that affects many organs." B. "In bronchial asthma, an airway obstruction can be caused by inflammation." C. "In chronic bronchitis, the tissue damage is only temporary and reversible." D. "Smoking cessation reverses the tissue damage caused by emphysema."

B

51. The nurse is working for a manufacturing company and is responsible for routine employee health issues. Which primary prevention is most important for those employees at high risk for occupational pulmonary disease? A. Screen all employees by use of chest x-ray films twice a year. B. Advise employees not to smoke, and to use masks and ventilation equipment. C. Perform pulmonary function tests once a year on all employees. D. Refer at-risk employees to a social worker for information about pensions.

B

63. The nurse has determined that a patient with COPD has the priority problem of impaired oxygenation related to reduced airway size, ventilator muscle fatigue, and excessive mucus production. Which action is best to delegate to the unlicensed assistive personnel (UAP)? A. Observe the patient for fatigue, shortness of breath, or change of breathing pattern during activities of daily living (ADLs). B. Report a respiratory rate of greater than 24/min at rest or 30/min after ambulating to the nurses' station. C. Encourage the patient to cough up sputum, and examine the color, consistency, and amount. D. Record and monitor the patient's intake and output, and give fluids to keep the secretions thin.

B

68. A patient is diagnosed with cor pulmonale secondary to pulmonary hypertension and is receiving an infusion of epoprostenol (Flolan) through a small portable IV pump. What is the critical priority for this patient? A. Strict aseptic technique must be used to prevent sepsis. B. Infusion must not be interrupted, even for a few minutes. C. The patient must have a daily dose of warfarin (Coumadin). D. The patient must be assessed for angina like chest pain and fatigue.

B

9. The nurse teaches a patient with asthma to monitor for which problem while exercising? A. Increased peak expiratory flow rates B. Wheezing from bronchospasm C. Swelling in the feet and ankles D. Respiratory muscle fatigue

B

A client is being discharged home with active tuberculosis. Which information does the nurse include in the discharge teaching plan? A) "You are not contagious unless you stop taking your medication." B) "You will not be contagious to the people you have been living with." C) "You will have to take these medications for at least 1 year." D) "Your sputum may turn a rust color as your condition gets better." (Chp. 31; elsevier resources)

B) "You will not be contagious to the people you have been living with." (Chp. 31; elsevier resources)

Which of these clients should the charge nurse assign to the LPN/LVN working on the medical-surgical unit? A) Client with group A beta-hemolytic streptococcal pharyngitis who has stridor B) Client with pulmonary tuberculosis who is receiving multiple medications C) Client with sinusitis who has just arrived after having endoscopic sinus surgery D) Client with tonsillitis who has a thick-sounding voice and difficulty swallowing (Chp. 31; elsevier resources)

B) Client with pulmonary tuberculosis who is receiving multiple medications (Chp. 31; elsevier resources)

A client with pneumonia caused by aspiration after alcohol intoxication has just been admitted. The client is febrile and agitated. Which health care provider order should the nurse implement first? A) Administer levofloxacin (Levaquin) 500 mg IV. B) Draw aerobic and anaerobic blood cultures. C) Give lorazepam (Ativan) as needed for agitation. D) Refer to social worker for alcohol counseling. (Chp. 31; elsevier resources)

B) Draw aerobic and anaerobic blood cultures. (Chp. 31; elsevier resources)

Which method is the best way to prevent outbreaks of pandemic influenza? A) Avoiding public gatherings at all times B) Early recognition and quarantine C) Vaccinating everyone with pneumonia vaccine D) Widespread distribution of antiviral drugs (Chp. 31; elsevier resources)

B) Early recognition and quarantine (Chp. 31; elsevier resources)

The charge nurse at an assisted-living facility receives report from an emergency department (ED) nurse about one of the resident clients. The client was sent to the ED with a fever, chills, muscle aches, and headache. The ED nurse reports the client's rapid influenza report came back from the laboratory positive for influenza A. What action by the nurse at the assisted-living facility is most appropriate? A) Prepare to administer antibiotics. B) Have the resident eat meals in his room. C) Provide oseltamivir (Tamiflu) to the staff. D) Arrange a follow-up chest x-ray in 2 weeks. (Chp. 31; p. 587)

B) Have the resident eat meals in his room. (Chp. 31; p. 587)

55. Which sites are commonly affected by lung cancer metastasis? (Select all that apply.) A. Heart B. Bone C. Liver D. Colon E. Brain

B, C, E

The nurse is teaching the client newly diagnosed with migraine about trigger control. Which statement made by the client demonstrates good understanding of the teaching plan? A. "I can still eat Chinese food." B. "I must not miss meals." C. "It is okay to drink a few wine coolers." D. "I need to use fake sugar in my coffee."

B. "I must not miss meals." Rationale A. Monosodium glutamate (MSG)-containing foods are a trigger for many people suffering from migraines and should be eliminated until the triggers are identified. B. Missing meals is a trigger for many people suffering from migraines. The client should not skip any meals until the triggers are identified. C. Alcohol is a trigger for many people suffering from migraines and should be eliminated until the triggers are identified. D. Artificial sweeteners are a trigger for many people suffering from migraines and should be eliminated until the triggers are identified.

The female client with newly diagnosed migraine is being discharged with a prescription for sumatriptan (Imitrex). Which comment by the client indicates an understanding of the nurse's discharge instructions? A. "Sumatriptan should be taken as a last resort." B. "I must report any chest pain right away." C. "Birth control is not needed while taking sumatriptan." D. "St. John's wort can also be taken to help my symptoms."

B. "I must report any chest pain right away." Rationale A. Sumatriptan must be taken as soon as migraine symptoms appear. B. Chest pain must be reported immediately with the use of sumatriptan. C. Remind the client to use contraception (birth control) while taking the drug because it may not be safe for women who are pregnant. D. Triptans should not be taken with selective serotonin reuptake inhibitors (SSRIs) or St. John's wort, an herb used commonly for depression.

Which of these assessment findings will be of greatest concern when the nurse is assessing a client with emphysema? A. Barrel-shaped chest B. Bronchial breath sounds heard at the bases C. Hyperresonance to percussion of the chest D. Ribs lying horizontal

B. Bronchial breath sounds heard at the bases Correct: Bronchial breath sounds are not normally heard in the periphery and may indicate increased lung density, as in a tumor or an infective process such as pneumonia.

The nurse is performing a client assessment for the client's potential employer. The client reports dyspnea when climbing stairs but is not dyspneic at rest. Which dyspnea classification does the nurse assign to this client in the report to the employer? A. Class I, can perform perform manual labor B. Class II, can perform desk job C. Class III, minimally employable D. Class IV, must remain at home

B. Class II, can perform desk job Correct: This client is dyspneic when climbing stairs or walking on an incline but not on level walking. Therefore, this client is employable only for a sedentary job or under special circumstances.

In assessing the client's respiratory status, blood gas test results reveal pH of 7.50, PaO2 of 99, PaCO2 of 29, and HCO of 22. What action does the nurse need to take first? A. Call the physician. B. Encourage the client to slow his breathing rate. C. Nothing. These results are within the normal range. D. Provide oxygen support.

B. Encourage the client to slow his breathing rate. Correct: The arterial blood gases (ABGs) indicate respiratory alkalosis, which is commonly caused by hyperventilation. Encouraging the client to slow down his breathing rate may help him return to normal breathing and may correct this abnormality.

Which is the most effective way for the college student to minimize the risk for bacterial meningitis? A. Avoiding large crowds B. Getting the meningitis polysaccharide vaccine C. Taking a daily vitamin D. Taking prophylactic antibiotics

B. Getting the meningitis polysaccharide vaccine Rationale A. Avoiding large crowds is helpful but is not practical for the college student. B. People who live in highly populated areas, such as a college dorm, should get the meningitis polysaccharide vaccine (Menomune) to prevent infection. C. Taking a daily vitamin is helpful but is not the best way to safeguard against bacterial meningitis. D. Taking prophylactic antibiotics is inappropriate because it leads to antibiotic-resistant strains of microorganisms.

The nurse is caring for a client diagnosed with partial seizures after encephalitis, who is to receive carbamazepine (Tegretol). The nurse plans to monitor the client for which adverse effects? Select all that apply. A. Alopecia B. Headaches C. Dizziness D. Diplopia E. Increased blood glucose

B. Headaches C. Dizziness D. Diplopia Rationale Carbamazepine does not cause alopecia. Divalproex (Depakote) and valproic acid (Depakene) may cause alopecia.

The client returns to the medical unit after a therapeutic bronchoscopy. Which intervention does the nurse apply first? A. Assess the puncture site for drainage. B. Implement NPO (nothing by mouth) status. C. Monitor for signs of anaphylaxis. D. Perform aggressive chest physiotherapy.

B. Implement NPO (nothing by mouth) status. Correct: Until the client has a gag reflex and is fully alert, he should be maintained on NPO status to prevent aspiration.

A client with a history of seizures is placed on seizure precautions. What emergency equipment will the nurse provide at the bedside? Select all that apply. A. Padded tongue blade B. Oxygen setup C. Nasogastric tube D. Suction setup E. Artificial oral airway

B. Oxygen setup D. Suction setup E. Artificial oral airway Rationale Clients placed on seizure precautions should have an oxygen setup, suction equipment, and an artificial oral airway at the bedside. Maintaining a patent airway is the priority for this client's care. Intubation by an anesthesia provider or respiratory therapist may be necessary. Oxygen may need to be administered as indicated by the client's condition. Padded tongue blades do not belong at the bedside and should NEVER be inserted into the client's mouth because the jaw may clench down as soon as the seizure begins. Forcing a tongue blade or airway into the mouth is more likely to chip the teeth and increase the risk of aspirating tooth fragments than prevent the client from biting the tongue. Furthermore, improper placement of a padded tongue blade can obstruct the airway. There is no physiologic reason to place a nasogastric tube emergently for a client experiencing seizure activity. In fact, convulsions may make it difficult to place the tube and put the client at risk.

Why are the turbinates important? A. They decrease the weight of the skull on the neck. B. They increase the surface area of the nose for heating and filtering. C. They move inspired particles from nose to throat for removal. D. They separate two nasal passages down the middle.

B. They increase the surface area of the nose for heating and filtering. Correct: The turbinates increase the surface area of the nose, so that more heating, filtering, and humidifying of inspired air can occur before air passes into the nasopharynx.

The nurse has received report on a group of clients. Which client requires the nurse's attention first? A. Adult who is lethargic after a generalized tonic-clonic seizure B. Young adult who has experienced four tonic-clonic seizures within the past 30 minutes C. Middle-aged adult with absence seizures who is staring at a wall and does not respond to questions D. Older adult with a seizure disorder who has a temperature of 101.9° F (38.8° C)

B. Young adult who has experienced four tonic-clonic seizures within the past 30 minutes Rationale A. This not a medical emergency and does not require immediate attention. B. This client is experiencing status epilepticus, which is a medical emergency and requires immediate intervention. C. This is not a medical emergency and does not require immediate attention. D. A fever of 101.9° F (38.8° C) is not a medical emergency and does not require immediate attention.

Which assessment finding is associated with obstructive lung disease and not with interstitial lung disease?

Barrel chest Interstitial lung diseases are restrictive, not obstructive, so they do not cause barrel chest, which is the result of air trapping. Both types of pulmonary disease cause cough, dyspnea, and reduced gas exchange.

A client has just returned from cerebral angiography. Which symptom does the client display that causes the nurse to act immediately? Bleeding Increased temperature Severe headache Urge to void

Bleeding After a cerebral angiography, the nurse would immediately react if the client had any bleeding. If bleeding is present at the puncture site, manual pressure on the site is maintained along with immediate notification of the primary care provider.Increased temperature or the urge to void are not typical complications of cerebral angiography. Severe headache is a typical complication of a lumbar puncture, but not of cerebral angiography.

The nurse is caring for a client who has just returned to the surgical unit after a radical nephrectomy. Which assessment information alarms the nurse? Blood pressure is 98/56 mm Hg; heart rate is 118 beats/min. Urine output over the past hour was 80 mL. Pain is at a level 4 (on a 0-to-10 scale). Dressing has a 1-cm area of bleeding.

Bleeding is a complication of radical nephrectomy; tachycardia and hypotension may indicate impending hypovolemic or hemorrhagic shock. The surgeon should be notified immediately and fluids should be administered, complete blood count should be checked, and blood administered, if necessary. A urine output of 80 mL can be considered normal. The nurse can administer pain medication, but must address hemodynamic instability and possible hemorrhage first. Administering pain medication to a client who has developed shock will exacerbate hypotension. A dressing with a 1-cm area of bleeding is expected postoperatively.

Which assessment finding in the client with exacerbation of emphysema requires intervention by the nurse? Barrel-shaped chest Bronchial breath sounds heard at the bases Hyperresonance to percussion of the chest Ribs lying horizontal

Bronchial breath sounds heard at the bases Bronchial breath sounds are not normally heard in the periphery and may indicate increased lung density, as in a tumor or an infective process such as pneumonia. The anteroposterior diameter is the same as the lateral-to-lateral or side-to-side diameter in a client with emphysema, so he will generally have a barrel-shaped chest. Air-filled cavities, such as the lung, are hyperresonant to percussion. Air trapping causes the lungs in a client with emphysema to lie in a horizontal direction.

Which component of a client's family history is of particular importance to the home health nurse who is assessing a new client with asthma? Diabetes Obesity Brother allergic to peanuts Pregnant

Brother is allergic to peanuts. Clients with asthma often have a family history of allergies; it will be important to assess whether this client has any allergies that may serve as triggers for an asthma attack. Obesity, diabetes, and pregnancy are not correlated with asthma.

The nurse is preparing to administer oxygen to a client with chronic obstructive pulmonary disease (COPD) who is hypoxemic and hypercarbic. How will the nurse administer the oxygen for this client?

By maintaining oxygen saturations greater than 88% In the past, a client with COPD was thought to be at risk for extreme hypoventilation with oxygen therapy because of a decreased drive to breathe as blood oxygen levels increased. However, recent evidence does not support this; this idea has been responsible for ineffective management of hypoxia in clients with COPD. All hypoxic clients, even those with COPD and hypercarbia, should receive oxygen therapy at rates appropriate to reduce hypoxia and bring SpO2 levels up between 88% and 92%

18. The nurse is teaching a patient how to interpret peak expiratory flow readings and use this information to manage drug therapy at home. Which statement by the patient indicates a need for additional teaching? A. "If the reading is in the green zone, there is no need to increase the drug therapy." B. "Red is 50% below my 'personal best'; I should try a rescue drug and seek help." C. "If the reading is in the yellow zone, I should increase my use of my inhalers." D. "If frequent yellow readings occur, I should see my provider for a change in medications."

C

19. A patient with chronic bronchitis often shows signs of hypoxia. Which clinical manifestation is the priority to look out for in this patient? A. Chronic, nonproductive, dry cough B. Clubbing of fingers C. Large amounts of thick mucus D. Barrel chest

C

22. A patient has COPD with chronic difficult breathing. In planning this patient's care, what condition must the nurse acknowledge is present in this patient? A. Decreased need for calories and protein requirements since dyspnea causes activity intolerance B. COPD has no effect on calorie and protein needs, meal tolerance, satiety, appetite, and weight C. Increased metabolism and the need for additional calories and protein supplements D. Anabolic state, which creates conditions for building body strength and muscle mass

C

26. The nurse is instructing a patient regarding complications of COPD. Which statement by the patient indicates the need for additional teaching? A. "I have to be careful because I am susceptible to respiratory infections." B. "I could develop heart failure, which could be fatal if untreated." C. "My COPD is serious, but it can be reversed if I follow my doctor's orders." D. "The lack of oxygen could cause my heart to beat in an irregular pattern."

C

32. The nurse is developing a teaching plan for a patient with chronic airflow limitation using the priority patient problem of insufficient knowledge related to energy conservation. What does the nurse advise the patient to avoid? A. Performing activities at a relaxed pace throughout the day with rest periods B. Working on activities that require using arms at chest level or lower C. Eating three large meals per day D. Talking and performing activities separately

C

36. A patient with asthma has been prescribed a fluticasone (Flovent) inhaler. What is the purpose of this drug for the patient? A. Relaxes the smooth muscles of the airway B. Acts as a bronchodilator in severe episodes C. Reduces obstruction of airways by decreasing inflammation D. Reduces the histamine effect of the triggering agent

C

39. A patient has been prescribed cromolyn sodium (Intal) for the treatment of asthma. Which statement by the patient indicates a correct understanding of this drug? A. "It opens my airways and provides short-term relief." B. "It is the medication that should be used 30 minutes before exercise." C. "It is not intended for use during acute episodes of asthma attacks." D. "It is a steroid medication, so there are severe side effects."

C

4. The nurse is caring for an older adult patient with a history of chronic asthma. Which problem related to aging can influence the care and treatment of this patient? A. Asthma usually resolves with age, so the condition is less severe in older adult patients. B. It is more difficult to teach older adult patients about asthma than to teach younger patients. C. With aging, the beta-adrenergic drugs do not work as quickly or strongly. D. Older adult patients have difficulty manipulating handheld inhalers.

C

40. After the nurse has instructed a patient with COPD in the proper coughing technique, which action the next day by the patient indicates the need for additional teaching or intervention? A. Coughing upon rising in the morning B. Coughing before meals C. Coughing after meals D. Coughing at bedtime

C

41. A family member of a patient with COPD asks the nurse, "What is the purpose of making him cough on a routine basis?" What is the nurse's best response? A. "We have to check the color and consistency of his sputum." B. "We don't want him to feel embarrassed when coughing in public, so we actively encourage it." C. "It improves air exchange by increasing airflow in the larger airways." D. "If he cannot cough, the provider may elect to do a tracheostomy."

C

44. A patient is admitted with asthma. Which assessment findings are most likely to indicate that the patient's asthma condition is deteriorating and progressing toward respiratory failure? A. Crackles, rhonchi, and productive cough with yellow sputum B. Tachypnea; thick, tenacious sputum; and hemoptysis C. Audible breath sounds, wheezing, and use of accessory muscles D. Respiratory alkalosis; slow, shallow respiratory rate

C

46. The nurse assesses a patient and finds a dusky appearance with bluish mucous membranes and production of lots of mucus. What illness does the nurse suspect? A. Asthma B. Emphysema C. Chronic bronchitis D. Acute bronchitis

C

52. The nurse is caring for a patient with bronchiolitis obliterans organizing pneumonia (BOOP) that has been confirmed by biopsy. What treatment does the nurse expect for this patient? A. A course of 10 to 14 days of antibiotics B. Use of chest physiotherapy to mobilize secretions C. A short course of corticosteroid drug therapy D. Bronchodilation by MDI

C

53. A patient has prolonged occupational exposure to petroleum distillates and subsequently developed a chronic lung disease. This patient is advised to seek frequent health examinations because there is high risk for developing which respiratory disease condition? A. Tuberculosis B. Cystic fibrosis C. Lung cancer D. Pulmonary hypertension

C

57. Which statement is true about radiation therapy for lung cancer patients? A. It is given daily in "cycles" over the course of several months. B. It causes hair loss, nausea, and vomiting for the duration of treatment. C. It causes dry skin at the radiation site, fatigue, and changes in appetite with nausea. D. It is the best method of treatment for systemic metastatic disease.

C

61. The provider's prescriptions indicate an increase in the suction to -20cm for a patient with a chest tube. To implement this, the nurse performs which intervention? A. Increases the wall suction to the medium setting, and observes gentle bubbling in the suction chamber. B. Adds water to the suction and drainage chambers to the level of -20cm C. Stops the suction, adds sterile water to level of -20cm to the water seal chamber, and resumes the wall suction D. has the patient cough and deep-breathe, and monitors level of fluctuation to achieve -20cm

C

65. A patient is having pain resulting from bone metastases caused by lung cancer. What is the most effective intervention for relieving the patient's pain? A. Support the patient through chemotherapy. B. Handle and move the patient very gently. C. Administer analgesics around the clock. D. Reposition the patient, and use distraction.

C

66. A patient has a chest tube in place. What does the water in the water seal chamber do when the system is functioning correctly? A. Bubbles vigorously and continuously B. Bubbles gently and continuously C. Fluctuates with the patient's respirations D. Stops fluctuation, and bubbling is not observed

C

73. A patient presents to the walk-in clinic with extremely labored breathing and a history of asthma that is unresponsive to prescribed inhalers or medications. What is the priority nursing action? A. Establish IV access to give emergency medications. B. Obtain the equipment and prepare the patient for intubation. C. Place the patient in a high Fowler's position, and start oxygen. D. Call 911 and report that the patient has probable status asthmaticus.

C

The nurse is preparing a client for discharge who has undergone percutaneous needle aspiration of a peritonsillar abscess. Which is most important to teach the client about follow-up care? A) Completing the antibiotic medication regimen B) Taking pain medications every 4 to 6 hours C) Contacting the provider if the throat feels more swollen D) Using warm saline gargles and irrigations (Chp. 31; elsevier resources)

C) Contacting the provider if the throat feels more swollen (Chp. 31; elsevier resources)

A client who has recently traveled to Vietnam comes to the emergency department with fatigue, lethargy, night sweats, and a low-grade fever. What is the nurse's first action? A) Contact the health care provider for tuberculosis (TB) medications. B) Perform a TB skin test. C) Place a respiratory mask on the client. D) Test all family members for TB. (Chp. 31; elsevier resources)

C) Place a respiratory mask on the client. (Chp. 31; elsevier resources)

A client is admitted to the emergency department (ED) with a possible diagnosis of avian influenza ("bird flu"). Which of these actions included in the hospital protocol for avian influenza will the nurse take first? A) Ensure that ED staff members receive oseltamivir (Tamiflu). B) Obtain specimens for the H5 polymerase chain reaction test. C) Place the client in a negative air pressure room. D) Start an IV line and administer rehydration therapy. (Chp. 31; elsevier resources)

C) Place the client in a negative air pressure room. (Chp. 31; elsevier resources)

The nursing instructor asks the student nurse caring for a client with Alzheimer's disease who has been prescribed donepezil (Aricept) how the drug works. Which response by the nursing student best explains the action of donepezil? A. "The reuptake of serotonin is blocked." B. "Donepezil prevents the increase in the protein beta amyloid." C. "It delays the destruction of acetylcholine by acetylcholinesterase." D. "Dopamine levels are increased."

C. "It delays the destruction of acetylcholine by acetylcholinesterase." Rationale A. Donepezil is not a serotonin reuptake inhibitor. B. Donepezil is a cholinesterase inhibitor and does not work on the protein beta amyloid. C. By delaying the destruction of acetylcholine, donepezil improves cholinergic neurotransmission in the central nervous system (CNS), thus delaying the onset of cognitive decline. D. Donepezil does not work on dopamine receptors.

The home health nurse is checking in on the client with dementia and the client's spouse. The spouse confides to the nurse, "I am so tired and worn out." What is the nurse's best response? A. "Can't you take care of your spouse?" B. "Establishing goals and a daily plan can help." C. "Make sure you take some time off and take care of yourself too." D. "That's not a very nice thing to say."

C. "Make sure you take some time off and take care of yourself too." Rationale A. This response is not supportive and may offend the spouse. B. A better response would be, "Take one day at a time." C. This response is supportive and reminds the spouse that he or she cannot care for the client when exhausted. Of course, further assessment and planning will be necessary. D. This response is judgmental and inappropriate.

A client with Alzheimer's disease asks the nurse to find her mother, who is deceased. What is the nurse's best response? A. "Your mother died over 20 years ago." B. "I'll find your mother as soon as I finish passing meds." C. "What did your mother look like?" D. "I'll ask your daughter to find your mother."

C. "What did your mother look like?" Rationale A. For the client in the later stages of AD, reality orientation does not work and often increases agitation. Reference: p. 951, Psychosocial Integrity B. Telling the client to wait until medications have been issued or that her daughter will find her mother is not consistent with validation therapy because it reinforces the client's belief that her mother is still alive. Reference: p. 951, Psychosocial Integrity C. The nurse should use validation therapy for the client with moderate or severe Alzheimer's disease (AD). In validation therapy, the staff member recognizes and acknowledges the client's feelings and concerns. This response is not argumentative but also does not reinforce the client's belief that her mother is still living. Reference: p. 951, Psychosocial Integrity D. Telling the client to wait until medications have been issued or that her daughter will find her mother is not consistent with validation therapy because it reinforces the client's belief that her mother is still alive. Reference: p. 951, Psychosocial Integrity

You are a charge nurse on a surgical floor. The LPN/LVN informs you that a new client who had an earlier bronchoscopy has the following vital signs: heart rate 132, respiratory rate 26, and blood pressure 98/50. The client is anxious and his skin is cyanotic. What will be your first action? A. Call the Rapid Response Team. B. Give methylene blue 1% 1 to 2 mg/kg by IV injection C. Administer oxygen. D. Notify the physician immediately.

C. Administer oxygen. Correct: Administering oxygen and reassessing vital signs to observe for improvement is the first action. Administration of oxygen by itself may help relieve the client's anxiety.

The client admitted with cerebral edema suddenly begins to have a seizure while the nurse is in the room. What will the nurse do first? A. Administer phenytoin (Dilantin) B. Draw blood C. Assess the need for additional support D. Start an intravenous (IV) line

C. Assess the need for additional support Rationale A. Phenytoin (Dilantin) is administered to prevent the recurrence of seizures, not to treat a seizure already under way. B. Drawing blood is not the priority in this situation. C. Convulsive status epilepticus must be treated promptly and aggressively. After a quick assessment by the nurse, the health care provider must be notified immediately, and intubation by an anesthesiologist, nurse anesthetist, or respiratory therapist may be necessary. D. Starting an IV is not the priority in this situation.

Four clients arrive in the emergency department simultaneously with chest pain. The client with which type of chest pain requires immediate attention by the nurse? A. Client with pain on deep inspiration B. Client with pain on palpation C. Client with pain radiating to the shoulder D. Client with pain that is rubbing in nature

C. Client with pain radiating to the shoulder Correct: Chest pain radiating to the shoulder should be assumed to be cardiac in origin until proven otherwise; this requires the immediate attention of the nurse.

A client is admitted to the surgical floor with chest pain, shortness of breath, and hypoxemia after having a knee replacement. What diagnostic test will the nurse teach the client about to help confirm the diagnosis? A. Bronchoscopy B. Chest x-ray C. Computed tomography (CT) scan D. Thoracoscopy

C. Computed tomography (CT) scan Correct: CT scans, especially spiral or helical CT scans, with injected contrast can detect pulmonary emboli.

Which nursing intervention is the priority in preparing the client for pulmonary function testing (PFT)? A. Administer bronchodilator medication on call. B. Encourage clear fluid intake 12 hours before the procedure. C. Ensure no smoking 6 hours before the test. D. Provide supplemental oxygen as testing begins.

C. Ensure no smoking 6 hours before the test. Correct: If the client has been smoking, this may alter parts of the PFT (diffusing capacity [DlCO]), yielding inaccurate results.

The nurse is providing medication instructions for a client for whom phenytoin (Dilantin) has been requested for treatment of epilepsy. The nurse plans to instruct the client to avoid which beverage? A. Apple juice B. Grape juice C. Grapefruit juice D. Milk

C. Grapefruit juice Rationale A. Apple juice does not interact with phenytoin. B. Grape juice does not interact with phenytoin. C. Grapefruit juice can interfere with the metabolism of phenytoin. D. Milk does not interact with phenytoin.

A client comes to the emergency department with a productive cough. Which symptom does the nurse look for that will require immediate attention? A. Blood in the sputum B. Mucoid sputum C. Pink frothy sputum D. Yellow sputum

C. Pink frothy sputum Correct: Pink frothy sputum is common with pulmonary edema and requires immediate attention and intervention to prevent the client's condition from getting worse.

Which factor is an indicator for a diagnosis of hydronephrosis? History of nocturia History of urinary stones Recent weight loss Urinary incontinence

Causes of hydronephrosis or hydroureter include tumors, stones, trauma, structural defects, and fibrosis. Nocturia is a key feature of polycystic kidney disease and pyelonephritis, but it is not associated with hydronephrosis. Recent weight loss and urinary incontinence may be factors in renal cell carcinoma, but are not associated with hydronephrosis.

Which client does the charge nurse on the medical unit assign to an RN who has floated from the postanesthesia care unit (PACU)? Client with possible ulcer who just returned from an endoscopy Client with emphysema who needs teaching about pulmonary function testing Client with pancreatitis who needs a preoperative chest x-ray Client who had 1200 mL of pleural fluid removed by thoracentesis

Client who had 1200 mL of pleural fluid removed by thoracentesis A nurse working in the PACU would be most familiar with assessing vital signs and respiratory status for a postoperative client after an invasive procedure such as thoracentesis. When a large volume of fluid has been removed, there is a greater risk for instability. This client is within this nurse's skill set.Endoscopy is typically performed with sedation, not general anesthesia, which will not require the critical rescue skills of the PACU nurse. Pulmonary function testing is not a procedure the PACU nurse would typically encounter nor will it require the skill level of the PACU nurse. Although a client with pancreatitis is seriously ill and would require a chest x-ray before undergoing operative procedures, a nurse with a PACU monitoring skill set would not be required.

The nurse has just received report on a group of clients. Which client does the nurse assess first? Client who was in a car accident and has a Glasgow Coma Scale score of 14 Client who had a cerebral arteriogram and has a cool, pale leg Client who has a headache after undergoing a lumbar puncture Client who has expressive aphasia after a left-sided stroke

Client who had a cerebral arteriogram and has a cool, pale leg The nurse first assesses the client with a cool, pale leg after an arteriogram. This assessment finding could indicate clot formation at the catheter insertion site and loss of blood flow to the extremity.The client with a GCS of 14, the client with a headache following a lumbar puncture, and the client with expressive aphasia need to be assessed as soon as possible.

The nurse is working in an urgent care clinic where four clients are waiting to be seen. Which client needs to be evaluated first by the nurse? Client who is short of breath after walking up two flights of stairs Client with a 10 mm area of redness on the arm after receiving purified protein derivative (Mantoux) skin test Client with sore throat and fever of 102.2°F (39°C) oral Client who is speaking in three-word sentences and has an SpO2 of 90%

Client who is speaking in three-word sentences and has an SpO2 of 90% The client that requires first and immediate evaluation by the nurse is the client who is speaking in three-word sentences and displaying dyspnea. This, coupled with an SpO2 of 90%, indicates hypoxemia.The client displaying shortness of breath after walking up two flights of stairs may be displaying signs/symptoms of underlying cardiopulmonary disease. This is not an emergency as there is no indication of dyspnea at rest. Induration, not redness, reflects a positive Mantoux test with possible TB. This develops slowly and will not take priority over airway and breathing. Sore throat and fever are symptoms of infection that require further evaluation, but not emergently.

The nurse has just received report on a group of clients on the neurosurgical unit. Which client is the nurse's first priority? Client whose deep tendon reflexes have become hyperactive Client who displays plantar flexion when the bottom of the foot is stroked Client who consistently demonstrates decortication when stimulated Client whose Glasgow Coma Scale (GCS) has changed from 15 to 13.

Client whose Glasgow Coma Scale (GCS) has changed from 15 to 13. After receiving report on a group of clients, the nurse's first priority is to assess the client whose GCS has changed from 15 to 13. A decrease of 2 or more points in the Glasgow Coma Scale total is clinically significant and indicates a major change in neurologic status. This finding must be reported immediately to the primary health care provider (PHCP).The client with hyperactive reflexes, the client displaying plantar flexion when the bottom of the foot is stroked, and the client with decortication upon stimulation will need to be assessed, but they do not require immediate attention.

All of these clients are being cared for on the intensive care stepdown unit. Which client should the charge nurse assign to an RN who has floated from the pediatric unit?

Client with acute asthma episode who is receiving oxygen at FiO2 of 60% by non-rebreather mask. Because asthma is a common pediatric diagnosis, the pediatric nurse would be familiar with the assessment and care needed for a client with this diagnosis. Although chronic pleural effusions can occur in the pediatric population, this diagnosis is more common in the adult population. If this client has not already received teaching for this procedure, he or she may have questions that the pediatric nurse would not be as comfortable answering as a nurse who is regularly assigned to the stepdown unit. Emphysema is a diagnosis associated with an adult population; although an RN could instruct a client about home oxygen therapy, this client might have questions that would be better answered by an RN with adult experience. The adult client who has just had a lobectomy needs careful assessment from an RN with adult stepdown unit experience.

The RN has received report about all of these clients. Which client needs the most immediate assessment? Client with acute asthma who has an oxygen saturation of 89% by pulse oximetry Client admitted 3 hours ago for a scheduled thoracentesis in 30 minutes Client with bronchogenic lung cancer who returned from bronchoscopy 3 hours ago Client with pleural effusion who has decreased breath sounds at the right base

Client with acute asthma who has an oxygen saturation of 89% by pulse oximetry An oxygen saturation level less than 91% indicates hypoxemia and requires immediate assessment and intervention to improve blood and tissue oxygenation. The client who is scheduled for a thoracentesis will be able to receive teaching and will have the opportunity to ask questions and have them answered before the procedure is performed, if this was not already completed. The client who had a bronchoscopy 3 hours ago and has returned to the floor does not require the most immediate attention at this moment. It would not be unusual to have diminished breath sounds at the base of the lung of the client with pleural effusion.

The change-of-shift report has just been completed on the medical-surgical unit. Which client will the oncoming nurse plan to assess first?

Client with cystic fibrosis (CF) who has an elevated temperature and a respiratory rate of 38 breaths/min. The client with CF with an elevated temperature and respiratory rate is exhibiting signs of an exacerbation and needs to be assessed first. The nurse will need to speak with the client who has COPD to help find a plan that will enable the client to obtain his or her prescribed medications; this may involve contacting case management or social services and discussing the discharge with the discharge health care provider. An oxygen saturation of 89% may be normal for a hospice client with terminal pulmonary fibrosis; not enough information is provided to determine whether this client is in distress. The client who needs an IV antibiotic could have the medication administered by another RN, or it could be administered in the operating room.

The home health nurse is assigned to visit all of these clients when a change in agency staffing requires that one of the clients should be rescheduled for a visit on the following day. Which of these clients would be best to reschedule? A. Client with emphysema who has been on home oxygen for a month and has SPO2 levels of 91% to 93% B. Client with history of a cough, weight loss, and night sweats who has just had a positive Mantoux test C. Client with newly diagnosed pleural effusion who needs an admission visit and an initial intake assessment D. Client with percutaneous lung biopsy yesterday who called in to report increased dyspnea

Client with emphysema who has been on home oxygen for a month and has SPO2 levels of 91% to 93% Correct: This client has an appropriate Spo2 for home oxygen use.

The home health nurse is assigned to visit all of these clients when a change in agency staffing requires that one of the clients be rescheduled for a visit on the following day. Which client would be best to reschedule? Client with emphysema who has been on home oxygen for a month and has SpO2 levels of 91% to 93% Client with history of a cough, weight loss, and night sweats who has just had a positive Mantoux test Client with newly diagnosed pleural effusion who needs an admission visit and an initial intake assessment Client with percutaneous lung biopsy yesterday who called in to report increased dyspnea

Client with emphysema who has been on home oxygen for a month and has SpO2 levels of 91% to 93% The client with emphysema has an appropriate SpO2 for home oxygen use. A positive Mantoux test, in addition to a history of cough, weight loss, and night sweats, is highly suspicious for tuberculosis and needs follow-up and reporting, because this becomes a public and a personal health issue. A client with a newly diagnosed pleural effusion needs a complete and thorough admission and intake assessment to ensure that he has adequate respiratory function to meet his basic oxygenation needs. Although a percutaneous lung biopsy may be an outpatient procedure, pneumothorax or hemothorax is a possible life-threatening complication of this procedure that would cause dyspnea and requires assessment in a timely manner by the home health nurse.

Which of these clients should the charge nurse assign to the LPN/LVN working on the medical-surgical unit?

Client with pulmonary tuberculosis who is receiving multiple medications The LPN/LVN scope of practice includes medication administration, so a client receiving multiple medications can be managed appropriately by an LPN/LVN. Stridor is an indication of respiratory distress; this client needs to be managed by the RN. A client in the immediate postoperative period requires frequent assessments by the RN to watch for deterioration. A client with a thick-sounding voice and difficulty swallowing is at risk for deterioration and needs careful assessment and monitoring by the RN.

The nurse is caring for a client who just returned from an open lung biopsy and has a prescription for morphine by client controlled analgesia (PCA). Which of these actions to detect early opioid induced respiratory depression does the nurse recommend? Continuous pulse oximetry Serial arterial blood gas measurements Continuous capnography Apnea monitoring

Continuous capnography For early detection of opioid-induced respiratory depression, the nurse recommends continuous capnography. Capnography detects exhaled carbon dioxide which increases during opioid-induced respiratory depression.Capnography, to detect opioid-induced respiratory depression, has been proven to be superior for early detection of respiratory changes and is a more sensitive indicator of respiratory depression than pulse oximetry. Arterial blood gas measurement is painful and expensive, and is not practical to use this methodology on a continuous basis. Apnea monitoring will detect a lack of breathing, but capnography will alert the nurse to respiratory depression prior to that time.

An RN from pediatrics has "floated" to the medical-surgical unit. Which client is assigned to the float nurse? a) A 42-year-old with sickle cell disease receiving a transfusion of packed red blood cells b) A 50-year-old with pancytopenia needing assessment of risk factors for aplastic anemia c) A 55-year-old with folic acid deficiency anemia caused by alcohol abuse who needs counseling d) A 60-year-old with newly diagnosed polycythemia vera who needs teaching about the disease

Correct Answer: a Because sickle cell disease is commonly diagnosed during childhood, the pediatric nurse will be familiar with the disease and with red blood cell transfusion; therefore, he or she should be assigned to the client with sickle cell disease. Aplastic anemia, folic acid deficiency, and polycythemia vera are problems more commonly seen in adult clients who should be cared for by nurses who are more experienced in caring for adults.

The nurse is teaching a client with vitamin B12 deficiency anemia about dietary intake. Which type of food does the nurse encourage the client to eat? a) Dairy products b) Grains c) Leafy vegetables d) Starchy vegetables

Correct Answer: a Dairy products such as milk, cheese, and eggs will provide the vitamin B12 that the client needs. Grains, leafy vegetables, and starchy vegetables are not a source of vitamin B12.

Which client is at greatest risk for having a hemolytic transfusion reaction? a) A 34-year-old client with type O blood b) A 42-year-old client with allergies c) A 58-year-old immune-suppressed client d) A 78-year-old client

Correct Answer: a Hemolytic transfusion reactions are caused by blood type or Rh incompatibility. When blood that contains antigens different from the client's own antigens is infused, antigen-antibody complexes are formed in the client's blood. Type O is considered the universal donor, but not the universal recipient. The client with allergies would be most susceptible to an allergic transfusion reaction. The immune-suppressed client would be most susceptible to a transfusion-associated graft-versus-host disease. The older adult client would be most susceptible to circulatory overload.

The nurse is teaching a client about induction therapy for acute leukemia. Which client statement indicates a need for additional education? a) "After this therapy, I will not need to have any more." b) "I will need to avoid people with a cold or flu." c) "I will probably lose my hair during this therapy." d) "The goal of this therapy is to put me in remission."

Correct Answer: a Induction therapy is not a cure for leukemia, it is a treatment; therefore, the client needs more education to understand this. Because of infection risk, clients with leukemia should avoid people with a cold or flu. Induction therapy will most likely cause the client with leukemia to lose his or her hair. The goal of induction therapy is to force leukemia into remission.

The nurse is assessing a newly admitted client with thrombocytopenia. Which factor needs immediate intervention? a) Nosebleed b) Reports of pain c) Decreased urine output d) Increased temperature

Correct Answer: a The client with thrombocytopenia has a high risk for bleeding. The nosebleed should be attended to immediately. The client's report of pain, decreased urine output, and increased temperature are not the highest priority.

The nurse assesses multiple clients who are receiving transfusions of blood components. Which assessment indicates the need for the nurse's immediate action? a) A partial thromboplastin time (PTT) that is 1.2 times normal in a client who received a transfusion of fresh-frozen plasma (FFP) b) Respiratory rate of 36 breaths/min in a client receiving red blood cells c) Sleepiness in a client who received diphenhydramine (Benadryl) as a premedication d) Temperature of 99.1° F (37.3° C) for a client with a platelet transfusion

Correct Answer: b An increased respiratory rate indicates a possible hemolytic transfusion reaction; the nurse should quickly stop the transfusion and assess the client further. Because FFP is not usually given until the PTT is 1.5 times above normal, a PTT that is 1.2 times normal indicates that the FFP has had the desired response. Sleepiness is expected when Benadryl is administered. Temperature elevations are not an indication of an allergic reaction to a platelet transfusion, although the nurse may administer acetaminophen (Tylenol) to decrease the fever.

The nurse is assessing the endurance level of a client in a long-term care facility. What question does the nurse ask to get this information? a) "Are your feet or hands cold, even when you are in bed?" b) "Do you feel more tired after you get up and go to the bathroom?" c) "How much exercise do you get?" d) "What is your endurance level?"

Correct Answer: b Asking about feeling tired after using the bathroom is pertinent to the client's activity and provides a comparison. The specific activity helps the client relate to the question and provide needed answers. Asking about cold feet or hands does not address the client's endurance. The hospitalized client typically does not get much exercise; this would be a difficult assessment for a client in long-term care facility to make. Asking the client about his or her endurance level is too vague; the client may not know how to answer this question.

Which client does the nurse assign as a roommate for the client with aplastic anemia? a) A 23-year-old with sickle cell disease who has two draining leg ulcers b) A 28-year-old with glucose-6-phosphate dehydrogenase (G6PD) deficiency anemia who is receiving mannitol (Osmitrol) c) A 30-year-old with leukemia who is receiving induction chemotherapy d) A 34-year-old with idiopathic thrombocytopenia who is taking steroids

Correct Answer: b Because clients with aplastic anemia usually have low white blood cell counts that place them at high risk for infection, roommates such as the client with G6PD deficiency anemia should be free from infection or infection risk. The client with sickle cell disease has two draining leg ulcer infections that would threaten the diminished immune system of the client with aplastic anemia. The client with leukemia who is receiving induction chemotherapy and the client with idiopathic thrombocytopenia who is taking steroids are at risk for development of infection, which places the client with aplastic anemia at risk, too.

The nurse is transfusing 2 units of packed red blood cells to a postoperative client. What post-transfusion electrolyte imbalance does the nurse want to rule out? a) Hypercalcemia b) Hyperkalemia c) Hypomagnesemia d) Hyponatremia

Correct Answer: b During transfusion, some cells are damaged. These cells release potassium, thus raising the client's serum potassium level (hyperkalemia). This complication is especially common with packed cells and whole-blood products. High serum calcium levels, low magnesium levels, or low sodium levels are not expected with blood transfusions.

A 32-year-old client is recovering from a sickle cell crisis. His discomfort is controlled with pain medications and he is to be discharged. What medication does the nurse expect to be prescribed for him before his discharge? a) Heparin (Heparin) b) Hydroxyurea (Droxia) c) Tissue plasminogen activator (t-PA) d) Warfarin (Coumadin)

Correct Answer: b Hydroxyurea (Droxia) has been used successfully to reduce sickling of cells and pain episodes associated with sickle cell disease (SCD). Clients with SCD are not prescribed anticoagulants such as heparin or warfarin (Coumadin). t-PA is used as a "clot buster" in clients who have had ischemic strokes.

The nurse is caring for a client with sickle cell disease. Which action is most effective in reducing the potential for sepsis in this client? a) Administering prophylactic drug therapy b) Frequent and thorough handwashing c) Monitoring laboratory values to look for abnormalities d) Taking vital signs every 4 hours, day and night

Correct Answer: b Prevention and early detection strategies are used to protect the client in sickle cell crisis from infection. Frequent and thorough handwashing is of the utmost importance. Drug therapy is a major defense against infections that develop in the client with sickle cell disease, but is not the most effective way that the nurse can reduce the potential for sepsis. Continually assessing the client for infection and monitoring the daily complete blood count with differential white blood cell count is early detection, not prevention. Taking vital signs every 4 hours will help with early detection of infection, but is not prevention.

Which client statement indicates that stem cell transplantation that is scheduled to take place in his home is not a viable option? a) "I don't feel strong enough, but my wife said she would help." b) "I was a nurse, so I can take care of myself." c) "I will have lots of medicine to take." d) "We live 5 miles from the hospital."

Correct Answer: b Stem cell transplantation in the home setting requires support, assistance, and coordination from others. The client cannot manage this type of care on his own. The client must be emotionally stable to be a candidate for this type of care. It is acceptable for the client's spouse to support the client undergoing this procedure. It is not unexpected for the client to be taking several prescriptions. Five miles is an acceptable distance from the hospital, in case of emergency.

The nurse is reinforcing information about genetic counseling to a client with sickle cell disease who has a healthy spouse. What information does the nurse include? a) "Sickle cell disease will be inherited by your children." b) "The sickle cell trait will be inherited by your children." c) "Your children will have the disease, but your grandchildren will not." d) "Your children will not have the disease, but your grandchildren could."

Correct Answer: b The children of the client with sickle cell disease will inherit the sickle cell trait, but may not inherit the disease. If both parents have the sickle cell trait, their children could get the disease.

A recently admitted client who is in sickle cell crisis requests "something for pain." What does the nurse administer? a) Intramuscular (IM) morphine sulfate b) Intravenous (IV) hydromorphone (Dilaudid) c) Oral ibuprofen (Motrin) d) Oral morphine sulfate (MS-Contin)

Correct Answer: b The client needs IV pain relief, and it should be administered on a routine schedule (i.e., before the client has to request it). Morphine is not administered intramuscularly (IM) to clients with sickle cell disease (SCD). In fact, all IM injections are avoided because absorption is impaired by poor perfusion and sclerosed skin. Nonsteroidal anti-inflammatory drugs may be used for clients with SCD for pain relief once their pain is under control; however, in a crisis, this choice of analgesic is not strong enough. Moderate pain may be treated with oral opioids, but this client is in a sickle cell crisis; IV analgesics should be used until his or her condition stabilizes.

The nurse is educating a group of young women who have sickle cell disease (SCD). Which comment from a class member requires correction? a) "Frequent handwashing is an important habit for me to develop." b) "Getting an annual 'flu shot' would be dangerous for me." c) "I must take my penicillin pills as prescribed, all the time." d) "The pneumonia vaccine is protection that I need."

Correct Answer: b The client with SCD should receive annual influenza and pneumonia vaccinations; this helps prevent the development of these infections, which could cause a sickle cell crisis. Handwashing is a very important habit for the client with SCD to develop because it reduces the risk for infection. Prophylactic penicillin is given to clients with SCD orally twice a day to prevent the development of infection.

Which intervention most effectively protects a client with thrombocytopenia? a) Avoiding the use of dentures b) Encouraging the use of an electric shaver c) Taking rectal temperatures d) Using warm compresses on trauma sites

Correct Answer: b The client with thrombocytopenia should be advised to use an electric shaver instead of a razor. Any small cuts or nicks can cause problems because of the prolonged clotting time. Dentures may be used by clients with thrombocytopenia as long as they fit properly and do not rub. To prevent rectal trauma, rectal thermometers should not be used. Oral or tympanic temperatures should be taken. Ice (not heat) should be applied to areas of trauma.

A 56-year-old client admitted with a diagnosis of acute myelogenous leukemia (AML) is prescribed IV cytosine arabinoside for 7 days and an infusion of daunorubicin for the first 3 days. An infection develops. What knowledge does the nurse use to determine that the appropriate antibiotic has been prescribed for this client? a) Evaluating the client's liver function tests (LFTs) and serum creatinine levels b) Evaluating the client's white blood cell (WBC) count level c) Checking the culture and sensitivity test results to be certain that the requested antibiotic is effective against the organism causing the infection d) Recognizing that vancomycin (Vancocin) is the drug of choice used to treat all infections in clients with AML

Correct Answer: c Checking the culture and sensitivity test results to be certain that the requested antibiotic is effective against the organism causing the infection is the best action to take. Drug therapy is the main defense against infections that develop in clients undergoing therapy for AML. Agents used depend on the client's sensitivity to various antibiotics for the organism causing the infection. Although LFTs and kidney function tests may be influenced by antibiotics, these tests do not determine the effectiveness of the antibiotic. Although the WBC count is elevated in infection, this test does not influence which antibiotic will be effective in fighting the infection. Vancomycin may not be effective in all infections; culturing of the infection site and determining the organism's sensitivity to a cohort of drugs are needed, which will provide data on drugs that are capable of eradicating the infection in this client.

A hematology unit is staffed by RNs, LPN/LVNs, and unlicensed assistive personnel (UAP). When the nurse manager is reviewing documentation of staff members, which entry indicates that the staff member needs education about his or her appropriate level of responsibility and client care? a) "Abdominal pain relieved by morphine 4 mg IV; client resting comfortably and denies problems. B.C., RN" b) "Ambulated in hallway for 40 feet and denies shortness of breath at rest or with ambulation. T.Y., LPN" c) "Client reporting increased shortness of breath; oxygen increased to 4 L by nasal cannula. M.N., UAP" d) "Vital signs 37.0° C, heart rate 60, respiratory rate 20, blood pressure 110/68, and oximetry 98% on room air. L.D., UAP"

Correct Answer: c Determination of the need for oxygen and administration of oxygen should be done by licensed nurses who have the education and scope of practice required to administer it. All other documentation entries reflect appropriate delegation and assignment of care.

A client with leukemia is being discharged from the hospital. After hearing the nurse's instructions to keep regularly scheduled follow-up provider appointments, the client says, "I don't have transportation." How does the nurse respond? a) "A pharmaceutical company might be able to help." b) "I might be able to take you." c) "The local American Cancer Society may be able to help." d) "You can take the bus."

Correct Answer: c Many local units of the American Cancer Society offer free transportation to clients with cancer, including those with leukemia. Suggesting a pharmaceutical company is not the best answer; drug companies typically do not provide this type of service. Although the nurse offering to take the client is compassionate, it is not appropriate for the nurse to offer the client transportation. Telling the client to take the bus is dismissive and does not take into consideration the client's situation (e.g., the client may live nowhere near a bus route).

The nurse is caring for a client with neutropenia who has a suspected infection. Which intervention does the nurse implement first? a) Hydrate the client with 1000 mL of IV normal saline. b) Initiate the administration of prescribed antibiotics. c) Obtain requested cultures. d) Place the client on Bleeding Precautions.

Correct Answer: c Obtaining cultures to identify the infectious agent correctly is the priority for this client. Hydrating the client is not the priority. Administering antibiotics is important, but antibiotics should always be started after cultures are obtained. Placing the client on Bleeding Precautions is unnecessary.

A 32-year-old client recovering from a sickle cell crisis is to be discharged. The nurse says, "You and all clients with sickle cell disease are at risk for infection because of your decreased spleen function. For this reason, you will most likely be prescribed an antibiotic before discharge." Which drug does the nurse anticipate the health care provider will request? a) Cefaclor (Ceclor) b) Gentamicin (Garamycin) c) Penicillin V (Pen-V K) d) Vancomycin (Vancocin)

Correct Answer: c Prophylactic therapy with twice-daily oral penicillin reduces the incidence of pneumonia and other streptococcal infections and is the correct drug to use. It is a standard protocol for long-term prophylactic use in clients with sickle cell disease. Cefaclor (Ceclor) and vancomycin (Vancocin) are antibiotics more specific for short-term use and would be inappropriate for this client. Gentamicin (Garamycin) is a drug that can cause liver and kidney damage with long-term use.

The nurse is infusing platelets to a client who is scheduled for a hematopoietic stem cell transplant. What procedure does the nurse follow? a) Administer intravenous corticosteroids before starting the transfusion. b) Allow the platelets to stabilize at the client's bedside for 30 minutes. c) Infuse the transfusion over a 15- to 30-minute period. d) Set up the infusion with the standard transfusion Y tubing.

Correct Answer: c The volume of platelets—200 or 300 mL (standard amount)—needs to be infused rapidly over a 15- to 30-minute period. Administering steroids is not standard practice in administering platelets. Platelets must be administered immediately after they are received; they are considered to be quite fragile. A special transfusion set with a smaller filter and shorter tubing is used to get the platelets into the client quickly and efficiently.

The nurse is to administer packed red blood cells to a client. How does the nurse ensure proper client identification? a) Asks the client's name b) Checks the client's armband c) Reviews all information with another registered nurse d) Verifies the client's room number

Correct Answer: c With another registered nurse, verify the client by name and number, check blood compatibility, and note expiration time. Human error is the most common cause of ABO incompatibility reactions, even for experienced nurses. Asking the client's name and checking the client's armband are not adequate for identifying the client before transfusion therapy. Using the room number to verify client identification is never appropriate.

What is the most important environmental risk for developing leukemia? a) Direct contact with others with leukemia b) Family history c) Living near high-voltage power lines d) Smoking cigarettes

Correct Answer: d According to the American Cancer Society (ACS), the only proven lifestyle-related risk factor for leukemia is cigarette smoking. Leukemia is not contagious. Genetics is a strong indicator, but it is not an environmental risk factor. According to the ACS, living near high-voltage power lines is not a proven risk factor for leukemia.

The nurse is mentoring a recent graduate RN about administering blood and blood products. What does the nurse include in the data? a) Obtain the client's initial set of vital signs (VS) within the first 10 minutes of the infusion. b) Remain with the client who is receiving the blood for the first 5 minutes of the infusion. c) Use a 22-gauge needle to obtain venous access when starting the infusion. d) Verify with another RN all of the data on blood products.

Correct Answer: d All data are checked by two RNs. Human error is the most common cause of ABO incompatibilities in administering blood and blood products. Initial VS should be recorded before the start of infusion of blood, not after it has begun. The nurse remains with the client for the first 15 to 30 minutes (not 5) of the infusion. This is the period when any transfusion reactions are likely to happen. A 20-gauge needle (or a central line catheter) is used; the 22-gauge needle is too small.

The nurse assesses the client with which hematologic problem first? a) A 32-year-old with pernicious anemia who needs a vitamin B12 injection b) A 40-year-old with iron deficiency anemia who needs a Z-track iron injection c) A 67-year-old with acute myelocytic leukemia with petechiae on both legs d) An 81-year-old with thrombocytopenia and an increase in abdominal girth

Correct Answer: d An increase in abdominal girth in a client with thrombocytopenia indicates possible hemorrhage; this warrants further assessment immediately. The 32-year-old with pernicious anemia, the 40-year-old with iron deficiency anemia, and the 67-year-old with acute myelocytic leukemia do not indicate any acute complications, so the nurse can assess them after assessing the client with thrombocytopenia.

The nurse is caring for a client who is in sickle cell crisis. What action does the nurse perform first? a) Apply cool compresses to the client's forehead. b) Encourage the client's use of two methods of birth control. c) Increase food sources of iron in the client's diet. d) Provide pain medications as needed.

Correct Answer: d Analgesics are needed to treat sickle cell pain. Warm soaks or compresses can help reduce pain perception. Cool compresses do not help the client in sickle cell crisis. Birth control is not the priority for this client. Increasing iron in the diet is not pertinent for the client in sickle cell crisis.

The nurse is transfusing a unit of whole blood to a client when the health care provider requests the following: "Furosemide (Lasix) 20 mg IV push." What does the nurse do? a) Add furosemide to the normal saline that is infusing with the blood. b) Administer furosemide to the client intramuscularly (IM). c) Piggyback furosemide into the infusing blood. d) Wait until the transfusion has been completed to administer furosemide.

Correct Answer: d Completing the transfusion before administering furosemide is the best course of action in this scenario. Drugs are not to be administered with infusing blood products; they can interact with the blood, causing risks for the client. Stopping the infusing blood to administer the drug and then restarting it is also not the best decision. Changing the admission route is not a nursing decision.

Which would be an appropriate task to delegate to unlicensed assistive personnel (UAP) working on a medical-surgical unit? a) Administering erythropoietin to a client with myelodysplastic syndrome b) Assessing skin integrity on an anemic client who fell during ambulation c) Assisting a client with folic acid deficiency in making diet choices d) Obtaining vital signs on a client receiving a blood transfusion

Correct Answer: d Obtaining vital signs on a client is within the scope of practice for UAP. Administering medication, assessing clients, and assisting with prescribed diet choices are complex actions that should be done by licensed nurses.

A client who is receiving a blood transfusion suddenly exclaims to the nurse, "I don't feel right!" What does the nurse do next? a) Call the Rapid Response Team. b) Obtain vital signs and continue to monitor. c) Slow the infusion rate of the transfusion. d) Stop the transfusion.

Correct Answer: d The client may be experiencing a transfusion reaction; the nurse should stop the transfusion immediately. Calling the Rapid Response Team or obtaining vital signs is not the first thing that should be done. The nurse should not slow the infusion rate, but should stop it altogether.

A client with thrombocytopenia is being discharged. What information does the nurse incorporate into the teaching plan for this client? a) "Avoid large crowds." b) "Drink at least 2 liters of fluid per day." c) "Elevate your lower extremities when sitting." d) "Use a soft-bristled toothbrush."

Correct Answer: d Using a soft-bristled toothbrush reduces the risk for bleeding in the client with thrombocytopenia. Avoiding large crowds reduces the risk for infection, but is not specific to the client with thrombocytopenia. Increased fluid intake reduces the risk for dehydration, but is not specific to the client with thrombocytopenia. Elevating extremities reduces the risk for dependent edema, but is not specific to the client with thrombocytopenia.

What are the risk factors for the development of leukemia? (Select all that apply.) a) Bone marrow hypoplasia b) Chemical exposure c) Down syndrome d) Ionizing radiation e) Multiple blood transfusions f) Prematurity at birth

Correct Answers: a, b, c, d Reduced production of blood cells in the bone marrow is one of the risk factors for developing leukemia. Exposure to chemicals through medical need or by environmental events can also contribute. Certain genetic factors contribute to the development of leukemia; Down syndrome is one such condition. Radiation therapy for cancer or other exposure to radiation, perhaps through the environment, also contributes. There is no indication that multiple blood transfusions are connected to clients who have leukemia. Although some genetic factors may influence the incidence of leukemia, prematurity at birth is not one of them.

A distant family member arrives to visit a female client recently diagnosed with leukemia. The family member asks the nurse, "What should I say to her?" Which responses does the nurse suggest? (Select all that apply.) a) "Ask her how she is feeling." b) "Ask her if she needs anything." c) "Tell her to be brave and to not cry." d) "Talk to her as you normally would when you haven't seen her for a long time." e) "Tell her what you know about leukemia."

Correct Answers: a, b, d Asking the client how she is feeling is a broad general opening and would be nonthreatening to the client. Asking if she needs anything is a therapeutic communication of offering self and would be considered to be therapeutic and helpful to the client. The family member should talk to her as she normally would when she hasn't seen her in a long time. There is no need to act differently with the client. If she wants to offer her feelings, keeping a normal atmosphere facilitates that option. Acting as if things are "different" because she has cancer takes the control of the situation from the client. Telling her to be brave and not to cry is callous and unfeeling; if the client is feeling vulnerable and depressed, telling her to "be brave" shuts off any opportunity for her to express her feelings. There is no need to inform the client about her disease, unless she asks about it. Opening the conversation with discussion about leukemia should be the client's prerogative.

The nurse is teaching a client with newly diagnosed anemia about conserving energy. What does the nurse tell the client? (Select all that apply.) a) "Allow others to perform your care during periods of extreme fatigue." b) "Drink small quantities of protein shakes and nutritional supplements daily." c) "Perform a complete bath daily to reduce your chance of getting an infection." d) "Provide yourself with four to six small, easy-to-eat meals daily." e) "Perform your care activities in groups to conserve your energy." f) "Stop activity when shortness of breath or palpitations are present."

Correct Answers: a, b, d, f It is critical to have others help the anemic client who is extremely tired. Although it may be difficult for him or her to ask for help, this practice should be stressed to the client. Drinking small protein or nutritional supplements will help rebuild the client's nutritional status. Having four to six small meals daily is preferred over three large meals; this practice conserves the body's expenditure of energy used in digestion and assimilation of nutrients. Stopping activities when strain on the cardiac or respiratory system is noted is critical. A complete bath should be performed only every other day; on days in between, the client can be taught to take a "mini" sponge bath, which will conserve energy and still be safe in preventing the risks for infection. Care activities should be spaced every hour or so rather than in groups to conserve energy; the time just before and after meals should be avoided.

What are serious side effects of antiviral agents prescribed for a client with acute myelogenous leukemia? (Select all that apply.) a) Cardiomyopathy b) Nephrotoxicity c) Ototoxicity d) Stroke e) Diarrhea

Correct Answers: b, c Antiviral agents, although helpful in combating severe infection, have serious side effects, especially nephrotoxicity and ototoxicity. Cardiomyopathy and stroke are not serious side effects of antiviral agents. Diarrhea is a mild side effect associated with antibiotic therapy.

What are the typical clinical manifestations of anemia? (Select all that apply.) a) Decreased breath sounds b) Dyspnea on exertion c) Elevated temperature d) Fatigue e) Pallor f) Tachycardia

Correct Answers: b, d, e, f Difficulty breathing—especially with activity—is common with anemia. Lower levels of hemoglobin carry less O2 to the cells of the body. Fatigue is a classic symptom of anemia; lowered O2 levels contribute to a faster pulse (i.e., cardiac rate) and tend to "wear out" a client's energy. Lowered O2 levels deliver less oxygen to all cells, making clients with anemia pale—especially their ears, nail beds, palms, and conjunctivae and around the mouth. Respiratory problems with anemia do not include changes in breath sounds; dyspnea and decreased oxygen saturation levels are present. Skin is cool to the touch, and an intolerance to cold is noted; elevated temperature would signify something additional, such as infection.

An 82-year-old client with anemia is requested to receive 2 units of whole blood. Which assessment findings cause the nurse to discontinue the transfusion because it is unsafe for the client? (Select all that apply.) a) Capillary refill less than 3 seconds b) Decreased pallor c) Flattened superficial veins d) Hypertension e) Hypotension f) Rapid, bounding pulse

Correct Answers: d, e, f In an older adult receiving a transfusion, hypertension is a sign of overload, low blood pressure is a sign of a transfusion reaction, and a rapid and bounding pulse is a sign of fluid overload. In this scenario, 2 units, or about a liter of fluid, could be problematic. Capillary refill time that is less than 3 seconds is considered to be normal and would not pose a problem. Increased (not decreased) pallor and cyanosis are signs of a transfusion reaction, while swollen (not flattened) superficial veins are present in fluid overload in older adult clients receiving transfusions.

The nurse is caring for a client with heart failure and acute kidney injury. For which of these breath sounds will the nurse assess? Crackles Rhonchi Pleural friction rub Wheeze

Crackles When caring for a client with heart failure and acute kidney disease, the nurse would assess for crackles. Crackles are described as a popping, discontinuous sound caused by air moving into previously deflated airways or areas of fluid.Rhonchi are low-pitched, coarse snoring sounds caused by thick secretions in larger airways. A pleural friction rub sounds grating, loud, or scratchy as inflamed surfaces of the pleura rub together. Wheezes are frequently referred to as musical or squeaky sounds caused by bronchospasm. They may occur on inspiration or on expiration as air rushes through narrowed airways.

The nurse auscultates popping, discontinuous sounds over the client's anterior chest. How does the nurse classify these sounds? Crackles Rhonci Wheeze Pleural Friction Rub

Crackles Crackles are described as a popping, discontinuous sound caused by air moving into previously deflated airways. The airways have been deflated due to the presence of fluids in the lungs, and crackles should be considered to be a sign of fluid overload. Rhonchi are low-pitched, coarse snoring sounds caused by fluid or secretions in larger airways. A pleural friction rub sounds grating, loud, or scratchy as inflamed surfaces of the pleura rub together. Wheezes are frequently referred to as musical or squeaky; they may occur on inspiration or on expiration and may be heard without a stethoscope as air rushes through narrowed airways.

21. A patient has chronic bronchitis. The nurse plans interventions for inadequate oxygenation based on which set of clinical manifestations? A. Chronic cough, thin secretions, and chronic infection B. Respiratory alkalosis, decreased PaCO2, and increased PaO2 C. Areas of chest tenderness and sputum production (often with hemoptysis) D. Large amounts of thick secretions and repeated infections

D

27. What is the purpose of pulmonary function testing? A. Determines the oxygen liter flow rates required by the patient B. Measures blood gas levels before bronchodilators are administered C. Evaluates the movement of oxygenated blood from the lung to the heart D. Distinguishes airway disease from restrictive lung disease

D

30. What principle guides the nurse when providing oxygen therapy for a patient with COPD? A. The patient depends on a high serum carbon dioxide level to stimulate the drive to breathe. B. The patient requires a low serum oxygen level for the stimulus to breathe to work. C. The patient who receives oxygen therapy at a high flow rate is at risk for a respiratory arrest. D. The patient should receive oxygen therapy at rates to reduce hypoxia and bring SpO2 levels up between 88% and 92%.

D

37. What is the advantage of using the aerosol route for administering short-acting beta2 agonists? A. Achieves a rapid and effective anti-inflammatory action. B. Reduces risk for fungal infections C. Increases patient compliance because it is easy to use D. Provides rapid therapy with fewer systemic side effects

D

58. The nurse is taking a report on a patient who had a pneumonectomy 4 days ago. Which question is the best to ask during the shift report? A. "Does the provider want us to continue encouraging use of the spirometer?" B. "How much drainage did you see in the Pleur-Evac during your shift?" C. "Do we have to request to 'milk' the patient's chest tube? D. "Does the surgeon want the patient placed on the nonoperative side?"

D

60. Upon observation of a chest tube setup, the nurse reports to the provider that there is a leak in the chest tube and system. How has the nurse identified this problem? A. Drainage in the collection chamber has decreased. B. The bubbling in the suction chamber has suddenly increased. C. Fluctuation in the water seal chamber has stopped. D. There was onset of continuous vigorous bubbling in the water seal chamber.

D

67. Which intervention promotes comfort in dyspnea management for a patient with lung cancer? A. Administer morphine only when the patient requests it. B. Place the patient in a supine position with a pillow under the knees and legs C. Encourage coughing and deep-breathing and independent ambulation. D. Provide supplemental oxygen via cannula or mask.

D

8. The nurse is taking a medical history on a new patient who has come to the office for a check up. The patient states that he was supposed to take a medication called montelukast (Singulair), but that he never got the prescription filled. What is the best response by the nurse? A. "When did you first get diagnosed with a respiratory disorder?" B. "Why didn't you get the prescription filled?" C. "Tell me how you feel about your decision to not fill the prescription." D. "Tell me about how your asthma has been recently?"

D

The nurse is caring for a client with severe acute respiratory syndrome. What is the most important precaution the nurse should take when preparing to suction this client? A) Keeping the head of the bed elevated 30 to 45 degrees B) Performing oral care after suctioning the oropharynx C) Washing hands and donning gloves prior to the procedure D) Wearing a disposable particulate mask respirator and protective eyewear (Chp. 31; elsevier resources)

D) Wearing a disposable particulate mask respirator and protective eyewear (Chp. 31; elsevier resources)

The nurse is preparing a teaching plan for a client with migraine headaches who is receiving propranolol (Inderal) for migraine headaches. What health teaching by the nurse is important for the client? A. "Take this drug only when you have symptoms at the beginning of a migraine headache." B. "This drug is low dose, so you don't have to worry about your heart rate or blood pressure." C. "This drug will relieve the pain during the aura phase soon after a headache has started." D. "Take this drug as prescribed every day, even when feeling well, to prevent a migraine."

D. "Take this drug as prescribed every day, even when feeling well, to prevent a migraine." Rationale A. For prevention purposes, this drug should be taken daily, not intermittently. Abruptly stopping a beta-blocker may cause adverse symptoms. Reference: p. 931, Physiological Integrity B. This drug can lower blood pressure and decrease pulse rate. Reference: p. 931, Physiological Integrity C. Inderal is considered a preventive drug; efficacy as an abortive drug has not been substantiated by research. Reference: p. 931, Physiological Integrity D. Propranolol (Inderal) is a beta-blocker and is taken to prevent the development of a migraine headache. Reference: p. 931, Physiological Integrity

The client has Parkinson disease (PD). Which nursing intervention best protects the client from injury? A. Discouraging the client from activity B. Encouraging the client to watch the feet when walking C. Suggesting that the client obtain assistance in performing ADLs D. Monitoring the client's sleep patterns

D. Monitoring the client's sleep patterns Rationale A. Active and passive range-of-motion (ROM) exercises, muscle stretching, and activity are important to keep the client with PD mobile and flexible. B. The client with PD should avoid watching his or her feet when walking to prevent falls. C. The client with PD should be encouraged to participate as much as possible in self-management, including ADLs. Occupational and physical therapists can provide training in ADLs and the use of adaptive devices, as needed, to facilitate independence. D. Clients with PD tend to not sleep well at night because of drug therapy and the disease itself. Some clients nap for short periods during the day and may not be aware that they have done so. This sleep misperception could put the client at risk for injury (e.g., falling asleep while driving).

An RN and an LPN/LVN are working together to provide care for a client hospitalized with dyspnea who requires all of these nursing actions. Which of these actions is best accomplished by the RN? A. Administer the purified protein derivative (PPD) for tuberculosis testing. B. Assess vital signs and the puncture site after thoracentesis. C. Monitor oxygen saturation using pulse oximetry every 4 hours. D. Plan client and family teaching regarding upcoming pulmonary function testing.

D. Plan client and family teaching regarding upcoming pulmonary function testing. Correct: Developing the teaching plan is the most complex of the skills listed and requires RN education and licensure.

The nurse is administering the intake assessment for a newly admitted client with a history of seizures. The client suddenly begins to seize. What does the nurse do next? A. Documents the length and time of the seizure B. Forces a tongue blade in the mouth C. Restrains the client D. Positions the client on the side

D. Positions the client on the side Rationale A. Documenting the length and time of seizures is important, but not while the seizure is occurring. B. Forcing a tongue blade in the mouth can cause damage. C. Restraining the client can cause injury. D. Turning the client on the side during a generalized tonic-clonic or complex partial seizure is indicated because he or she may lose consciousness.

A client with pneumonia caused by aspiration after alcohol intoxication has just been admitted. The client is febrile and agitated. Which health care provider order should the nurse implement first?

Draw aerobic and anaerobic blood cultures Obtaining aerobic and anaerobic cultures is the first action the nurse should perform and is standard procedure in a febrile client for whom antibiotics have been requested. Levofloxacin, an antibiotic, is important to administer, but blood cultures should be drawn before antibiotics are started. Unless this client is a danger to self or staff, giving lorazepam (Ativan) for agitation is not the first action; the question indicates that the client is agitated but does not indicate whether other attempts to control agitation have been tried, such as decreasing stimulation. A referral to social work for alcohol counseling will be initiated before the time of discharge; this client is febrile and agitated, and a referral is not the immediate concern.

Which method is the best way to prevent outbreaks of pandemic influenza?

Early recognition and quarantine The recommended approach to disease prevention consists of early recognition of new cases and implementing community and personal quarantine to reduce exposure to the virus. Public gatherings should be avoided only if a widespread outbreak has occurred in a community. No vaccine is available for pandemic influenza. The pneumonia vaccine is recommended for high-risk populations because pneumonia may be a complication of influenza. The current influenza vaccine is updated, re-evaluated, and changed yearly to meet anticipated changes in the virus. When a cluster of cases is discovered in an area, the antiviral drugs oseltamivir (Tamiflu) and zanamivir (Relenza) should be widely distributed to help reduce the severity of the infection and to decrease mortality.

A client is admitted to the medical floor with a new diagnosis of lung cancer. How does the nurse assist the client initially with the anxiety associated with the new diagnosis? Encourage the client to ask questions and verbalize concerns. Provide privacy for the client to be alone to deal with his or her own feelings. Medicate the client with diazepam for anxiety every 8 hours. Provide journals about cancer treatment.

Encourage the client to ask questions and verbalize concerns. Anxiety causes increased oxygen consumption, and oxygen availability is limited in lung cancer; the availability of the nurse to answer questions and listen to the client's concerns will decrease anxiety. The client may choose to be alone, although this may be a maladaptive coping behavior. Scheduled medication does not solve the anxiety associated with the new diagnosis, although administering Valium (diazepam) every 8 hours will assist with reducing the anxiety; it is more important to work with the client to determine the cause of the anxiety and assist him or her in dealing with those issues first. Knowledge about cancer may help relieve anxiety but is not the best initial step in a newly diagnosed client. The nurse must first assess how the client learns best and what the client's needs are. The nurse also must be aware of the plan of care for the client.

Which nursing intervention is the priority in preparing a client for pulmonary function testing (PFT)? Administer bronchodilator medication on call. Encourage clear fluid intake 12 hours before the procedure. Ensure the client does not smoke for 6 hours before the test. Provide supplemental oxygen.

Ensure the client does not smoke for 6 hours before the test. The essential nursing intervention for a client being prepared for a PFT is to make sure that the client does not smoke for 6 hours before the test. Smoking can alter parts of the PFT (diffusing capacity [DLCO]), yielding inaccurate results.Administering bronchodilators is not indicated for PFT, but they may be withheld for 4 to 6 hours before the test. Encouraging fluid intake does not have an effect on PFT testing. Supplemental oxygen is not required and will alter the results of PFT. However, oxygen may be given if the client develops distress during testing.

The nurse is preparing the client for a diagnostic bronchoscopy. Which nursing intervention is essential for the nurse to perform prior to the procedure? Obtain informed consent. Ensure the client has had nothing by mouth. Review dietary and medication allergies. Perform aggressive chest physiotherapy.

Ensure the client has had nothing by mouth. When preparing a client for a diagnostic bronchoscopy, it is essential for the nurse to make sure the client is NPO for 4 to 8 hours before the procedure to reduce the risk for aspiration.It is important to verify allergies, however ensuring NPO status is maintained is essential to prevent aspiration, which can be life threatening. The nurse will verify that consent for the procedure was obtained. Until the client has a gag reflex and is fully alert, he or she should be maintained on NPO status to prevent aspiration. Aggressive chest physiotherapy is not indicated in a client who has had a bronchoscopy and may cause bleeding if biopsies have been obtained.

What does the nurse do first when setting up a safe environment for the new client on oxygen?

Ensures that no combustion hazards are present in the room Oxygen is highly flammable. The nurse must ensure that no open flames or combustion hazards are present in a room where oxygen is in use. Protective clothing is not necessary for a client who requires oxygen therapy other than the use of Standard Precautions. The oxygen delivery setting is usually determined in conjunction with the respiratory therapy care partner. Although the setting is important for safe administration, it is not necessary for a safe environment. Pulse oximetry would be useful for monitoring the client's oxygenation status, but is not necessary for a safe environment.

The nurse in the medical clinic is performing an assessment on an older adult client. Which finding requires further assessment by the nurse? Inability to state name and date of birth Slight kyphoscoliosis Soft speaking voice Need to rest after activity

Inability to state name and date of birth The nurse would further assess the client who is unable to state name and date of birth. The older adult has a higher risk for hypoxemia than a younger client. The older adult can become confused during acute respiratory conditions, which requires additional investigation.Progressive Kyphoscoliosis occurs with aging because the thorax becomes shorter. With aging, laryngeal muscles lose elasticity, and airways lose cartilage causing the client's voice to become soft and difficult to understand. This is due to age-related changes in chest wall compliance and elasticity. Increased need for rest periods during exercise may occur.

In the older adult client, which respiratory change does not require further assessment by the nurse? A. Increased anteroposterior (AP) diameter B. Increased respiratory rate C. Shortness of breath D. Sputum production

Increased anteroposterior (AP) diameter Correct: Increased AP diameter is normal with aging.

A client who has been homeless and has spent the past 6 months living in shelters has been diagnosed with confirmed tuberculosis (TB). Which medications does the nurse expect to be ordered for the client?

Isoniazid (INH), rifampin (Rifadin), pyrazinamide (Zinamide), ethambutol (Myambutol) The combination of isoniazid, rifampin, pyrazinamide, and ethambutol is used to treat TB. Metronidazole is used to treat anaerobic bacteria and some parasites, but is not effective against TB. Acyclovir is used to treat viral infection. Flunisolide is a corticosteroid that is useful in asthma or other airway disease to prevent wheezing. Prednisone is a steroid. Guaifenesin is a mucolytic. Ketorolac is a nonsteroidal anti-inflammatory drug that is used for short-term pain relief. Salmeterol and cromolyn sodium would most likely be given to clients with respiratory difficulties such as poorly controlled asthma from allergic sources. Dexamethasone is a steroid.

When caring for a client with hemorrhage secondary to kidney trauma, the nurse provides volume expansion. Which element does the nurse anticipate will be used? Fresh-frozen plasma Platelet infusions 5% dextrose in water Normal saline solution (NSS)

Isotonic solutions and crystalloid solutions are administered for volume expansion; 0.9% sodium chloride (NSS) and 5% dextrose in 0.45% sodium chloride may be used. Clotting factors, contained in fresh-frozen plasma, are given for bleeding, not for volume expansion. Platelet infusions are administered for deficiency of platelets. A solution hypotonic to the client's blood, 5% dextrose, is administered for nutrition or hypernatremia, not for volume expansion.

The nurse has just received change-of-shift report about a group of clients on the neurosurgical unit. Which client does the nurse attend to first? Adult postoperative left craniotomy client whose hand grip is weaker on the right Middle-aged adult client who had a cerebral aneurysm clipping and is becoming increasingly confused Older adult client who had a carotid endarterectomy and is unable to state the day of the week Young adult client involved in a motor vehicle crash (MVC) who is yelling obscenities at the nursing staff

Middle-aged adult client who had a cerebral aneurysm clipping and is becoming increasingly confused After a change-of-shift report, the neurosurgical nurse would first attend to the middle-aged client who had a clipping of a cerebral aneurysm and is now becoming increasingly confused. A change in level of consciousness is an early indication that central neurologic function has declined. The primary care provider must be notified immediately.The other clients are not the nurse's first priority. The young adult who is post-MVC does need to be assessed, but the client's behavior does not indicate a decline in neurologic function. The postoperative left craniotomy client and the older adult also need to be assessed, but these clients' neurologic assessment indicates better function.

A client has returned to the medical surgical unit after a bronchoscopy. Which nursing task is best for the charge nurse to delegate to the experienced unlicensed assistive personnel (UAP)? Assess breath sounds. Offer clear liquids when gag reflex returns. Determine level of consciousness. Monitor blood pressure and pulse.

Monitor blood pressure and pulse. The best nursing task for the charge nurse to delegate to the experienced unlicensed assistive personnel (UAP) is monitoring blood pressure and pulse. An experienced UAP would have experience in taking client vital signs after procedures requiring conscious sedation or anesthesia.Evaluating breath sounds, gag reflex, and determining level of consciousness are considered nursing assessments and require the skill and knowledge of a higher-level provider or professional nurse.

A client who has recently traveled to Vietnam comes to the emergency department with fatigue, lethargy, night sweats, and a low-grade fever. What is the nurse's first action?

Place a respiratory mask on the client. The concern is that this client has TB. A respiratory mask should be placed on the client immediately. Requesting medications for TB is not appropriate until the client has been evaluated and a diagnosis has been made. Performing a TB test will be important, but this is not the top priority. It is important to remember to let the client know that results will not be available for at least 48 hours after the test is administered. Further testing of this client needs to be completed and a diagnosis made before family members are tested.

A client is admitted to the emergency department (ED) with a possible diagnosis of avian influenza ("bird flu"). Which of these actions included in the hospital protocol for avian influenza will the nurse take first?

Place the client in a negative air pressure room. If a client is exhibiting symptoms of avian flu or any other pandemic influenza, he or she is assumed to be contagious until proven otherwise. Preventing the spread of disease to the community is the top priority, so placing the client in a negative air pressure room is the nurse's first action. If avian influenza is diagnosed, it is important that those exposed receive oseltamivir or zanamivir (Relenza) within 48 hours of contact with the client. Obtaining specimens will be important to determine whether the client has avian influenza; this test takes approximately 40 minutes to complete. A client with avian flu will become dehydrated because of diarrhea, so starting an IV to administer rehydration fluid is important, but is not the first priority.

The RN and the LPN/LVN are working together to provide care for a client hospitalized with dyspnea who requires all of these nursing actions. Which action is best accomplished by the RN? Administer purified protein derivative (PPD) for tuberculosis testing. Assess vital signs and the puncture site one day post thoracentesis. Monitor oxygen saturation using pulse oximetry every 4 hours. Plan client and family teaching regarding upcoming pulmonary function testing.

Plan client and family teaching regarding upcoming pulmonary function testing. The most appropriate action for the RN to perform is developing the teaching plan for upcoming pulmonary function test. These skills are complex, requiring use of the nursing process, and are not in the scope of practice of the LPN/LVN.Medication administration and monitoring of vital signs and client status after procedures can be accomplished by the LPN/LVN. Monitoring of oxygen saturation by pulse oximetry can also be included in the vital signs assessment.

A client had a thoracentesis 1 day ago. He calls the home health agency and tells the nurse that he is very short of breath and anxious. What is the major concern of the nurse? Pulmonary Emboli Pneumonia Formation of abcess Pneumothorax

Pneumothorax A pneumothorax would be the complication of thoracentesis that causes the greatest concern, along with these symptoms. Although it is possible that an abscess has formed, this is not the most likely diagnosis because it would not cause a great deal of shortness of breath. It is not likely that pneumonia would develop this rapidly, causing this level of symptoms. Thoracentesis is not a cause of pulmonary emboli.

Which information is most important for the nurse to communicate to the primary care provider (PCP) about a client who is scheduled for CT angiography? Allergy to penicillin History of bacterial meningitis Poor skin turgor and dry mucous membranes The client's dose of metformin (Glucophage) held today

Poor skin turgor and dry mucous membranes The most important information for the nurse to communicate to the PCP about a client scheduled for a CT angiography is the client with poor skin turgor and dry mucous membranes. This assessment indicates dehydration which places the client at risk for contrast induced nephropathy.Allergy to penicillin, history of bacterial meningitis, and withheld metformin will need to be reported as part of the client hand-off to Radiology (SAFETY).

The nurse is preparing to admit an adult client with pertussis. Which symptom does the nurse anticipate finding in this client?

Post-cough emesis Clients with pertussis will have paroxysms of coughing often followed by changes in color and/or vomiting. Adults do not usually have the characteristic whooping sound associated with coughing that children with pertussis exhibit. Hemoptysis may occur after the acute phase when changes in the respiratory mucosa occur. Mild, cold-like symptoms occur in the initial stages of pertussis and generally do not require hospitalization.

Community health nurses are tasked with providing education on prevention of respiratory infection for diseases such as the flu. Which target audience is given the highest priority?

Prison staff and inmates High-risk groups for respiratory infection include those who live in crowded areas such as long-term care facilities, prisons, and mental health facilities. Although homeless people are a high priority, they are not the group at greatest risk of those listed. Education could be provided in shelters or during outreach activities. Hospital staff are at risk owing to their contact with ill clients and family members; however, they are already aware of how to prevent respiratory infection. Politicians are not at higher risk for respiratory infection than any other group with public exposure.

When preparing a client for nephrostomy tube insertion, it is essential for the nurse to monitor which factor before the procedure? Blood urea nitrogen (BUN) and creatinine Hemoglobin and hematocrit (H&H) Intake and output (I&O) Prothrombin time (PT) and international normalized ratio (INR)

Prothrombin time (PT) and international normalized ratio (INR) The procedure will be cancelled or delayed if coagulopathy in the form of prolonged PT/INR exists because dangerous bleeding may result. Nephrostomy tubes are placed to prevent and treat kidney damage; monitoring BUN and creatinine is important, but is not essential before this procedure. H&H is monitored to detect anemia and blood loss; this would not occur before the procedure. This client should be on I&O during the entire hospitalization; it is not necessary only before the procedure, but throughout the admission.

The nurse notices a visitor walking into the room of a client on airborne isolation with no protective gear. What does the nurse do?

Provides a mask to the visitor Because the visitor is entering the client's isolation environment, the visitor must wear a mask. The client typically must wear a mask only when he or she is outside of an isolation environment. Turning the visitor away is inappropriate and unnecessary. It would not be necessary for the visitor to wear an air respirator.

When taking the health history of a client with acute glomerulonephritis (GN), the nurse questions the client about which related cause of the problem? Recent respiratory infection Hypertension Unexplained weight loss Neoplastic disease

Recent respiratory infection An infection often occurs before the kidney manifestations of acute GN. The onset of symptoms is about 10 days from the time of infection. Hypertension is a result of glomerulonephritis, not a cause. Weight gain, not weight loss, is symptomatic of fluid retention in GN. Cancers are not part of the cause of GN.

When caring for a client who has just undergone thoracentesis, which of these interventions does the nurse perform first? Encourage coughing and deep breathing. Schedule an immediate chest x-ray. Document the volume of removed fluid in the medical record. Set up a water seal drainage unit.

Schedule an immediate chest x-ray. After thoracentesis, the nurse first makes sure a chest x-ray is performed to rule out possible pneumothorax and mediastinal shift (shift of central thoracic structures toward one side).Coughing and deep breathing is done to promote lung expansion as part of the treatment for the underlying disorder. This can wait until a chest x-ray is completed. The volume of fluid will be recorded in the medical record, after the nurse schedules the x-ray to ensure a pneumothorax did not occur. Pigtail drain catheters may be left in place to a waterseal drainage system, rather than performing thoracentesis aspiration on a recurring basis, but this action is not standard.

The emergency department nurse is assessing a client who believes he has sustained a pneumothorax after an outpatient thoracentesis earlier today. For which of these symptoms will the nurse assess? Select all that apply. Slowing heart rate Sensation of air hunger Tracheal deviation Pain on the unaffected side Blue discoloration of the lips

Sensation of air hunger Tracheal deviation Blue discoloration of the lips The nurse would assess for a pneumothorax if the client has a sensation of air hunger, tracheal deviation, and blue discoloration of the lips. All clients need to be taught to go to the ED for symptoms of a pneumothorax after a thoracentesis. Symptoms include pain on the affected side, rapid heart rate, rapid, shallow respirations, sensation of air hunger, prominence of the affected side that does not move in and out with respiratory effort, tracheal deviation to the unaffected, new onset of "nagging" cough and cyanosis.Tachycardia, rather than bradycardia, is consistent with a pneumothorax. Pain occurs on the affected side, not the unaffected side.

The client is a marathon runner who has asthma. Which category of medication is used as a rescue inhaler?

Short-acting beta agonists Short-acting beta agonist medications have a rapid onset and cause bronchodilation; they would be excellent for marathon running because some types of asthma may be exercise-induced. Corticosteroids disrupt production pathways of inflammatory mediators. Maximum effectiveness requires 48 to 72 hours of continued use; therefore, they are not appropriate as a rescue medication. Long-acting beta agonists do cause bronchodilation, but have a slow onset; they are not used as rescue inhalers. NSAIDs stabilize the membranes of mast cells and prevent release of inflammatory mediators. They have a slow onset of action and are used for prevention of symptoms, not as rescue medication.

The nurse has been teaching improved airflow techniques to the client, who has continued to have restrictive breathing problems. Which is the best indicator of success?

SpO2 level of 92% after ambulating 50 feet Maintaining a baseline SpO2 of 92% after ambulating 50 feet is an excellent indicator that the client has achieved better airflow, and that the nurse's teaching has been effective. A yellow reading means "caution," which indicates narrowing airways. Although a productive cough may be an indication of success, it can also be an indication of infection. ABGs are invasive, costly, and painful and are not the most effective indicator of successful teaching in this situation.

A newly admitted client who is diabetic and has pyelonephritis and prescriptions for intravenous antibiotics, blood glucose monitoring every 2 hours, and insulin administration should be cared for by which staff member? RN whose other assignments include a client receiving chemotherapy for renal cell carcinoma RN who is caring for a client who just returned after having renal artery balloon angioplasty RN who has just completed preoperative teaching for a client who is scheduled for nephrectomy RN who is currently admitting a client with acute hypertension and possible renal artery stenosis

The client scheduled for nephrectomy is the most stable client; the RN caring for this client will have time to perform the frequent monitoring and interventions that are needed for the newly admitted client. The client receiving chemotherapy will require frequent monitoring by the RN. The client after angioplasty will require frequent vital sign assessment and observation for hemorrhage and arterial occlusion. The client with acute hypertension will need frequent monitoring and medication administration.

The nurse is caring for four clients who came to the emergency department with a productive cough. Which of these clients requires immediate intervention by the nurse? The client with blood in the sputum The client with mucoid sputum The client with pink, frothy sputum The client with yellow sputum

The client with pink, frothy sputum The nurse would immediately assess and interview the client with a productive cough and pink, frothy sputum. Pink, frothy sputum is common with pulmonary edema, a life-threatening exacerbation of heart failure. This client requires immediate assessment and intervention.Blood in the sputum may occur with chronic bronchitis or lung cancer. These conditions develop over time and therefore do not require immediate attention. Mucoid sputum may be related to smoking and does not require immediate attention. Although yellow sputum may indicate an infection that requires treatment, the condition is not life threatening.

Which sign or symptom, when assessed in a client with chronic glomerulonephritis (GN), warrants a call to the health care provider? Mild proteinuria Third heart sound (S3) Serum potassium of 5.0 mEq/L Itchy skin

Third heart sound (S3) S3 indicates fluid overload secondary to failing kidneys; the provider should be notified and instructions obtained. Mild proteinuria is an expected finding in GN. A serum potassium of 5.0 mEq/L reflects a normal value; intervention would be needed for hyperkalemia. Although itchy skin may be present as kidney function declines, it is not a priority over fluid excess.

The nurse in a life care community for geriatric clients is providing education to a group of residents on expected changes during aging. Which of these activities does the nurse encourage the older adult to perform to maintain respiratory function? Stay in bed to prevent fatigue. Walk as tolerated each day. Consume adequate calcium. Perform oral hygiene twice daily.

Walk as tolerated each day. The best activity for the older adult to perform in order to maintain respiratory function is to try and walk each day. Ambulation to the client's ability is easily performed in an older adult facility as it does not require special equipment. Health and fitness help keep losses in respiratory functioning to a minimum.Older clients have less tolerance for exercise and may need increased rest periods during exercise. However, bedrest is not necessary or desirable. Encouraging adequate calcium intake to prevent osteoporosis is more helpful prior to menopause, and is less helpful with elderly clients. Oral hygiene aids in the removal of secretions when present, but is not the best intervention to maintain respiratory function.

Which client diagnosed with neurologic injury is typically at highest risk for depression? Older man with a mild stroke Older woman with a seizure Young man with a spinal cord injury Young woman with a minor closed head injury

Young man with a spinal cord injury A young man with a spinal cord injury is at highest risk for depression. Although each individual responds differently, young adults who experience a spinal cord injury and loss of independent movement are more likely to experience depression.Keeping in mind people's differences in personal experiences, the client with a mild stroke without long-term deficits, the client who had a seizure or the young woman who sustained a minor head injury are generally at a lower risk of depression.

Which pt with the highest risk for developing cancer of the larynx should be alerted about relevant lifestyle modifications to decrease this risk? a. 57-year-old male with alcoholism b. 18-year-old marijuana smoker c. 28-year-old female with diabetes d. 34-year-old male who snorts cocaine

a

Question 9 of 29 The spouse of the client with Alzheimer's disease is listening to the hospice nurse explaining the client's drug regimen. Which statement by the spouse indicates an understanding of the nurse's instruction? a. "Donepezil (Aricept) will treat the symptoms of Alzheimer's disease." b. "Memantine (Namenda) is indicated for treatment of early symptoms of Alzheimer's disease. c. "Rivastigmine (Exelon) is used to treat depression." d. "Sertraline (Zoloft) will treat the symptoms of Alzheimer's disease."

a "Donepezil (Aricept) will treat the symptoms of Alzheimer's disease." Cholinesterase inhibitors (e.g., donepezil) are approved for the symptomatic treatment of Alzheimer's disease. Memantine (Namenda) is indicated for advanced Alzheimer's disease. Rivastigmine (Exelon) is a cholinesterase inhibitor that is used to treat Alzheimer's symptoms. Selective serotonin reuptake inhibitors are antidepressants and may be used in Alzheimer's clients who develop depression. Some clients with Alzheimer's disease experience depression and may be treated with antidepressants such as sertraline.

Question 14 of 29 A client has been diagnosed with Huntington disease (HD). The nurse is teaching the client and her parents about the genetic aspects of the disease. Which statement made by the parents demonstrates a good understanding of the nurse's teaching? a. "If she has children, she'll pass the gene on to her kids." b. "She could only have gotten the disease from both of us." c. "Because she got the gene from her father, she'll live longer than others with HD." d. "More testing should definitely be done to see if she's really got the gene."

a "If she has children, she'll pass the gene on to her kids." An autosomal dominant trait with high penetrance, such as HD, means that a person who inherits just one mutated allele has an almost 100% chance of developing the disease. Only one defective gene is needed to inherit HD. The client could have inherited it from her father or mother. If the client inherited the gene from her mother, she would live a longer life than other people with the disease. If she inherited the gene from her father, her life would be shorter. Additional testing is not necessary. If the client has HD, then the client has the gene.

Question 26 of 29 The parents of a young child report that their child sometimes stares blankly into space for just a few seconds and then gets very tired. The nurse anticipates that the child will be assessed for which seizure disorder? a. Absence b. Myoclonic c. Simple partial d. Tonic

a Absence Absence seizures are more common in children and consist of brief (often just seconds) periods of loss of consciousness and blank staring, as though he or she is daydreaming. Myoclonic seizures are characterized by brief jerking or stiffening of the extremities, which may occur singly or in groups. Partial seizures are most often seen in adults. Tonic seizures are characterized by an abrupt increase in muscle tone, loss of consciousness, and autonomic changes lasting from 30 seconds to several minutes.

Question 23 of 29 A client newly diagnosed with Parkinson disease is being discharged. Which instruction is best for the nurse to provide to the client's spouse? a. Administer medications promptly on schedule to maintain therapeutic drug levels. b. Complete activities of daily living for the client. c. Speak loudly for better understanding. d. Provide high-calorie, high-carbohydrate foods to maintain the client's weight.

a Administer medications promptly on schedule to maintain therapeutic drug levels. Administering medications promptly on schedule is a correct statement. The client should be encouraged to do as much as possible on his own. Slow speech rather than loud speech, and small, frequent meals are more effective for the client with Parkinson disease.

Question 10 of 29 A client with a migraine is lying in a darkened room with a wet cloth on the head after receiving analgesic drugs. What does the nurse do next? a. Allow the client to remain undisturbed. b. Assess the client's vital signs. c. Remove the cloth because it can harbor microorganisms. d. Turn on the lights for a neurologic assessment.

a Allow the client to remain undisturbed. At the beginning of a migraine attack, the client may be able to alleviate pain with analgesics and by lying down in a darkened room with a cool cloth on his or her forehead. If the client falls asleep, he or she should remain undisturbed until awakening. Assessing the client's vital signs will disturb the client unnecessarily. A cool cloth is helpful for the client with a migraine and does not present enough of a risk that it should be removed. Turning on the lights for a neurologic assessment is not appropriate because light can cause the migraine to worsen.

Question 27 of 29 The wife of a client with Alzheimer's disease mentions to the home health nurse that, although she loves him, she is exhausted caring for her husband. What does the nurse do to alleviate caregiver stress? a. Arranges for respite care b. Provides positive reinforcement and support to the wife c. Restrains the client for a short time each day, to allow the wife to rest d. Teaches the client improved self-care

a Arranges for respite care Respite care can give the wife some time to re-energize and will provide a social outlet for the client. Providing positive reinforcement and support is encouraging, but does not help the wife's situation. Restraints are almost never appropriate and are used only as an absolute last resort. The client with Alzheimer's disease typically is unable to learn improved self-care.

Question 8 of 29 A client is admitted with bacterial meningitis. Which nursing intervention is the highest priority for this client? a. Assessing neurologic status at least every 2 to 4 hours b. Decreasing environmental stimuli c. Managing pain through drug and nondrug methods d. Strict monitoring of hourly intake and output

a Assessing neurologic status at least every 2 to 4 hours The most important nursing intervention for clients with meningitis is the accurate monitoring and recording of their neurologic status, vital signs, and vascular assessment. The client's neurologic status and vital signs should be assessed at least every 4 hours, or more often if clinically indicated. The priority for care is to monitor for early neurologic changes that may indicate increased intracranial pressure, such as decreased level of consciousness. Decreasing environmental stimuli is helpful for the client with bacterial meningitis, but is not the highest priority. Clients with bacterial meningitis report severe headaches requiring pain management, but this is the second-highest priority. Assessing fluid balance while preventing overload is not the highest priority.

Question 2 of 29 A client is being discharged to home with progressing stage I Alzheimer's disease. The family expresses concern to the nurse about caring for their parent. What is the priority for best continuity of care? a. Assigning a case manager b. Ensuring that all family questions are answered before discharge c. Providing a safe environment d. Referring the family to the Alzheimer's Association

a Assigning a case manager Whenever possible, the client and family should be assigned a case manager who can assess their needs for health care resources and facilitate appropriate placement throughout the continuum of care. Ensuring all questions are answered and providing a safe environment are necessary for family support, but are not relevant for continuity of care. Referring the family to the Alzheimer's Association is necessary for appropriate resource referral, but is not relevant for continuity of care.

Question 11 of 29 The nurse is reviewing the history of a client who has been prescribed topiramate (Topamax) for treatment of intractable partial seizures. The nurse plans to contact the health care provider if the client has which condition? a. Bipolar disorder b. Diabetes mellitus c. Glaucoma d. Hypothyroidism

a Bipolar disorder Cases of suicide have been associated with topiramate when it is used in larger doses of 400 mg daily, most often in clients with bipolar disorder. Topiramate is not contraindicated in clients with diabetes mellitus, glaucoma, or hypothyroidism.

Question 6 of 29 A client receiving sumatriptan (Imitrex) for migraine headaches is experiencing adverse effects after taking the drug. Which adverse effect is of greatest concern to the nurse? a. Chest tightness b. Skin flushing c. Tingling feelings d. Warm sensation

a Chest tightness Triptan drugs are contraindicated in clients with coronary artery disease because they can cause arterial narrowing. Clients taking triptan drugs should report angina or chest discomfort to prevent cardiac injury associated with myocardial ischemia. Skin flushing, tingling feelings, and a warm sensation are common adverse effects with triptan medications and are not indications to avoid using this group of drugs.

Question 7 of 29 Which change in the cerebrospinal fluid (CSF) indicates to the nurse that a client may have bacterial meningitis? a. Cloudy, turbid CSF b.Decreased white blood cells c. Decreased protein d. Increased glucose

a Cloudy, turbid CSF Cloudy, turbid CSF is a sign of bacterial meningitis. Clear fluid is a sign of viral meningitis. Increased white blood cells, increased protein, and decreased glucose are signs of bacterial meningitis.

Question 28 of 29 A client with Parkinson disease is being discharged home with his wife. To ensure success with the management plan, which discharge action is most effective? a. Involving the client and his wife in developing a plan of care b. Setting up visitations by a home health nurse c. Telling his wife what the client needs d. Writing up a detailed plan of care according to standards

a Involving the client and his wife in developing a plan of care Involving the client and spouse in developing a plan of care is the best way to ensure success with the management plan. Home health nurse visitations are generally helpful, but may not be needed for this client. Instructing the spouse about the client's needs and providing the spouse with a written plan of care do not reinforce the spouse's involvement and buy-in with the management plan.

Question 22 of 29 A client with early-stage Alzheimer's disease is admitted to the surgical unit for a biopsy. Which client problem is the priority? a. Potential for injury related to chronic confusion and physical deficits b. Risk for reduced mobility related to progression of disability c. Potential for skin breakdown related to immobility and/or impaired nutritional status d. Lack of social contact related to personality and behavior changes

a Potential for injury related to chronic confusion and physical deficits The priority for interdisciplinary care is safety. Chronic confusion and physical deficits place the client with Alzheimer's disease at high risk for injury. The rest of the problems are usually the result of long-term care and not a priority for a short hospital stay.

The pt with laryngeal cancer that is being treated with radiation is experiencing hoarseness. What teaching points must the nurse stress with this pt? SELECT ALL THAT APPLY a. your voice should improve within 4-6 weeks after the radiation therapy is completed b. typically the hoarseness becomes worse during the radiation therapy c. gargle 4-6x/day with an alcohol based mouthwash d. rest your voice and use alternative communication methods during radiation therapy e. wash your neck 3X/day with a strong antiseptic soap f. the speech therapist can offer options for alternative communication methods

a b d f

The pt with a total laryngectomy has a laryngectomy button in place. What important teaching points must the nurse include when teaching the pt about a laryngectomy button? SELECT ALL THAT APPLY a. a laryngectomy button is shorter and softer than a laryngectomy tube b. a laryngectomy button comes in only one size c. a laryngectomy button is more comfortable than a laryngectomy tube d. a laryngectomy button requires use of an alternative form of communication e. a laryngectomy button requires the use of sterile procedure when it is changed f. a laryngectomy button is longer and narrow like an endotracheal tube

a c d

Packing has been removed from a pt with epistaxis. Which discharge instructions would the nurse be sure to teach the pt and his family? SELECT ALL THAT APPLY a. use saline spray to add moisture and prevent rebleeding b. use lots of petroleum jelly to coat the inside and outside of the nasal passages for comfort c. avoid vigorous nose blowing d. do not take aspirin containing products/NSAIDS e. no strenuous lifting for at least 1 month f. consume only small meal for 2 weeks

a c d e

A pt with facial trauma has undergone surgical intervention to wire the jaw shut. In performing discharge teaching with this pt, which topics does the nurse cover? SELECT ALL THAT APPLY a. oral care b. activity c. use of wire cutters d. communication e. aspiration prevention f. dental liquid diet

a c d e f

Following radiation therapy for head and neck cancer, the nurse instructs the pt about which potential side effects? SELECT ALL THAT APPLY a. skin redness and tenderness b. numbness of the mouth, lips, or face c. difficulty swallowing d. hoarseness e. dry mouth f. impaired taste

a c d e f

The nurse is interviewing a pt to assess for risk factors related to head and neck cancer. Which questions are appropriate to include? SELECT ALL THAT APPLY a. how many servings/day of alcohol do you typical drink? b. have you had frequent episodes of acute/chronic visual problems? c. have you had a problem with sores in your mouth? d. when was the last time you saw your dentist? e. do you have recurrent laryngitis/frequent episodes of sore throat? f. how many packs/day do you smoke and for how many years?

a c e f

Which side effects would a pt with obstructive sleep apnea report? SELECT ALL THAT APPLY a. excessive daytime sleepiness b. excessive daytime hyperactivity c. inability to concentrate d. excessive production of sputum e. irritability f. heavy snoring

a c e f

A pt has had an inner maxillary fixation for a mandibular fracture. Which piece of equipment should be kept at the bedside at ALL times? a. waterpik b. wire cutters c. pair of hemostats d. emesis basin

b

Question 12 of 29 The nurse is teaching a client newly diagnosed with migraines about trigger control. Which statement made by the client demonstrates good understanding of the teaching plan? a. "I can still eat Chinese food." b. "I must not miss meals." c. "It is okay to drink a few wine coolers." d. "I need to use fake sugar in my coffee."

b "I must not miss meals." Missing meals is a trigger for many people suffering from migraines. The client should not skip any meals until the triggers are identified. Monosodium glutamate-containing foods, alcohol, and artificial sweeteners are triggers for many people suffering from migraines and should be eliminated until the triggers are identified.

Question 16 of 29 A female client with newly diagnosed migraines is being discharged with a prescription for sumatriptan (Imitrex). Which comment by the client indicates an understanding of the nurse's discharge instructions? a. "Sumatriptan should be taken as a last resort." b. "I must report any chest pain right away." c. "Birth control is not needed while taking sumatriptan." d. "St. John's wort can also be taken to help my symptoms."

b "I must report any chest pain right away." Chest pain must be reported immediately with the use of sumatriptan. Sumatriptan must be taken as soon as migraine symptoms appear. Remind the client to use contraception (birth control) while taking the drug because it may not be safe for women who are pregnant. Triptans should not be taken with selective serotonin reuptake inhibitors or St. John's wort, an herb used commonly for depression.

Question 13 of 29 The nurse is caring for a client with advanced Alzheimer's disease. Which communication technique is best to use with this client? a. Providing the client with several options to choose from b. Assuming that the client is not totally confused c. Waiting for the client to express a need d. Writing down instructions for the client

b Assuming that the client is not totally confused Never assume that the client with Alzheimer's is totally confused and cannot understand what is being communicated. Choices should be limited; too many choices cause frustration and increased confusion in the client. Rather than waiting for the client to express a need, try to anticipate the client's needs and interpret nonverbal communication. Rather than writing down instructions, provide the client instructions with pictures, and put them in a highly visible place.

Question 4 of 29 Which is the most effective way for a college student to minimize the risk for bacterial meningitis? a. Avoid large crowds. b. Get the meningococcal vaccine. c. Take a daily vitamin. d. Take prophylactic antibiotics.

b Get the meningococcal vaccine. Individuals ages 16 to 21 years have the highest rates of meningococcal infection and should be immunized against the virus. Adults are advised to get an initial or booster vaccine if living in a shared residence (residence hall, military barracks, group home), traveling or residing in countries in which the disease is common, or immunocompromised due to a damaged or surgically removed spleen or a serum complement deficiency. Avoiding large crowds is helpful, but is not practical for a college student. Taking a daily vitamin is helpful, but is not the best way to safeguard against bacterial meningitis. Taking prophylactic antibiotics is inappropriate because it leads to antibiotic-resistant strains of microorganisms.

A pt has demonstrated anxiety since a diagnosis of neck cancer. The surgery and radiation therapy are completed. Which behavior indicates that a pt's fears are decreasing? a. repeatedly asks the same questions and seeks to re-validate all info b. states that he is less anxious but is irritable and tense whenever questioned c. makes a plan to contact the American Cancer Society Visitor Program d. makes a plan to share personal belongings with friends and family

c

A pt has sustained a mandible fracture and the surgeon has explained that the repair will be made using a resorbable plate. The pt discloses to the nurse that he has not told the surgeon about his substance abuse and illicit drug dependence. What is the nurse's BEST response? a. why didn't you talk to the surgeon about this issue? b. you should tell the surgeon, but it's your choice c. it is important for your surgeon to know about this info d. you shouldn't be ashamed; your surgeon will still repair your fracture

c

A pt returns from surgery following a rhinoplasty. The UAP places the pt in a supine position to encourage rest and sleep. Which actions should the nurse take FIRST? a. teach the pt how to use the bed controls to position herself b. explain the purpose of the semi-Fowler's position to the nursing assistant c. place the pt in a semi-Fowler's position and assess for aspiration d. post a notice at the head of the bed to remind personnel about positioning

c

A pt who was in a motor vehicle accident and sustained laryngeal trauma is being treated in the ED with humidified O2 and is being monitored every 15-30 min for respiratory distress. Which assessment finding indicates the URGENT need for further intervention? a. RR 24; PaO2 80-100; no difficulty with communication b. pulse ox 96%; anxious, fatigued, blood in sputum, abdominal breathing c. confused and disoriented, difficulty producing sounds, pulse ox 80% d. anxious, RR 30, talking rapidly about the accident, warm to touch

c

Question 5 of 29 The nursing instructor asks the student nurse caring for a client with Alzheimer's disease who has been prescribed donepezil (Aricept) how the drug works. Which response by the nursing student best explains the action of donepezil? a. "The reuptake of serotonin is blocked." b. "Donepezil prevents the increase in the protein beta amyloid." c. "It delays the destruction of acetylcholine by acetylcholinesterase." d. "Dopamine levels are increased."

c "It delays the destruction of acetylcholine by acetylcholinesterase." By delaying the destruction of acetylcholine, donepezil improves cholinergic neurotransmission in the central nervous system, thus delaying the onset of cognitive decline in some clients. Donepezil is not a serotonin reuptake inhibitor. It is a cholinesterase inhibitor and does not work on the protein beta amyloid, nor does it work on dopamine receptors.

Question 19 of 29 The home health nurse is checking in on a client with dementia and the client's spouse. The spouse confides to the nurse, "I am so tired and worn out." What is the nurse's best response? a. "Can't you take care of your spouse?" b. "Establishing goals and a daily plan can help." c. "Make sure you take some time off and take care of yourself too." d. "That's not a very nice thing to say."

c "Make sure you take some time off and take care of yourself too." This response is supportive and reminds the spouse that he or she cannot care for the client when exhausted. Of course, further assessment and planning will be necessary. Questioning the spouse's ability to provide care is not supportive and may offend the spouse. Establishing goals and a daily plan is not a helpful response. A better response would be, "Take one day at a time." Suggesting that the spouse's comment was not nice is judgmental and inappropriate.

Question 1 of 29 A client admitted with cerebral edema suddenly begins to have a seizure while the nurse is in the room. What does the nurse do first? a. Administer phenytoin (Dilantin). b. Draw the client's blood. c. Assess the need for additional support. d. Start an intravenous (IV) line.

c Assess the need for additional support. The primary goal is to assess the client for the need of additional support during the seizure. Interventions to protect the client from injury, turning the client on the side, and monitoring the client are indicated. After a quick assessment by the nurse, the health care provider must be notified immediately, and intubation by an anesthesiologist, nurse anesthetist, or respiratory therapist may be necessary. Phenytoin (Dilantin) is administered to prevent the recurrence of seizures, not to treat a seizure already underway. Drawing blood or starting an IV is not the priority in this situation.

Question 3 of 29 A client is admitted into the emergency department with frontal-temporal pain, preceded by a visual disturbance. The client is upset and thinks it is a stroke. What does the nurse suspect may be occurring? a. Stroke b. Tension headache c. Classic migraine d. Cluster headache

c Classic migraine The client's symptoms match those of a classic migraine. Symptoms of a stroke include sudden, severe headache with unknown cause, facial drooping, sudden confusion, and sudden difficulty walking or standing. A tension headache is characterized by neck and shoulder muscle tenderness and bilateral pain at the base of the skull and in the forehead. Symptoms of a cluster headache include intense, unilateral pain occurring in the fall or spring and lasting 30 minutes to 2 hours.

A pt with a recent diagnosis of sinus cancer states that he wants another course of ABX because he believes he simply has another sinus infection. What is the nurse's BEST response? a. I'll call the physician for the antibiotic prescription b. why are you doubting your doctor's diagnosis? c. let me bring you some info about sinus cancer d. what did the dr. say to you about your condition?

d

An older adult pt who is talking and laughing while eating begins to choke on a piece of meat. What is the INITIAL emergency management for this pt? a. several sharp blows between the scapula b. call the RRT c. nasotracheal suctioning d. abdominal thrusts (Heimlich maneuver)

d

The nurse is caring for a pt with a nasal fracture. The pt has clear secretions that react positively when tested for glucose. Which complication does the nurse suspect? a. jaw fracture b. facial fracture c. vertebral fracture d. skull fracture

d

Question 15 of 29 A client has Parkinson disease (PD). Which nursing intervention best protects the client from injury? a. Discouraging the client from activity b. Encouraging the client to watch the feet when walking c. Suggesting that the client obtain assistance in performing activities of daily living (ADLs) d. Monitoring the client's sleep patterns

d Monitoring the client's sleep patterns Clients with PD tend to not sleep well at night because of drug therapy and the disease itself. Some clients nap for short periods during the day and may not be aware that they have done so. This sleep misperception could put the client at risk for injury (e.g., falling asleep while driving). Active and passive range-of-motion exercises, muscle stretching, and activity are important to keep the client with PD mobile and flexible. The client with PD should avoid watching his or her feet when walking to prevent falls and should be encouraged to participate as much as possible in self-management, including ADLs. Occupational and physical therapists can provide training in ADLs and the use of adaptive devices, as needed, to facilitate independence.

The nurse is about to administer a contrast medium to the client undergoing diagnostic testing. Which question does the nurse first ask the client? "Are you in pain?" "Are you taking ibuprofen daily" "Are you wearing any metal?" "Do you know what this test is for?"

"Are you taking ibuprofen daily" The first question the nurse asks is if the client uses Ibuprofen on a daily basis. Ibuprofen is an NSAID, and daily use may place the client's renal function at risk. The client would also be asked about allergies to contrast agents, daily use of Metformin, and any conditions that may compromise kidney function.Inquiring if a client is in pain is always part of nursing assessment but would not be the first question to ask. The nurse would use this opportunity for education to confirm the client knows the reason for the test and take this time to answer any questions. Diagnostic testing involving magnetic resonance imaging, not contrast medium, requires precautions around metal objects.

A client with chronic kidney disease asks the nurse about the relationship between the disease and high blood pressure. What is the nurse's best response? "Because the kidneys cannot get rid of fluid, blood pressure goes up." "The damaged kidneys no longer release a hormone that prevents high blood pressure." "The waste products in the blood interfere with other mechanisms that control blood pressure." "This is a compensatory mechanism that increases blood flow through the kidneys in effort to get rid of some of the waste products."

"Because the kidneys cannot get rid of fluid, blood pressure goes up." In chronic kidney disease, fluid levels increase in the circulatory system. The statements asserting that damaged kidneys no longer release a hormone to prevent high blood pressure, waste products in the blood interfere with other mechanisms controlling blood pressure, and high blood pressure is a compensatory mechanism that increases blood flow through the kidneys in attempt excrete waste products are not accurate regarding the relationship between chronic kidney disease and high blood pressure.

The nurse is preparing a client with possible pulmonary embolism for a CT scan with contrast. Prior to the scan, which of these assessment questions is essential for the nurse to ask? "Do you take supplements containing vitamin K?" "Did you take metformin today?" "Are you allergic to peanuts?" "Have you had shortness of breath recently?"

"Did you take metformin today?" The assessment question that is essential for the nurse to ask is, "Did you take metformin today?" IV contrast material can be nephrotoxic. Metformin is stopped at least 24 hours before contrast dye is used and is not restarted until adequate kidney function is confirmed.If pulmonary embolism is confirmed, warfarin may be prescribed. If so, vitamin K-containing foods and vitamins will need to be limited. Peanut allergy does not pose a risk with contrast. Shortness of breath is a typical finding when a PE is present, and is not the priority assessment prior to CT.

Which statement by a client with chronic obstructive pulmonary disease (COPD) indicates the need for additional follow-up instruction?

"I don't need to get a flu shot." An annual influenza vaccine (flu shot) is important for all clients with COPD. At the same time, a pneumonia vaccine could be offered, since pneumonia is one of the most common complications of COPD. The client who is hypoxemic and also has chronic hypercarbia requires lower levels of oxygen delivery, and may not need it all the time. Increased work of breathing in a client with COPD raises calorie and protein needs, which can lead to protein-calorie malnutrition. Clients with COPD often have chronic fatigue.

After receiving education on the correct use of emergency drug therapy for asthma, which statement by the client indicates a correct understanding of the nurse's instructions?

"I must have my emergency inhaler with me at all times." Because asthma attacks cannot always be predicted, clients with asthma must always carry a rescue inhaler such as a short-acting beta agonist (e.g., albuterol [Proventil]). Asthma medications are specific to the disease and should never be shared or used by anyone other than the person for whom they are prescribed. An emergency inhaler should be carried when activity is anticipated, as well as at other times. Preventive drugs are those that are taken every day to help prevent an attack from occurring. They are not able to stop an attack once it begins.

A client has been diagnosed with asthma. Which statement below indicates that the client correctly understands how to use an inhaler with a spacer?

"If the spacer makes a whistling sound, I am breathing in too rapidly." Slow and deep breaths ensure that the medication is reaching deeply into the lungs. The whistling noise serves as a reminder to the client of which technique needs to be used. The client must wait 1 minute between puffs. The client should rinse the mouth but not swallow the water. The mouth needs to be rinsed after using an inhaler with or without a spacer. This is especially important if the inhaled medication is a corticosteroid, to prevent the development of an oral fungal infection. An empty inhaler will float on its side in water; a full inhaler will sink. Shaking an inhaler helps ensure that the same dose is delivered in each puff.

The nurse has taught a client about influenza infection control. Which client statement indicates the need for further teaching?

"The intranasal vaccine can be given to everybody in the family." The intranasal flu vaccine is approved for healthy clients ages 2 to 49 who are not pregnant. Washing hands frequently is the best way to prevent the spread of illnesses such as the flu. Avoiding kissing and shaking hands are two ways to prevent transmission of the flu. A new recommendation from the Centers for Disease Control and Prevention for controlling the spread of the flu is to sneeze or cough into the upper sleeve rather than into the hand.

During discharge teaching for a client with kidney disease, what does the nurse teach the client to do? "Drink 2 liters of fluid and urinate at the same time every day." "Eat breakfast and go to bed at the same time every day." "Check your blood sugar and do a urine dipstick test." "Weigh yourself and take your blood pressure."

"Weigh yourself and take your blood pressure." Regular weight assessment monitors fluid restriction control, while blood pressure control is necessary to reduce cardiovascular complications and slow the progression of kidney dysfunction. Fluid intake and urination, and breakfast time and bedtime, do not need to be at the same time each day. Clients with diabetes, not kidney disease, should regularly check their blood sugar and perform a urine dipstick test.

42. A patient with a history of bronchitis for more than 20 years is hospitalized. With this patient's history, what is a potential complication? A. Right-sided heart failure B. Left-sided heart failure C. Renal disease D. Stroke

A

Which clinical manifestation in a client with pyelonephritis indicates that treatment has been effective? Decreased urine output Decreased white blood cells in urine Increased red blood cell count Increased urine specific gravity

A decreased presence of white blood cells in the urine indicates the eradication of infection. A decreased urine output, an increased red blood cell count, and increased urine specific gravity are not symptoms of pyelonephritis.

A 70-year-old client has a complicated medical history, including chronic obstructive pulmonary disease. Which client statement indicates the need for further teaching about the disease? A) "I am here to receive the yearly pneumonia shot again." B) "I am here to get my yearly flu shot again." C) "I should avoid large gatherings during cold and flu season." D) "I should cough into my upper sleeve instead of my hand." (Chp. 31; elsevier resources)

A) "I am here to receive the yearly pneumonia shot again." (Chp. 31; elsevier resources)

An older adult resident in a long-term-care facility becomes confused and agitated, telling the nurse, "Get out of here! You're going to kill me!" Which action will the nurse take first? A) Check the resident's oxygen saturation. B) Do a complete neurologic assessment. C) Give the prescribed PRN lorazepam (Ativan). D) Notify the resident's primary care provider. (Chp. 31; elsevier resources)

A) Check the resident's oxygen saturation. (Chp. 31; elsevier resources)

A client is being admitted for pneumonia. The sputum culture is positive for streptococcus, and the client asks about the length of the treatment. On what does the nurse base the answer? A) The client will be treated for 5 to 7 days. B) The client will require IV antibiotics for 7 to 10 days. C) The client will complete 6 days of therapy. D) The client must be afebrile for 24 hours. (Chp. 31; elsevier resources)

A) The client will be treated for 5 to 7 days. (Chp. 31; elsevier resources)

23. In obtaining a history for a patient with chronic airflow limitation, which risk factors are related to potentially causing or triggering the disease process? (Select all that apply.) A. Cigarette smoking B. Occupational and air pollution C. Genetic tendencies D. Smokeless tobacco E. Occupation

A, B, C, E

34. The patient with COPD is taking systemic theophylline. What specific precautions must the nurse use when caring for this patient? (Select all that apply.) A. Monitor serum theophylline levels. B. Alert the healthcare provider for any abnormal values. C. Administer the drug using a metered-dose inhaler (MDI). D. Assess the patient for adverse reactions related to a toxic level. E. Monitor the patient's heart rate.

A, B, D, E

6. A patient with chronic obstructive pulmonary disease (COPD) is likely to have which findings on assessment? (Select all that apply.) A. Increased anteroposterior (AP) diameter of the chest B. Sitting in a chair leaning forward with elbows on knees C. Unintentional weight gain D. Decreased appetite E. Unexplained weight loss

A, B, D, E

2. Which are characteristics of asthma? (Select all that apply.) A. Narrowed airway lumen due to inflammation B. Increased eosinophils C. Decreased breathing cycle D. Intermittent bronchospasm E. Loss of elastic recoil F. Stimulation of disease process by allergies

A, B, D, F

31. In assisting a patient with chronic airflow limitation to relive dyspnea, which sitting positions are beneficial to the patient for breathing? (Select all that apply.) A. On edge of chair, leaning forward with arms folded and resting on a small table B. In a low semi-reclining position with the shoulders back and knees apart C. Forward in a chair with feet spread apart and elbows placed on the knees D. Head slightly flexed, with feet spread apart and shoulders relaxed E. Low semi-Fowler's position with knees elevated

A, C, D

3. The nurse is caring for an older adult patient with a chronic respiratory disorder. Which interventions are best to use in caring for this patient? (Select all that apply.) A. Provide rest periods between activities such as bathing, meals, and ambulation. B. Place the patient in a supine position after meals to allow fore rest. C. Schedule drug administration around routine activities to increase adherence to drug therapy. D. Arrange chairs in strategic locations to allow the patient to walk and rest. E. Teach the patient to avoid getting the pneumococcal vaccine. F. Encourage the patient to have an annual flu vaccination.

A, C, D, F

49. The nurse is caring for a patient who has CF. Which assessment findings indicate the need for exacerbation therapy? (Select all that apply.) A. New-onset crackles B. Increased activity tolerance C. Increased frequency of coughing D. Increased chest congestion E. Increased SaO2 F. At least a 10% decrease in FEV1

A, C, D, F

56. Which of the following may be warnings signs of lung cancer? (Select all that apply.) A. Dyspnea B. Dark yellow-colored sputum C. Persistent cough or change in cough D. Abdominal pain and frequent stools E. Recurring episodes of pleural effusion

A, C, E

71. The nurse is providing discharge instructions to a patient with pulmonary fibrosis and the family. What instructions are appropriate for this patient's diagnosis? (Select all that apply.) A. Using home oxygen B. Maintaining activity level as before C. Preventing respiratory infections D. Limiting fluid intake E. Energy conservation measures

A, C, E

10. A patient with asthma is repeatedly not compliant with the medication regimen, which has resulted in the patient being hospitalized for a severe asthma attack. Which interventions does the nurse suggest to help the patient manage asthma on a daily basis? (Select all that apply.) A. Encourage active participation in the plan of care. B. Help the patient develop a flexible plan of care. C. have the pharmacist establish a plan of care. D. Teach the patient about asthma and its treatment plan. E. Assess symptom severity using a peak flowmeter 1 to 2 times per week.

A, D

1. Which of the following are characteristics of pulmonary emphysema? (Select all that apply.) A. Decreased surface area of alveoli B. Chronic thickening of bronchial walls C. Decreased respiratory rate D. Hypercapnia E. Arterial blood gases (ABGs) show chronic respiratory acidosis F. Increased eosinophils

A, D, E

The nurse is reviewing the history of a client who has been prescribed topiramate (Topamax) for treatment of intractable partial seizures. The nurse plans to contact the health care provider if the client has which condition? A. Bipolar disorder B. Diabetes mellitus C. Glaucoma D. Hypothyroidism

A. Bipolar disorder Rationale A. Cases of suicide have been reported, most often in clients with bipolar disorder. B. Topiramate is not contraindicated in clients with diabetes mellitus. C. Topiramate is not contraindicated in clients with glaucoma. D. Topiramate is not contraindicated in clients with hypothyroidism.

Which of the components of a client's family history are of particular importance to the home health nurse who is assessing a new client with asthma? A. Brother is allergic to peanuts. B. Father is obese. C. Mother is diabetic. D. Sister is pregnant.

A. Brother is allergic to peanuts. Correct: Clients with asthma often have a family history of allergies. It will be important to assess whether this client has any allergies that may serve as triggers for an asthma attack.

1. An emergency department nurse cares for a client who is severely dehydrated and is prescribed 3 L of intravenous fluid over 6 hours. At what rate (mL/hr) should the nurse set the intravenous pump to infuse the fluids? (Record your answer using a whole number.) ____ mL/hr

ANS: 500 mL/hr Because IV pumps deliver in units of milliliters per hour, the pump would have to be set at 500 mL/hr to deliver 3 L (3000 mL) over 6 hours. 6x = 3000 x = 500

A nurse cares for a client with diabetes mellitus who is visually impaired. The client asks, "Can I ask my niece to prefill my syringes and then store them for later use when I need them?" How should the nurse respond? a. "Yes. Prefilled syringes can be stored for 3 weeks in the refrigerator in a vertical position with the needle pointing up." b. "Yes. Syringes can be filled with insulin and stored for a month in a location that is protected from light." c. "Insulin reacts with plastic, so prefilled syringes are okay, but you will need to use glass syringes." d. "No. Insulin syringes cannot be prefilled and stored for any length of time outside of the container."

ANS: A Insulin is relatively stable when stored in a cool, dry place away from light. When refrigerated, prefilled plastic syringes are stable for up to 3 weeks. They should be stored in the refrigerator in the vertical position with the needle pointing up to prevent suspended insulin particles from clogging the needle.

A nurse auscultates a client's lung fields. Which pathophysiologic process should the nurse associate with this breath sound? (Click the media button to hear the audio clip.) a. Inflammation of the pleura b. Constriction of the bronchioles c. Upper airway obstruction d. Pulmonary vascular edema

ANS: A A pleural friction rub can be heard when the pleura is inflamed and rubbing against the lung wall. The other pathophysiologic processes would not cause a pleural friction rub. Constriction of the bronchioles may be heard as a wheeze, upper airway obstruction may be heard as stridor, and pulmonary vascular edema may be heard as crackles.

After teaching a client who is recovering from a vertebroplasty, the nurse assesses the client's understanding. Which statement by the client indicates a need for additional teaching? a. "I can drive myself home after the procedure." b. "I will monitor the puncture site for signs of infection." c. "I can start walking tomorrow and increase my activity slowly." d. "I will remove the dressing the day after discharge."

ANS: A Before discharge, a client who has a vertebroplasty should be taught to avoid driving or operating machinery for the first 24 hours. The client should monitor the puncture site for signs of infection. Usual activities can resume slowly, including walking and slowly increasing activity over the next few days. The client should keep the dressing dry and remove it the next day.

The client's chart indicates genu varum. What does the nurse understand this to mean? a. Bow-legged b. Fluid accumulation c. Knock-kneed d. Spinal curvature

ANS: A Genu varum is a bow-legged deformity. A fluid accumulation is an effusion. Genu valgum is knock-kneed. A spinal curvature could be kyphosis or lordosis.

A nurse contacts the health care provider after reviewing a client's laboratory results and noting a blood urea nitrogen (BUN) of 35 mg/dL and a creatinine of 1.0 mg/dL. For which action should the nurse recommend a prescription? a. Intravenous fluids b. Hemodialysis c. Fluid restriction d. Urine culture and sensitivity

ANS: A Normal BUN is 10 to 20 mg/dL. Normal creatinine is 0.6 to 1.2 mg/dL (males) or 0.5 to 1.1 mg/dL (females). Creatinine is more specific for kidney function than BUN, because BUN can be affected by several factors (dehydration, high-protein diet, and catabolism). This client's creatinine is normal, which suggests a non-renal cause for the elevated BUN. A common cause of increased BUN is dehydration, so the nurse should anticipate giving the client more fluids, not placing the client on fluid restrictions. Hemodialysis is not an appropriate treatment for dehydration. The lab results do not indicate an infection

A nurse provides diabetic education at a public health fair. Which disorders should the nurse include as complications of diabetes mellitus? (Select all that apply.) a. Stroke b. Kidney failure c. Blindness d. Respiratory failure e. Cirrhosis

ANS: A, B, C Complications of diabetes mellitus are caused by macrovascular and microvascular changes. Macrovascular complications include coronary artery disease, cerebrovascular disease, and peripheral vascular disease. Microvascular complications include nephropathy, retinopathy, and neuropathy. Respiratory failure and cirrhosis are not complications of diabetes mellitus.

A nurse cares for a client with a fracture injury. Twenty minutes after an opioid pain medication is administered, the client reports pain in the site of the fracture. Which actions should the nurse take? (Select all that apply.) a. Administer additional opioids as prescribed. b. Elevate the extremity on pillows. c. Apply ice to the fracture site. d. Place a heating pad at the site of the injury. e. Keep the extremity in a dependent position.

ANS: A, B, C The client with a new fracture likely has edema; elevating the extremity and applying ice probably will help in decreasing pain. Administration of an additional opioid within the dosage guidelines may be ordered. Heat will increase edema and may increase pain. Dependent positioning will also increase edema.

A client has been diagnosed with an empyema. What interventions should the nurse anticipate providing to this client? (Select all that apply.) a. Assisting with chest tube insertion b. Facilitating pleural fluid sampling c. Performing frequent respiratory assessment d. Providing antipyretics as needed e. Suctioning deeply every 4 hours

ANS: A, B, C, D The client with an empyema is often treated with chest tube insertion, which facilitates obtaining samples of the pleural fluid for analysis and re-expands the lungs. The nurse should perform frequent respiratory system assessments. Antipyretic medications are also used. Suction is only used when needed and is not done deeply to prevent tissue injury.

A client is suspected to have muscular dystrophy. About what diagnostic testing does the nurse educate the client? (Select all that apply.) a. Electromyography b. Muscle biopsy c. Nerve conduction studies d. Serum aldolase e. Serum creatinine kinase

ANS: A, B, D, E Diagnostic testing for muscular dystrophy includes electromyography, muscle biopsy, serum aldolase and creatinine kinase levels. Nerve conduction is not related to this disorder.

A student nurse learns that the spleen has several functions. What functions do they include? (Select all that apply.) a. Breaks down hemoglobin b. Destroys old or defective red blood cells (RBCs) c. Forms vitamin K for clotting d. Stores extra iron in ferritin e. Stores platelets not circulating

ANS: A, B, E Functions of the spleen include breaking down hemoglobin released from RBCs, destroying old or defective RBCs, and storing the platelets that are not in circulation. Forming vitamin K for clotting and storing extra iron in ferritin are functions of the liver.

A nurse teaches a client with diabetes mellitus about foot care. Which statements should the nurse include in this client's teaching? (Select all that apply.) a. "Do not walk around barefoot." b. "Soak your feet in a tub each evening." c. "Trim toenails straight across with a nail clipper." d. "Treat any blisters or sores with Epsom salts." e. "Wash your feet every other day."

ANS: A, C Clients who have diabetes mellitus are at high risk for wounds on the feet secondary to peripheral neuropathy and poor arterial circulation. The client should be instructed to not walk around barefoot or wear sandals with open toes. These actions place the client at higher risk for skin breakdown of the feet. The client should be instructed to trim toenails straight across with a nail clipper. Feet should be washed daily with lukewarm water and soap, but feet should not be soaked in the tub. The client should contact the provider immediately if blisters or sores appear and should not use home remedies to treat these wounds.

A nursing student studying the musculoskeletal system learns about important related hormones. What information does the student learn? (Select all that apply.) a. A lack of vitamin D can lead to rickets. b. Calcitonin increases serum calcium levels. c. Estrogens stimulate osteoblastic activity. d. Parathyroid hormone stimulates osteoclastic activity. e. Thyroxine stimulates estrogen release.

ANS: A, C, D Vitamin D is needed to absorb calcium and phosphorus. A deficiency of vitamin D can lead to rickets. Estrogen stimulates osteoblastic activity. Parathyroid hormone stimulates osteoclastic activity. Calcitonin decreases serum calcium levels when they get too high. Thyroxine increases the rate of protein synthesis in all tissue types.

A nurse assesses a client who is experiencing diabetic ketoacidosis (DKA). For which manifestations should the nurse monitor the client? (Select all that apply.) a. Deep and fast respirations b. Decreased urine output c. Tachycardia d. Dependent pulmonary crackles e. Orthostatic hypotension

ANS: A, C, E DKA leads to dehydration, which is manifested by tachycardia and orthostatic hypotension. Usually clients have Kussmaul respirations, which are fast and deep. Increased urinary output (polyuria) is severe. Because of diuresis and dehydration, peripheral edema and crackles do not occur.

A nurse teaches a client about self-monitoring of blood glucose levels. Which statement should the nurse include in this client's teaching to prevent bloodborne infections? a. "Wash your hands after completing each test." b. "Do not share your monitoring equipment." c. "Blot excess blood from the strip with a cotton ball." d. "Use gloves when monitoring your blood glucose."

ANS: B Small particles of blood can adhere to the monitoring device, and infection can be transported from one user to another. Hepatitis B in particular can survive in a dried state for about a week. The client should be taught to avoid sharing any equipment, including the lancet holder. The client should be taught to wash his or her hands before testing. The client would not need to blot excess blood away from the strip or wear gloves.

A nurse teaches a client with diabetes mellitus about sick day management. Which statement should the nurse include in this client's teaching? a. "When ill, avoid eating or drinking to reduce vomiting and diarrhea." b. "Monitor your blood glucose levels at least every 4 hours while sick." c. "If vomiting, do not use insulin or take your oral antidiabetic agent." d. "Try to continue your prescribed exercise regimen even if you are sick."

ANS: B When ill, the client should monitor his or her blood glucose at least every 4 hours. The client should continue taking the medication regimen while ill. The client should continue to eat and drink as tolerated but should not exercise while sick.

An older client's serum calcium level is 8.7 mg/dL. What possible etiologies does the nurse consider for this result? (Select all that apply.) a. Good dietary intake of calcium and vitamin D b. Normal age-related decrease in serum calcium c. Possible occurrence of osteoporosis or osteomalacia d. Potential for metastatic cancer or Paget's disease e. Recent bone fracture in a healing stage

ANS: B, C This slightly low calcium level could be an age-related decrease in serum calcium or could indicate a metabolic bone disease such as osteoporosis or osteomalacia. A good dietary intake would be expected to produce normal values. Metastatic cancer, Paget's disease, or healing bone fractures will elevate calcium.

A nurse assesses a client with chronic obstructive pulmonary disease. Which questions should the nurse ask to determine the client's activity tolerance? (Select all that apply.) a. "What color is your sputum?" b. "Do you have any difficulty sleeping?" c. "How long does it take to perform your morning routine?" d. "Do you walk upstairs every day?" e. "Have you lost any weight lately?"

ANS: B, C, E Difficulty sleeping could indicate worsening breathlessness, as could taking longer to perform activities of daily living. Weight loss could mean increased dyspnea as the client becomes too fatigued to eat. The color of the client's sputum would not assist in determining activity tolerance. Asking whether the client walks upstairs every day is not as pertinent as determining if the client becomes short of breath on walking upstairs, or if the client goes upstairs less often than previously.

An emergency nurse assesses a client who is admitted with a pelvic fracture. Which assessments should the nurse monitor to prevent a complication of this injury? (Select all that apply.) a. Temperature b. Urinary output c. Blood pressure d. Pupil reaction e. Skin color

ANS: B, C, E With a pelvic fracture, internal organ damage may result in bleeding and hypovolemic shock. The nurse monitors the client's heart rate, blood pressure, urine output, skin color, and level of consciousness frequently to determine whether shock is manifesting. It is important to monitor the urine for blood to assess whether the urinary system has been damaged with the pelvic fracture. Changes in temperature and pupil reactions are not directly associated with hypovolemic shock. Temperature changes are usually associated with hypo- or hyperthermia or infectious processes. Pupillary changes occur with brain injuries, bleeds, or neurovascular accidents.

A nursing student learns that many drugs can impair the immune system. Which drugs does this include? (Select all that apply.) a. Acetaminophen (Tylenol) b. Amphotericin B (Fungizone) c. Ibuprofen (Motrin) d. Metformin (Glucophage) e. Nitrofurantoin (Macrobid)

ANS: B, C, E Amphotericin B, ibuprofen, and nitrofurantoin all can disrupt the hematologic (immune) system. Acetaminophen and metformin do not.

A nurse assesses a client with an injury to the medulla. Which clinical manifestations should the nurse expect to find? (Select all that apply.) a. Loss of smell b. Impaired swallowing c. Visual changes d. Inability to shrug shoulders e. Loss of gag reflex

ANS: B, D, E Cranial nerves IX (glossopharyngeal), X (vagus), XI (accessory), and XII (hypoglossal) emerge from the medulla, as do portions of cranial nerves VII (facial) and VIII (acoustic). Damage to these nerves causes impaired swallowing, inability to shrug shoulders, and loss of the gag reflex. The other manifestations are not associated with damage to the medulla.

A nurse cares for a client who is infected with Burkholderia cepacia. Which action should the nurse take first when admitting this client to a pulmonary care unit? a. Instruct the client to wash his or her hands after contact with other people. b. Implement Droplet Precautions and don a surgical mask. c. Keep the client isolated from other clients with cystic fibrosis. d. Obtain blood, sputum, and urine culture specimens.

ANS: C Burkholderia cepacia infection is spread through casual contact between cystic fibrosis clients, thus the need for these clients to be separated from one another. Strict isolation measures will not be necessary. Although the client should wash his or her hands frequently, the most important measure that can be implemented on the unit is isolation of the client from other clients with cystic fibrosis. There is no need to implement Droplet Precautions or don a surgical mask when caring for this client. Obtaining blood, sputum, and urine culture specimens will not provide information necessary to care for a client with Burkholderia cepacia infection.

A nurse cares for a client who has developed esophagitis after undergoing radiation therapy for lung cancer. Which diet selection should the nurse provide for this client? a. Spaghetti with meat sauce, ice cream b. Chicken soup, grilled cheese sandwich c. Omelet, soft whole wheat bread d. Pasta salad, custard, orange juice

ANS: C Side effects of radiation therapy may include inflammation of the esophagus. Clients should be taught that bland, soft, high-calorie foods are best, along with liquid nutritional supplements. Tomato sauce may prove too spicy for a client with esophagitis. A grilled cheese sandwich is too difficult to swallow with this condition, and orange juice and other foods with citric acid are too caustic.

A nurse working in a geriatric clinic sees clients with "cold" symptoms and rhinitis. Which drug would be appropriate to teach these clients to take for their symptoms? a. Chlorpheniramine (Chlor-Trimeton) b. Diphenhydramine (Benadryl) c. Fexofenadine (Allegra) d. Hydroxyzine (Vistaril)

ANS: C First-generation antihistamines are not appropriate for use in the older population. These drugs include chlorpheniramine, diphenhydramine, and hydroxyzine. Fexofenadine is a second-generation antihistamine.

A nurse sees clients in an osteoporosis clinic. Which client should the nurse see first? a. Client taking calcium with vitamin D (Os-Cal) who reports flank pain 2 weeks ago b. Client taking ibandronate (Boniva) who cannot remember when the last dose was c. Client taking raloxifene (Evista) who reports unilateral calf swelling d. Client taking risedronate (Actonel) who reports occasional dyspepsia

ANS: C The client on raloxifene needs to be seen first because of the manifestations of deep vein thrombosis, which is an adverse effect of raloxifene. The client with flank pain may have had a kidney stone but is not acutely ill now. The client who cannot remember taking the last dose of ibandronate can be seen last. The client on risedronate may need to change medications.

A nurse assesses a client with diabetes mellitus 3 hours after a surgical procedure and notes the client's breath has a "fruity" odor. Which action should the nurse take? a. Encourage the client to use an incentive spirometer. b. Increase the client's intravenous fluid flow rate. c. Consult the provider to test for ketoacidosis. d. Perform meticulous pulmonary hygiene care.

ANS: C The stress of surgery increases the action of counterregulatory hormones and suppresses the action of insulin, predisposing the client to ketoacidosis and metabolic acidosis. One manifestation of ketoacidosis is a "fruity" odor to the breath. Documentation should occur after all assessments have been completed. Using an incentive spirometer, increasing IV fluids, and performing pulmonary hygiene will not address this client's problem.

A nurse assesses a client with asthma and notes bilateral wheezing, decreased pulse oxygen saturation, and suprasternal retraction on inhalation. Which actions should the nurse take? (Select all that apply.) a. Administer prescribed salmeterol (Serevent) inhaler. b. Assess the client for a tracheal deviation. c. Administer oxygen to keep saturations greater than 94%. d. Perform peak expiratory flow readings. e. Administer prescribed albuterol (Proventil) inhaler.

ANS: C, E Suprasternal retraction caused by inhalation usually indicates that the client is using accessory muscles and is having difficulty moving air into the respiratory passages because of airway narrowing. Wheezing indicates a narrowed airway; a decreased pulse oxygen saturation also supports this finding. The asthma is not responding to the medication, and intervention is needed. Administration of a rescue inhaler is indicated, probably along with administration of oxygen. The nurse would not do a peak flow reading at this time, nor would a code be called. Midline trachea is a normal and expected finding.

A nurse cares for a client with a urine specific gravity of 1.040. Which action should the nurse take? a. Obtain a urine culture and sensitivity. b. Place the client on restricted fluids. c. Assess the client's creatinine level. d. Increase the client's fluid intake.

ANS: D Normal specific gravity for urine is 1.005 to 1.030. A high specific gravity can occur with dehydration, decreased kidney blood flow (often because of dehydration), and the presence of antidiuretic hormone. Increasing the client's fluid intake would be a beneficial intervention. Assessing the creatinine or obtaining a urine culture would not provide data necessary for the nurse to make a clinical decision.

A nurse assesses a client with diabetes mellitus who self-administers subcutaneous insulin. The nurse notes a spongy, swelling area at the site the client uses most frequently for insulin injection. Which action should the nurse take? a. Apply ice to the site to reduce inflammation. b. Consult the provider for a new administration route. c. Assess the client for other signs of cellulitis. d. Instruct the client to rotate sites for insulin injection.

ANS: D The client's tissue has been damaged from continuous use of the same site. The client should be educated to rotate sites. The damaged tissue is not caused by cellulitis or any type infection, and applying ice may cause more damage to the tissue. Insulin can only be administered subcutaneously and intravenously. It would not be appropriate or practical to change the administration route.

A nurse cares for a client who had a chest tube placed 6 hours ago and refuses to take deep breaths because of the pain. Which action should the nurse take? a. Ambulate the client in the hallway to promote deep breathing. b. Auscultate the client's anterior and posterior lung fields. c. Encourage the client to take shallow breaths to help with the pain. d. Administer pain medication and encourage the client to take deep breaths.

ANS: D A chest tube is placed in the pleural space and may be uncomfortable for a client. The nurse should provide pain medication to minimize discomfort and encourage the client to take deep breaths. The other responses do not address the client's discomfort and need to take deep breaths to prevent complications.

A nurse is caring for a client who is scheduled to undergo a thoracentesis. Which intervention should the nurse complete prior to the procedure? a. Measure oxygen saturation before and after a 12-minute walk. b. Verify that the client understands all possible complications. c. Explain the procedure in detail to the client and the family. d. Validate that informed consent has been given by the client

ANS: D A thoracentesis is an invasive procedure with many potentially serious complications. Verifying that the client understands complications and explaining the procedure to be performed will be done by the physician or nurse practitioner, not the nurse. Measurement of oxygen saturation before and after a 12-minute walk is not a procedure unique to a thoracentesis.

11. A nurse teaches a client who is recovering from a nephrectomy secondary to kidney trauma. Which statement should the nurse include in this client's teaching? a. "Since you only have one kidney, a salt and fluid restriction is required." b. "Your therapy will include hemodialysis while you recover." c. "Medication will be prescribed to control your high blood pressure." d. "You need to avoid participating in contact sports like football."

ANS: D Clients with one kidney need to avoid contact sports because the kidneys are easily injured. The client will not be required to restrict salt and fluids, end up on dialysis, or have new hypertension because of the nephrectomy.

A nurse assesses clients who are at risk for diabetes mellitus. Which client is at greatest risk? a. A 29-year-old Caucasian b. A 32-year-old African-American c. A 44-year-old Asian d. A 48-year-old American Indian

ANS: D Diabetes is a particular problem among African Americans, Hispanics, and American Indians. The incidence of diabetes increases in all races and ethnic groups with age. Being both an American Indian and middle-aged places this client at highest risk.

After teaching a client who has diabetes mellitus and proliferative retinopathy, nephropathy, and peripheral neuropathy, the nurse assesses the client's understanding. Which statement made by the client indicates a correct understanding of the teaching? a. "I have so many complications, exercising is not recommended." b. "I will exercise more frequently because I have so many complications." c. "I used to run for exercise, I will start training for a marathon." d. "I should look into swimming or water aerobics to get my exercise."

ANS: D Exercise is not contraindicated for this client, although modifications based on existing pathology are necessary to prevent further injury. Swimming or water aerobics will give the client exercise without the worry of having the correct shoes or developing a foot injury. The client should not exercise too vigorously.

A client has just been admitted to the intensive care unit after having a left lower lobectomy with a video-assisted thoracoscopic surgery. Which of these requests will the nurse implement first?

Adjust oxygen flow rate to keep O2 saturation at 93% to 100% Airway and oxygenation are main priorities in the immediate postoperative period. The client will likely be intubated, so coordination of care with respiratory therapy will be important. Although antibiotic therapy may be ordered, this is not a priority at this time. Pain management in the postoperative period is important; a minimally invasive technique will be less painful than an open technique, but is still painful. Pain management is not the first priority, however. PRBCs may or may not need to be infused to maintain the oxygen-carrying capacity of the blood. Less blood is lost during minimally invasive techniques than during open surgical procedures.

After surgery for placement of a chest tube, the client reports burning in the chest. What does the nurse do first?

Assess the airway, breathing, and circulation. Assessing the "ABCs" is the priority to determine possible causes of burning in the client's chest. The client's situation does not require the Rapid Response Team to be called. The client's symptoms are not caused by a blockage of chest tubes. Listening for breath sounds would be an appropriate action for the nurse to take to evaluate the client's reported symptoms; however, this would not be the nurse's first action.

64. A patient is receiving a chemotherapy agent for lung cancer. The nurse anticipates that the patient is likely to have which common side effect? A. Diarrhea B. Nausea C. Flatulence D. Constipation

B

An older client presents to the emergency department with a 2-day history of cough, pain on inspiration, shortness of breath, and dyspnea. The client never had a pneumococcal vaccine. The client's chest x-ray shows density in both bases. The client has wheezing upon auscultation of both lungs. Would a bronchodilator be beneficial for this client? A) It would not be beneficial for this client. B) It would help decrease the bronchospasm. C) It would clear up the density in the bases of the client's lungs. D) It would decrease the client's pain on inspiration. (Chp. 31; elsevier resources)

B) It would help decrease the bronchospasm. (Chp. 31; elsevier resources)

12. Which are main purposes of asthma treatment? (Select all that apply.) A. Avoid secondhand smoke B. Improve airflow C. Relieve symptoms D. Improve exercise tolerance E. Prevent asthma episodes

B, C, E

48. A patient is undergoing diagnostic testing for possible cystic fibrosis (CF). Which nonpulmonary assessment findings does the nurse expect to observe in a patient with CF? (Select all that apply.) A. Peripheral edema B. Abdominal distention C. Steatorrhea D. Constipation E. Gastroesophageal reflux

B, C, E

Where does gas exchange occur? A. Acinus B. Alveolus C. Bronchus D. Carina

B. Alveolus Correct: The alveolus is the structural unit of the lung where gas exchange occurs.

The nurse is caring for the client with advanced Alzheimer's disease. Which communication technique is best to use with this client? A. Providing the client with several choices to choose from B. Assuming that the client is not totally confused C. Waiting for the client to express a need D. Writing down instructions for the client

B. Assuming that the client is not totally confused Rationale A. Choices should be limited. Too many choices causes frustration and increased confusion in the client. B. Never assume that the client is totally confused and cannot understand what is being communicated. C. Rather than waiting for the client to express a need, try to anticipate the client's needs and interpret nonverbal communication. D. Rather than writing down instructions, provide the client instructions with pictures, and put them in a highly visible place.

The client has been admitted with new-onset status epilepticus. Which seizure precautions does the nurse put in place? Select all that apply. A. Bite block at the bedside B. Intravenous access C. Continuous sedation D. Suction equipment at the bedside E. Siderails up

B. Intravenous access D. Suction equipment at the bedside E. Siderails up Rationale Bite blocks or padded tongue blades should not be used because the client's jaw may clench, causing teeth to break and possibly obstructing the airway.

A local hunter is admitted to the intensive care unit with a diagnosis of inhalation anthrax. Which medications does the RN anticipate the health care provider will order? A) Amoxicillin (Amoxil, Triamox) 500 mg orally every 8 hours B) Ceftriaxone (Rocephin) 2 g IV every 8 hours C) Ciprofloxacin (Cipro) 400 mg IV every 12 hours D) Pyrazinamide (Zinamide) 1000 to 2000 mg orally every day (Chp. 31; elsevier resources)

C) Ciprofloxacin (Cipro) 400 mg IV every 12 hours (Chp. 31; elsevier resources)

The client is admitted into the emergency department with frontal-temporal pain, preceded by a visual disturbance. The client is upset and thinks it is a stroke. What does the nurse suspect may be occurring? A. Stroke B. Tension headache C. Classic migraine D. Cluster headache

C. Classic migraine Rationale A. The client's symptoms do not indicate a stroke. B. The client's symptoms do not indicate a tension headache. C. The client's symptoms match those of a classic migraine. D. The client's symptoms do not indicate a cluster headache.

A client had a thoracentesis 1 day ago. He calls the home health agency and tells the nurse that he is very short of breath and anxious. What is the major concern of the nurse? A. Abscess B. Pneumonia C. Pneumothorax D. Pulmonary embolism

C. Pneumothorax Correct: A pneumothorax would be the complication of thoracentesis that causes the greatest concern, along with these symptoms.

The client with dementia and Alzheimer's disease is discharged to home. The client's daughter says, "He wanders so much, I am afraid he'll slip away from me." What resource does the nurse suggest? A. Alzheimer's Wandering Association B. National Alzheimer's Group C. Safe Return Program D. Lost Family Members Tracking Association

C. Safe Return Program Rationale C. The family should enroll the client in the Safe Return Program, a national, government-funded program of the Alzheimer's Association that assists in the identification and safe, timely return of those with dementia who wander off and become lost.

The client has a fever of 40° C. In which direction, if any, will this shift the oxyhemoglobin dissociation curve? A. Down B. To the left C. To the right D. Will not shift

C. To the right A client with fever has a higher demand for oxygen, so the curve will shift to the right for easier dissociation.

While the nurse is talking with the postoperative thoracic surgery client, the client coughs and the chest tube collection water seal chamber bubbles. What does the nurse do?

Calmly continues talking Gentle bubbling in the water seal chamber is normal during the client's exhalation, forceful cough, or position changes. Any bubbling that is occurring would stop if a kink or a blockage is present in the chest tube. The chest tube is functioning normally; there is no need to notify the health care provider. "Stripping the chest tube" greatly increases pressure inside the chest and could potentially damage lung tissue; any excessive manipulation should be avoided.

The nurse is caring for a client who is scheduled to have a transcranial Doppler (TCD). What does this diagnostic test evaluate? Cerebral vasospasm Cerebrospinal fluid Evoked potentials Intracranial pressure

Cerebral vasospasm A transcranial Doppler (TCD) is used to evaluate cerebral vasospasm or narrowing of arteries. It is noninvasive.Cerebrospinal fluid is obtained and measured during a lumbar puncture (LP). Evoked potentials measure the electrical signals in the brain during an EEG. Intracranial pressure is a measurement of blood, brain tissue, and cerebral spinal fluid and is not measured by TCD.

An older adult resident in a long-term-care facility becomes confused and agitated, telling the nurse, "Get out of here! You're going to kill me!" Which action will the nurse take first?

Check the resident's oxygen saturation. A common reason for sudden confusion in older clients is hypoxemia caused by undiagnosed pneumonia. The nurse's first action should be to assess oxygenation by checking the pulse oximetry. Determining the cause of the confusion is the primary goal of the RN. A complete neurologic examination may give the RN other indicators of the cause for the client's confusion and agitation; this will take several minutes to complete. Administering lorazepam may make the client more confused and agitated because antianxiety drugs may cause a paradoxical reaction, or opposite effect, in some older clients. Depending on the results of the client's pulse oximetry and neurologic examination, notifying the primary care provider may be an appropriate next step.

What is the greatest risk factor for lung cancer?

Cigarette smoking Cigarette smoking is the number-one risk factor for lung cancer and chronic obstructive pulmonary disease. Alcohol can cause some cancers and liver disease and can increase risky behaviors, but it is not a major cause of lung cancer. Although asbestos is carcinogenic and some components of marijuana are carcinogenic, neither is the major risk factor for lung cancer.

The medical-surgical unit has one negative-airflow room. Which of these four clients who have just arrived on the unit should the charge nurse admit to this room?

Client with possible pulmonary tuberculosis who currently has hemoptysis A client with possible tuberculosis should be admitted to the negative-airflow room to prevent airborne transmission of tuberculosis. A client with bacterial pneumonia does not require a negative-airflow room but should be placed in Droplet Precautions. A client with neutropenia should be in a room with positive airflow. The client with a right empyema who also has a chest tube and a fever should be placed in Contact Precautions but does not require a negative-airflow room.

The nurse is preparing a client for discharge who has undergone percutaneous needle aspiration of a peritonsillar abscess. Which is most important to teach the client about follow-up care?

Contacting the provider if the throat feels more swollen Clients with peritonsillar abscess are at risk for airway obstruction due to swelling and should notify the provider if signs of obstruction occur, such as stridor or drooling. It is important to complete the antibiotics to treat the infection, and to adhere to comfort measures such as analgesic medications and saline gargles, but none of these is the most important thing to teach the client.

The nurse prepares to assess a client with diabetes mellitus for sensory loss. Which equipment is the best choice for the nurse use to perform this assessment? Cotton-tipped applicator Glucometer Hammer Safety pin

Cotton-tipped applicator A cotton-tipped applicator is the nurse's best choice to assess sensory loss on a client with diabetes mellitus. Sensory loss is assessed with any sharp or dull object, such as a cotton-tipped applicator. The client indicates whether the touch is sharp or dull. The soft and hard ends of the applicator would be interchanged at random so that the client does not anticipate the next type of sensation. A glucometer tests blood sugar. A hammer tests tendon reflexes. Although a safety pin could be used to test for sensory loss, a cotton-tipped applicator is safer in the event the client is taking anticoagulants.

The nurse has taught a client about influenza infection control. Which client statement indicates the need for further teaching? A) "Handwashing is the best way to prevent transmission." B) "I should avoid kissing and shaking hands." C) "It is best to cough and sneeze into my upper sleeve." D) "The intranasal vaccine can be given to everybody in the family." (Chp. 31; elsevier resources)

D) "The intranasal vaccine can be given to everybody in the family." (Chp. 31; elsevier resources)

The nurse is preparing to admit an adult client with pertussis. Which symptom does the nurse anticipate finding in this client? A) "Whooping" after a cough B) Hemoptysis C) Mild cold-like symptoms D) Post-cough emesis (Chp. 31; elsevier resources)

D) Post-cough emesis (Chp. 31; elsevier resources)

Community health nurses are tasked with providing education on prevention of respiratory infection for diseases such as the flu. Which target audience is given the highest priority? A) Homeless people B) Hospital staff C) Politicians D) Prison staff and inmates (Chp. 31; elsevier resources)

D) Prison staff and inmates (Chp. 31; elsevier resources)

The nurse notices a visitor walking into the room of a client on airborne isolation with no protective gear. What does the nurse do? A) Ensures that the client is wearing a mask B) Tells the visitor that the client cannot receive visitors at this time C) Provides a particulate air respirator to the visitor D) Provides a mask to the visitor (Chp. 31; elsevier resources)

D) Provides a mask to the visitor (Chp. 31; elsevier resources)

A client comes to the emergency department with a sore throat. Examination reveals redness and swelling of the pharyngeal mucous membranes. Which diagnostic test does the nurse expect will be requested first? A) Chest x-ray B) Complete blood count (CBC) C) Tuberculosis (TB) skin test D) Throat culture (Chp. 31; elsevier resources)

D) Throat culture (Chp. 31; elsevier resources)

The nurse is working in an urgent clinic. Which of these four clients needs to be evaluated first by the nurse? A. Client who is short of breath after walking up two flights of stairs B. Client with soreness of the arm after receiving purified protein derivative (PPD) (Mantoux) skin test C. Client with sore throat and fever of 39° C oral D. Client who is speaking in three-word sentences and has SaO2 of 90% by pulse oximetry

D. Client who is speaking in three-word sentences and has SaO2 of 90% by pulse oximetry Correct: A client should be able to speak in sentences of more than three words, and Sao2 of 90% indicates hypoxemia that requires intervention on the part of the nurse.

3. 3. A client has returned to the postanesthesia care unit (PACU) after a bronchoscopy. Which of these nursing tasks is best for the charge nurse to delegate to the experienced nursing assistant working in PACU? A. Assess breath sounds. B. Check gag reflex. C. Determine level of consciousness. D. Monitor blood pressure and pulse.

D. Monitor blood pressure and pulse. Correct: A nursing assistant working in the PACU would have experience in taking client vital signs after the client has had conscious sedation or anesthesia.

The nurse is performing a neurologic assessment on an 81-year-old client. Which physiologic change does the nurse expect to find because of the client's age? Decreased coordination Increased sleeping during the night Increased touch sensation Nightly confusion

Decreased coordination When performing a neurologic assessment on an elderly client, the nurse expects to find decreased coordination. Older adults experience decreased coordination as a result of the aging process.Older adults frequently go to bed earlier and arise earlier than younger adults. Sensation to touch is decreased not increased. Nightly confusion, sometimes referred to as "sundowning," is not an expected change with all older adults.

Which assessment findings does the nurse expect in a client with kidney cancer? (Select all that apply.) Erythrocytosis Hypokalemia Hypercalcemia Hepatic dysfunction Increased sedimentation rate

Erythrocytosis Correct Hypokalemia Hypercalcemia Correct Hepatic dysfunction Correct Increased sedimentation rate Correct Erythrocytosis alternating with anemia and hepatic dysfunction with elevated liver enzymes may occur with kidney cancer. Parathyroid hormone produced by tumor cells can cause hypercalcemia. An elevation in sedimentation rate may occur in paraneoplastic syndromes. Potassium levels are not altered in kidney cancer, but hypercalcemia is present.

A client with asthma reports shortness of breath. What is the nurse assessing when auscultating this client's chest? Expiratory wheezing not cleared by coughing Bronchial breath sounds over the trachea Crackles throughout the lung fields Bronchovesicular breath sounds in the lung bases

Expiratory wheezing not cleared by coughing In a client with asthma and shortness of breath, the nurse expects to hear expiratory wheezing not cleared by coughing. Wheezes are squeaky, musical, continuous sounds associated with bronchospasm, typical with asthma. They may be heard without a stethoscope and usually do not clear with coughing.Bronchial breath sounds are normal breath sounds, heard over the trachea and larynx. Crackles, an adventitious breath sound, will sound like popping, discontinuous sounds caused by air moving into previously deflated airways or coarse rattling sounds caused by fluid. Bronchovesicular breath sounds are normal breath sounds heard over major bronchi where fewer alveoli are located. They are best heard between the scapula and anterior chest.

Which symptom of pneumonia may present differently in the older adult than in the younger adult?

Fever Older adults may not have fever and may have a lower-than-normal temperature with pneumonia. Crackles on auscultation may be present in all age groups as the result of fluid in the lungs. All age ranges may have a headache with pneumonia. Wheezing is an indication of narrowed airways and can be found in all age groups.

The nurse is performing a rapid neurologic assessment on a trauma client. Which assessment findings are normal? Select all that apply. Decerebrate posturing Glasgow Coma Score (GCS) 15 Lethargy Minimal response to stimulation Pupil constriction to light

Glasgow Coma Score (GCS) 15 Minimal response to stimulation Normal rapid neurologic assessment findings include a GCS (Glasgow Coma Score) of 15 and pupil constriction to light. The GCS range is between 3 and 15. Pupil constriction is a function of cranial nerve III. The pupils would be equal in size and round and regular in shape and would react to light and accommodation (PERRLA).Decerebrate or decorticate posturing is not normal, as well as pinpoint or dilated and nonreactive pupils. Both of findings are a late sign of neurologic deterioration. In addition, minimal response to stimulation and increased lethargy are not normal findings.

The nurse is teaching a client who has been newly diagnosed with cancer. For which side effect specific to radiation does the nurse teach prevention techniques?

Increased risk for sunburn Skin in the path of radiation is more sensitive to sun damage; therefore, clients must avoid direct skin exposure to the sun during treatment and for at least 1 year after radiation is completed. Alopecia, or hair loss, is a side effect of chemotherapy, not of radiation. Loss of appetite is not specific to radiation therapy. Radiation therapy itself is painless and sensation-free; however, the skin may become sore and prone to breakdown over the course of treatment.

An older client presents to the emergency department with a 2-day history of cough, pain on inspiration, shortness of breath, and dyspnea. The client never had a pneumococcal vaccine. The client's chest x-ray shows density in both bases. The client has wheezing upon auscultation of both lungs. Would a bronchodilator be beneficial for this client?

It would help decrease the bronchospasm. A bronchodilator would help decrease bronchospasm and would open up the airways, so it would be beneficial for this client. It would decrease dyspnea and feelings of shortness of breath. A bronchodilator would not be able to clear up the density in the bases of the client's lung. The cause of the density is unknown; however, an infection such as pneumonia is likely, which bronchodilators do not treat. Although a bronchodilator would help a client breathe easier, it does not have any analgesic properties.

Which staff member does the charge nurse assign to a client who has benign prostatic hyperplasia and hydronephrosis and needs an indwelling catheter inserted? RN float nurse who has 10 years of experience with pediatric clients LPN/LVN who has worked in the hospital's kidney dialysis unit until recently RN without recent experience who has just completed an RN refresher course LPN/LVN with 5 years of experience in an outpatient urology surgery center

LPN/LVN with 5 years of experience in an outpatient urology surgery center Catheterization of a client with an enlarged prostate, a skill within the scope of practice of the LPN/LVN, would be performed frequently in a urology center. The pediatric nurse would have little exposure to prostatic obstruction and adult catheterization. Dialysis clients do not typically have catheters inserted, so the LPN/LVN from the kidney dialysis unit would not be the best staff member to assign to the client. The nurse who has been out of practice for several years is not the best candidate to insert a catheter in a client with an enlarged prostate.

The nurse anticipates that a client who develops hypotension and oliguria post nephrectomy may need the addition of which element to the regimen? Increase in analgesics Addition of a corticosteroid Administration of a diuretic Course of antibiotic therapy

Loss of water and sodium occurs in clients with adrenal insufficiency, which is followed by hypotension and oliguria; corticosteroids may be needed. The nurse should use caution when administering analgesics to a hypotensive client; no indication suggests that pain is present in this client. A diuretic would further contribute to fluid loss and hypotension, potentially worsening kidney function. A few doses of antibiotics are used prophylactically preoperatively and postoperatively; additional therapy is used when evidence of infection exists.

Which goal for a client with diabetes will best help to prevent diabetic nephropathy? Heed the urge to void. Avoid carbohydrates in the diet. Take insulin at the same time every day. Maintain glycosylated hemoglobin (HbA1c).

Maintaining long-term control of blood glucose will help prevent the progression of diabetic nephropathy. Voiding when the client has the urge prevents the backflow of urine and infection. The diabetic diet is composed of carbohydrates, proteins, and fats. Although taking insulin at the same time each day may indirectly help control blood glucose, it is not the best option.

Which information suggests that a client with diabetes may be in the early stages of kidney damage? Elevation in blood urea nitrogen (BUN) Oliguria Microalbuminuria Painless hematuria

Microalbuminuria In the early stages of diabetic nephropathy, micro-levels of albumin are first detected in the urine. Progressive kidney damage occurs before dipstick procedures can detect protein in the urine. BUN may change in response to protein and fluid intake. Oliguria is a later finding in kidney disease and may also be present in dehydration. Painless hematuria often occurs with kidney cancer.

A client has returned to the postanesthesia care unit (PACU) after a bronchoscopy. Which nursing task is best for the charge nurse to delegate to the experienced nursing assistant working in the PACU? Assess breath sounds. Offer clear liquids when gag reflex returns. Determine level of consciousness. Monitor blood pressure and pulse.

Monitor blood pressure and pulse. A nursing assistant working in the PACU would have experience in taking client vital signs after the client has had conscious sedation or anesthesia. Evaluating breath sounds and gag reflex and determining level of consciousness require the skill and knowledge of a higher-level provider.

A client has asthma that gets worse during the summer. She tells the nurse that she takes a medication every day so she does not get short of breath when she walks to work. About which medicine does the nurse need to educate the client?

Montelukast (Singulair) Montelukast is a leukotriene antagonist that works well for asthma that occurs during certain seasons. It is taken on a daily basis as a preventive medication. Albuterol inhalers are beta2 agonists that are rescue medications used on an as-needed basis only. Guaifenesin is a mucolytic that does not provide any bronchodilation; it may or may not be taken daily. Omalizumab is an immunomodulator that is injected subcutaneously every 2 to 3 weeks; it is not commonly used because a high rate of anaphylaxis is associated with it.

A newly diagnosed client with asthma says that his peak flowmeter is reading 82% of his personal best. What does the nurse do?

Repeat the peak flow test. Since the client is newly diagnosed with asthma, this would be an excellent opportunity for the nurse to observe the client using the peak flowmeter to ensure that the client is using it properly, so readings are accurate and in the green zone, at least 80% of the client's personal best. The result of 82% is in the green zone, but this is not the best answer for a newly diagnosed client. Rescue drugs should be used only in the yellow zone, between 50% and 80% of the client's personal best. They should not be used in this situation, and the nurse does not need to seek emergency help until readings are in the red zone, or below 50% of the client's personal best.

Which condition may predispose a client to chronic pyelonephritis? Spinal cord injury Cardiomyopathy Hepatic failure Glomerulonephritis

Spinal cord injury Chronic pyelonephritis occurs with spinal cord injury, bladder tumor, prostate enlargement, or urinary tract stones. Weakness of the heart muscle may cause kidney impairment, not an infection. Pyelonephritis may damage the kidney, not the liver. Glomerulonephritis may result from infection, but may not cause infection of the kidney.

An environmental assessment of a factory finds inhalation exposure with a high level of particulate matter. What does the factory nurse do to generate the quickest compliance?

Teaches workers how to use a mask Teaching everyone to use a mask when working in areas with high levels of particulate matter can reduce individual exposure. Proper building ventilation often requires work orders, reconstruction, time, and money; this will need to be implemented, but it will not occur quickly. Particulate matter can be emitted from a variety of sources; smoking may be unrelated to the question. Suggesting that workers find another job does not solve the problem of particulate matter in a rapid or safe manner.

After receiving change-of-shift report on the urology unit, which client does the nurse assess first? Client post radical nephrectomy whose temperature is 99.8° F (37.6° C) Client with glomerulonephritis who has cola-colored urine Client who was involved in a motor vehicle crash and has hematuria Client with nephrotic syndrome who has gained 2 kg since yesterday

The nurse should be aware of the risk for kidney trauma after a motor vehicle crash; this client needs further assessment and evaluation to determine the extent of blood loss and the reason for the hematuria because hemorrhage can be life-threatening. Although slightly elevated, the low-grade fever of the client who is post radical nephrectomy is not life-threatening in the same way as a trauma victim with bleeding. Cola-colored urine is an expected finding in glomerulonephritis. Because of loss of albumin, fluid shifts and weight gain can be anticipated in a client with nephrotic syndrome.

A client comes to the emergency department with a sore throat. Examination reveals redness and swelling of the pharyngeal mucous membranes. Which diagnostic test does the nurse expect will be requested first?

Throat culture A throat culture is important for distinguishing a viral infection from a group A beta-hemolytic streptococcal infection. A chest x-ray or TB skin test is not indicated by the symptoms given. A CBC might be indicated to evaluate infection and dehydration, but would not be the first action.

A client has a fever of 104° F (40° C). In which direction, if any, does this shift the oxygen-hemoglobin dissociation curve? Shift downward Shift to the left Shift to the right Shift upward

To the right A client with fever has a higher demand for oxygen, so the curve will shift to the right for easier dissociation. The curve does not move up or down on the vertical axis. Moving to the left would cause hemoglobin to dissociate oxygen less easily.

The nurse is assessing a client with a neurologic condition who is reporting difficulty chewing when eating. The nurse suspects that which cranial nerve has been affected? Abducens (CN VI) Facial (CN VII) Trigeminal (CN V) Trochlear (CN IV)

Trigeminal (CN V) The nurse suspects that the trigeminal cranial nerve is affected when a client complains of difficulty chewing when eating. The trigeminal nerve affects the muscles of mastication.The abducens nerve affects eye movement via lateral rectus muscles. The facial nerve affects pain and temperature from the ear area, deep sensations in the face, and taste in the anterior two-thirds of the tongue. The trochlear nerve affects eye movement via superior oblique muscles.

When caring for a client who had a nephrostomy tube inserted 4 hours ago, which is essential for the nurse to report to the health care provider? Dark pink-colored urine Small amount of urine leaking around the catheter Tube that has stopped draining Creatinine of 1.8 mg/dL

Tube that has stopped draining The provider must be notified when a nephrostomy tube does not drain; it could be obstructed or dislodged. Pink or red drainage is expected for 12 to 24 hours after insertion and should gradually clear. The nurse may reinforce the dressing around the catheter to address leaking urine; however, the provider should be notified if there is a large quantity of leaking drainage, which may indicate tube obstruction. A creatinine level of 1.8 mg/dL is expected in a client early after nephrostomy tube placement (due to the minor kidney damage that required the nephrostomy tube).

The RN is working with unlicensed assistive personnel (UAP) in caring for a group of clients. Which action is best for the RN to delegate to UAP? Assessing the vital signs of a client who was just admitted with blunt flank trauma and hematuria Assisting a client who had a radical nephrectomy 2 days ago to turn in bed Helping the provider with a kidney biopsy for a client admitted with acute glomerulonephritis Palpating for bladder distention on a client recently admitted with a ureteral stricture

UAP would be working within legal guidelines when assisting a client to turn in bed. Although assessment of vital signs is within the scope of practice for UAP, the trauma victim should be assessed by the RN because interpretation of the vital signs is needed. Assisting with procedures such as kidney biopsy and assessment for bladder distention are responsibilities of the professional nurse that should not be delegated to staff members with a limited scope of education.

A client with pyelonephritis has been prescribed urinary antiseptic medication. What purpose does this medication serve? Decreases bacterial count Destroys white blood cells Enhances the action of antibiotics Provides comfort

Urinary antiseptic drugs such as nitrofurantoin (Macrodantin) are prescribed to provide comfort for clients with pyelonephritis. Antibiotics, not antiseptics, are used to decrease bacterial count and treat pyelonephritis infection; the action of antibiotics is not enhanced with antiseptics. White blood cells, along with antibiotics, fight infection.

What is the appropriate range of urine output for the client who has just undergone a nephrectomy? 23 to 30 mL/hr 30 to 50 mL/hr 41 to 60 mL/hr 50 to 70 mL/hr

Urine output of 30 to 50 mL/hr or 0.5 to 1 mL/kg/hr is considered within acceptable range for the client who is post nephrectomy. Output of less than 25 to 30 mL/hr suggests decreased blood flow to the kidney and the onset or worsening of acute kidney injury. A large urine output, followed by hypotension and oliguria, is a sign of hemorrhage and adrenal insufficiency.

Which cranial nerve allows a person to feel a light breeze on the face? I (olfactory) III (oculomotor) V (trigeminal) VII (facial)

V (trigeminal) Cranial nerve V (trigeminal) allows the person to feel a light breeze on the face. This nerve is responsible for sensation from the skin of the face and scalp and the mucous membranes of the mouth and nose.Cranial nerve I (olfactory) is responsible for smell. Cranial nerve III (oculomotor) is responsible for eye movement. Cranial nerve VII (facial) is responsible for pain and temperature from the ear area, deep sensations from the face, and taste from the anterior two thirds of the tongue.

A pt enters the ED after being punched in the throat. What does the nurse monitor for? a. aphonia b. dry cough c. crepitus d. loss of gag reflex

a

A pt has an inner maxillary fixation. The nurse encourages the pt to ear which kind of food? a. milkshakes b. cottage cheese c. tea and toast d. tuna noodle casserole

a

A pt has been diagnosed with airway obstruction during sleep. The nurse will likely include pt education about which device for home use? a. continuous positive airway pressure (CPAP) to deliver a positive airway pressure b. O2 via face mask to prevent hypoxia c. neck brace to support the head to facilitate breathing d. nebulizer treatments with bronchodilators

a

A pt has been diagnosed with sleep apnea. Which assessment findings indicate that the pt is having complications associated with sleep apnea? a. side effects of hypoxemia, hypercapnia and sleep deprivation b. decrease in arterial CO2 levels and sleep deprivation c. respiratory alkalosis with retention of CO2 d. irritability, obesity, and enlarged tonsils/adenoids

a

A pt has had neck dissection surgery with a reconstructive flap over the carotid artery. Which intervention is appropriate for the flap care? a. evaluate the flap every hour for the first 72 hours b. monitor the flap by gently placing a doppler on the flap c. position the pt so that the flap is in the dependent position d. apply a wet-to-dry dressing to the flap

a

A pt with an active nosebleed (epistaxis) is admitted to the ED. Which intervention does the nurse use FIRST? a. have the pt sit upright with the head forward b. insert nasal packing c. apply direct lateral pressure to the nose d. place a nasal catheter

a

After being treated in the ED for posterior nosebleed, the pt is admitted to the hospital. The nasal packing is in place and vital signs are stable. The pt has an IV of NS at 125 ml/hr. What is the PRIORITY for nursing care? a. airway management b. managing potential dehydration c. managing potential decreased cardiac output d. monitoring for potential infection

a

On post-op assessment, the nurse notes that the pt with a rhinoplasty repeatedly swallows. What is the nurse's FIRST action? a. examine the throat for bleeding b. provide ice chips to ease swallowing c. notify the HCP d. ask if the pt is hungry

a

The HCP orders the discontinuation of the NG tube for a pt with a total laryngectomy. Before discontinuing the tube, which action must be performed? a. the HCP and the nurse will assess the pt's ability to swallow b. reassure the pt that eating and swallowing will be painless and natural c. the nutritionist will evaluate the pt's nutritional status d. the pt will be offered a prn analgesic or an anxiolytic med

a

The nurse is assessing a pt who has had a neck dissection with removal of muscle tissue, lymph nodes, and the 11th cranial nerve. Which assessment finding is anticipated because of the surgical procedure? a. shoulder drop with an increased limitation of movement b. asymmetrical eye movements and a change of visual acuity c. weak, hoarse voice d. facial swelling with discoloration and bruising around the eyes

a

The nurse is assessing a pt's skin at the site of radiation therapy to the neck. Which skin condition is expected in relation to the radiation treatments? a. red, tender, and peeling b. shiny, pale and tight c. puffy and edematous d. pale, dry and cool

a

The nurse is caring for a pt who had a nasoseptoplasty. Which action is the best to assign to the LPN? a. administer a stool softener to ease bowel movements b. assess the pt's airway and breathing after general anesthesia c. evaluate the pr's emotional rxn to the facial edema and bruising d. take vital signs every 4 hrs as ordered by the physician

a

The nurse is providing post-op nursing care for a pt with surgical correction of a deviated septum. Which intervention is part of the standard care for this pt? a. apply ice to the nasal area and eyes to decrease swelling and pain b. encourage deep coughing to prevent atelectasis and clear secretions c. administer NSAIDs/Tylenol q4-6 hrs for pain d. apply moist heat and humidity to the nasal area for comfort and circulation

a

Which statement by the pt indicates understanding about radiation therapy for neck cancer? a. my voice will initially be hoarse, but should improve over time b. there are no side effects other than a hoarse voice c. dry mouth after radiation therapy is temporary and short term d. my throat is not directly affected by radiation

a

Which surgical procedure of the neck area poses no risk postoperatively for aspiration? a. total laryngectomy b. transoral cordectomy c. hemilaryngectomy d. partial laryngectomy

a

Which are warning signs of head and neck cancer? SELECT ALL THAT APPLY a. difficulty swallowing b. change in fit of dentures c. intermittent bilateral ear pain d. weight gain e. numbness in the mouth, lips or face f. lump in the mouth, neck or throat

a b e f

The pt has a diagnosis of mild sleep apnea. Which interventions will the nurse teach the pt that may correct this condition? SELECT ALL THAT APPLY a. change sleeping positions b. use a CPAP every night c. look into a weight loss program d. a position fixing device can prevent tongue subluxation e. you may need surgery to remodel your posterior oropharynx f. a prescription for modafinil may help promote wakefulness during the day

a c d

Which med-surg concepts take priority when the nurse is caring for a patient with a non-infectious upper respiratory problem? SELECT ALL THAT APPLY a. gas exchange b. acid-base balance c. fluid and electrolyte balance d. cellular regulation e. tissue integrity f. elimination

a d

A pt is receiving enteral feedings and an NG tube is in place. In order to prevent aspiration, which precautions are used? SELECT ALL THAT APPLY a. no bolus feedings are given at night b. hold the feeding if the residual volume exceeds 20 ml c. vary the time of feedings according to pt's preference d. elevate the head of the bed during and after feedings e. evaluate the pt's tolerance of the feedings f. allow the pt to indicate when he/she is ready for the next bite

a d e f

A pt is admitted for a posterior nosebleed. Posterior packing is in place and the pt is on O2 therapy, ABX, and opioid analgesics. What is the PRIORITY assessment? a. tolerance of packing or tubes b. gag and cough reflexes c. mouth breathing d. skin breakdown around the nares

b

An older adult with dehydration has altered mental status and inspissated (thickly crusted) oral and nasopharyngeal secretions. What PRIORITY instruction would the nurse give the UAP when providing care for this pt? a. bathe the pt 2X/day b. provide comprehensive oral care every 2 hrs c. ambulate the pt in the hall every 4 hrs d. check vital signs, including temp, every 6 hrs

b

The nurse is assessing a pt with significant and obvious facial trauma after being struck repeatedly in the face. Which finding is the PRIORITY and requires immediate intervention? a. asymmetry of the mandible b. restlessness with high-pitched respirations c. nonparallel extra-ocular movements d. pain upon palpation over nasal bridge

b

The nurse is teaching a pt about post rhinoplasty care. Which pt statement indicates an understanding of the instruction? a. i will have a very large dressing on my nose b. I will have bruising around my eyes, nose and face c. there will swelling that will cause a loss of sense of smell d. my nose will be 3 times its normal size for 3 weeks

b

The pt with a nasal fracture has clear fluid draining from the nose that dries on a piece of filter paper and leaves a yellow "halo" ring on the dried edge of the fluid. What is the nurse's FIRST action? a. document the finding b. notify the HCP c. send a sample to the lab d. place the pt in a supine position

b

Which intervention does the nurse use to assist the pt who suffers from chronic xerostomia secondary to past radiation treatments? a. offer small, frequent meals b. suggest a moisturizing spray c. explain fluid restrictions d. teach to wash with mild soap and water

b

Question 29 of 29 The nurse is caring for a client diagnosed with partial seizures after encephalitis, who is to receive carbamazepine (Tegretol). The nurse plans to monitor the client for which adverse effects? Select all that apply. a. Alopecia b. Headaches c. Dizziness d. Diplopia e. Increased blood glucose

b, c, d Headaches, Dizziness, Diplopia Headaches, dizziness, and diplopia are adverse effects of carbamazepine because this drug affects the central nervous system. Carbamazepine does not cause alopecia and does not increase blood glucose; divalproex (Depakote) and valproic acid (Depakene) may cause alopecia.

Question 18 of 29 A client has been admitted with new-onset status epilepticus. Which seizure precautions does the nurse put in place? Select all that apply. a. Bite block at the bedside b. Intravenous access c. Continuous sedation d. Suction equipment at the bedside e. Siderails up

b, d, e Intravenous access, Suction equipment at the bedside, Siderails up Intravenous access is needed to administer medications. Suctioning equipment should be available to suction secretions and facilitate an open airway during a seizure. Raised, padded siderails may be used to protect the client from falling out of bed during a seizure. Bite blocks or padded tongue blades should not be used because the client's jaw may clench, causing teeth to break and possibly obstructing the airway. Continuous sedation is a medical intervention and not a seizure precaution.

A pt arrives in the ED with a severe crush injury to the face with blood gurgling from the mouth and nose and obvious respiratory distress. The nurse prepares to assist the physician with which procedure to manage the airway? a. performing a needle thoracotomy b. inserting and endotracheal tube c. performing a tracheotomy d. inserting a nasal airway and giving O2

c

A pt had a rhinoplasty and is preparing for discharge home. A family member is instructed by the nurse to monitor the pt for postnasal drip by using a flashlight to look in the back of the throat. If the bleeding is noted, what does the nurse tell the family member to do? a. place ice packs on the back of the neck and apply pressure to the nose b. hyperextend the neck and apply pressure and ice packs as needed c. seek immediate medical attention for the bleeding d. monitor for 24 hrs if the bleeding appears to be a small amount

c

A pt is having radiation therapy to the neck and reports a sore throat and difficulty swallowing. Which statement by the nursing student indicates a correct understanding of symptoms relief for this pt? a. the pt should not swallow anything too cold or too hot b. I will give the pt a mouthwash with an alcohol base c. I will help the pt with a saline gargle d. the pt should be reassured that the sore throat is temporary

c

A pt is unable to speak following a cordectomy. Which action is delegated to the UAP to assist the pt in dealing with the communication issues? a. politely tell the pt not to communicate b. teach the pt how to use hand signals c. allow extra time to accomplish ADLs d. give step-by-step instructions during the ADLs and discourage 2 way communication

c

In order to facilitate comfort and breathing for the pt with a laryngeal tumor, the nurse should use which position? a. Sim's b. supine c. Fowler's d. prone

c

The nurse is caring for a pt who had a constructive neck surgery and observes bright red blood spurting from the tissue flap that is covering the carotid artery. Which action must the nurse take FIRST? a. call the surgeon and alert the OR b. call the RRT c. apply immediate, direct pressure to the site d. apply a bulky sterile dressing and secure the airway

c

The nurse is caring for several pts who are at risk because of problems related to the upper airway. What are the PRIORITY assessments and actions for these pts? a. thickness of oral secretions; encourage ingestion of oral fluids b. anxiety and pain; provide reassurance and NSAIDs c. adequacy of oxygenation; ensure an unobstructed airway d. evidence of spinal cord injuries; obtain an order for x-rays

c

The nurse observes that a pt is having difficulty swallowing and has initiated aspiration precautions. Which procedure does the nurse expect the HCP to order for this pt? a. chest x-ray of the neck and chest b. CT scan of the head and neck c. dynamic swallow study under fluoroscopy d. direct and indirect laryngoscopy

c

The nursing student is caring for an older adult pt who sustained a stroke, is confused and is having trouble swallowing. Which statement by the nursing student indicates an understanding of aspiration precautions for this pt? a. i will administer pills as whole tabs as they are easier to swallow b. if the pt coughs, I will discontinue feeding and contact the physician c. I will keep the head of the bed elevated during and after feeding d. I will encourage small amounts of fluids, such as water, tea or juice

c

The pt with laryngeal trauma develops stridor. What is the nurse's HIGHEST priority intervention? a. apply O2 by nasal cannula b. obtain ABGs c. call RRT d. perform a maneuver to open the airway

c

Which symptom suggests the possibility of sinus cancer? a. intermittent nasal obstruction b. little to no nasal drainage c. bloody nasal discharge d. lymph node enlargement on opposite side from tumor

c

While playing football at school, a pt injured his nose, resulting in a possible simple fracture. The pt's parents call the nurse seeking advice. What does the nurse tell the parents to do? a. ask the school nurse to insert a nasal airway to ensure patency b. apply an ice pack and allow the pt to rest in a supine position c. seek medical attn within 24 hours to minimize further complications d. monitor the symptoms for 24 hrs and contact the physician if there is bleeding

c

Question 25 of 29 A client with dementia and Alzheimer's disease is discharged to home. The client's daughter says, "He wanders so much, I am afraid he'll slip away from me." What resource does the nurse suggest? a. Alzheimer's Wandering Association b. National Alzheimer's Group c. Safe Return Program d. Lost Family Members Tracking Association

c Safe Return Program The family should enroll the client in the Safe Return Program, a national, government-funded program of the Alzheimer's Association that assists in the identification and safe, timely return of those with dementia who wander off and become lost. The Alzheimer's Wandering Association, National Alzheimer's Group, and Lost Family Members Tracking Association do not exist.

A pt has had surgery for cancer of the neck. Which behavior indicates that the pt understands how to perform self-care to prevent aspiration? a. chooses thin liquids that cause coughing but knows to take small sips b. eats small, frequent meals that include a variety of textures and nutrients c. asks for small, frequent sips of nutrition supplement as a bedtime snack d. positions self upright before eating/drinking anything

d

A pt in the ED with laryngeal trauma has developed shortness of breath with stridor and decreased O2 saturation. What is the PRIORITY action? a. insert an oral/nasal airway b. assess for tachypnea, anxiety and nasal flaring c. obtain equipment for a trach d. apply O2 and stay with the pt

d

A pt is experiencing acute anxiety related to hospitalization stress and an inability to accept changes related to laryngeal cancer. The pt wants to leave the hospital but agrees to try a med to "help me calm down". For which med does the nurse obtain a prn order? a. amitriptyline b. modafinil c. morphine sulfate d. lorazepam

d

The nurse is assessing a pt who reports being struck in the face and head several times. During the assessment, pink-tinged drainage from the nares is observed. Which nursing action provides relevant assessment data? a. have the pt gently blow the nose and observe for bloody mucous b. test the drainage with a reagent to check the pH c. ask the pt to describe the appearance of the face before the incident d. place a drop of the drainage on a filter paper and look for a yellow ring

d

The nurse is caring for a post-op pt who had a neck dissection. Which assessment finding is expected? a. bulky gauze dressing is present that is dry and intact over the site b. the pt can speak normally but reports a sore throat c. permanent gastrostomy tube is present with continuous tube feedings d. the pt has shoulder drop and limited range of motion

d

The nurse is caring for several pts who require treatment for laryngeal cancer. Which treatment/procedure requires pt education about aspiration precautions? a. total laryngectomy b. laser surgery c. radiation therapy d. supraglottic laryngectomy

d

The nursing student is preparing patient teaching materials about head and neck cancer. Which statement is accurate and included in the patient teaching info? a. it metastasizes often to the brain b. it usually develops over time c. it is often seen as red edematous areas d. it is often seen as white patchy mucosal lesions

d

What type of treatment has the HIGHEST cure rate for small cancers of the head and neck? a. surgery b. chemo c. laser surgery d. radiation therapy

d

(Chp. 31; elsevier resources)

(Chp. 31; elsevier resources)

A nurse assesses a 66-year-old client who is attempting to quit smoking. The client states, "I started smoking at age 16, and smoked one pack each day until 10 years ago. Then I decreased to a half of a pack per day." How many pack-years should the nurse document for this client? (Record your answer using a whole number.) ___ pack-years

45 pack-years 66 (current age) - 16 (year started smoking) = 50 years of smoking. (40 years ´ 1 pack per day) + (10 years ´ 0.5 pack per day) = 45 pack-years.

69. A patient has developed pulmonary hypertension. What is the goal of drug therapy for this patient? A. Dilate pulmonary vessels and prevent clot formation. B. Decrease pain and make the patient comfortable. C. Improve or maintain gas exchange. D. Maintain and manage pulmonary exacerbation.

A

7. The nurse is helping a patient learn about managing her asthma. What does the nurse instruct the patient to do? A. Keep a symptom diary to identify what triggers the asthma attacks. B. Make an appointment with an allergist for allergy therapy. C. Take a low dose of aspirin every day for the anti-inflammatory action. D. Drinks large amounts of clear fluid to keep mucus thin and watery.

A

70. A patient is newly diagnosed with sarcoidosis. Which statement by the patient indicates an understanding of the disease? A. "Corticosteroids are the main type of therapy for sarcoidosis." B. "Sarcoidosis is a type of lung cancer that is treatable if diagnosed early." C. "My condition can be treated with antibiotics." D. "Sarcoidosis is a type of pneumonia that is highly contagious."

A

The nurse auscultates popping, discontinuous sounds over the client's anterior chest. How does the nurse classify these sounds? A. Crackles B. Rhonchi C. Pleural friction rub D. Wheeze

A. Crackles Correct: Crackles are described as a popping, discontinuous sound caused by air moving into previously deflated airways. The airways have been deflated due to the presence of fluids in the lungs, and crackles should be considered to be a sign of fluid overload.

A nurse auscultates a harsh hollow sound over a client's trachea and larynx. Which action should the nurse take first? a. Document the findings. b. Administer oxygen therapy. c. Position the client in high-Fowler's position. d. Administer prescribed albuterol.

ANS: A Bronchial breath sounds, including harsh, hollow, tubular, and blowing sounds, are a normal finding over the trachea and larynx. The nurse should document this finding. There is no need to implement oxygen therapy, administer albuterol, or change the client's position because the finding is normal.

A client has been advised to perform weight-bearing exercises to help minimize osteoporosis. The client admits to not doing the prescribed exercises. What action by the nurse is best? a. Ask the client about fear of falling. b. Instruct the client to increase calcium. c. Suggest other exercises the client can do. d. Tell the client to try weight lifting.

ANS: A Fear of falling can limit participation in activity. The nurse should first assess if the client has this fear and then offer suggestions for dealing with it. The client may or may not need extra calcium, other exercises, or weight lifting.

A nurse reviews laboratory results for a client who was admitted for a myocardial infarction and cardiogenic shock 2 days ago. Which laboratory test result should the nurse expect to find? a. Blood urea nitrogen (BUN) of 52 mg/dL b. Creatinine of 2.3 mg/dL c. BUN of 10 mg/dL d. BUN/creatinine ratio of 8:1

ANS: A Shock leads to decreased renal perfusion. An elevated BUN accompanies this condition. The creatinine should be normal because no kidney damage occurred. A low BUN signifies overhydration, malnutrition, or liver damage. A low BUN/creatinine ratio indicates fluid volume excess or acute renal tubular acidosis.

A client had a bunionectomy with osteotomy. The client asks why healing may take up to 3 months. What explanation by the nurse is best? a. "Your feet have less blood flow, so healing is slower." b. "The bones in your feet are hard to operate on." c. "The surrounding bones and tissue are damaged." d. "Your feet bear weight so they never really heal."

ANS: A The feet are the most distal to the heart and receive less blood flow than other organs and tissues, prolonging the healing time after surgery. The other explanations are not correct.

The health care provider tells the nurse that a client is to be started on a platelet inhibitor. About what drug does the nurse plan to teach the client? a. Clopidogrel (Plavix) b. Enoxaparin (Lovenox) c. Reteplase (Retavase) d. Warfarin (Coumadin)

ANS: A Clopidogrel is a platelet inhibitor. Enoxaparin is an indirect thrombin inhibitor. Reteplase is a fibrinolytic agent. Warfarin is a vitamin K antagonist.

A nurse collaborates with the interdisciplinary team to develop a plan of care for a client who is newly diagnosed with diabetes mellitus. Which team members should the nurse include in this interdisciplinary team meeting? (Select all that apply.) a. Registered dietitian b. Clinical pharmacist c. Occupational therapist d. Health care provider e. Speech-language pathologist

ANS: A, B, D When planning care for a client newly diagnosed with diabetes mellitus, the nurse should collaborate with a registered dietitian, clinical pharmacist, and health care provider. The focus of treatment for a newly diagnosed client would be nutrition, medication therapy, and education. The nurse could also consult with a diabetic educator. There is no need for occupational therapy or speech therapy at this time.

The nurse is assessing a client for chronic osteomyelitis. Which features distinguish this from the acute form of the disease? (Select all that apply.) a. Draining sinus tracts b. High fevers c. Presence of foot ulcers d. Swelling and redness e. Tenderness or pain

ANS: A, C Draining sinus tracts and foot ulcers are seen in chronic osteomyelitis. High fever, swelling, and redness are more often seen in acute osteomyelitis. Pain or tenderness can be in either case.

A nurse cares for a client who has elevated levels of antidiuretic hormone (ADH). Which disorder should the nurse identify as a trigger for the release of this hormone? a. Pneumonia b. Dehydration c. Renal failure d. Edema

ANS: B ADH increases tubular permeability to water, leading to absorption of more water into the capillaries. ADH is triggered by a rising extracellular fluid osmolarity, as occurs in dehydration. Pneumonia, renal failure, and edema would not trigger the release of ADH.

A nurse assesses an older client. Which assessment findings should the nurse identify as normal changes in the nervous system related to aging? (Select all that apply.) a. Long-term memory loss b. Slower processing time c. Increased sensory perception d. Decreased risk for infection e. Change in sleep patterns

ANS: B, E Normal changes in the nervous system related to aging include recent memory loss, slower processing time, decreased sensory perception, an increased risk for infection, changes in sleep patterns, changes in perception of pain, and altered balance and/or decreased coordination.

A hospitalized client is being treated for Ewing's sarcoma. What action by the nurse is most important? a. Assessing and treating the client for pain as needed b. Educating the client on the disease and its treatment c. Handling and disposing of chemotherapeutic agents per policy d. Providing emotional support for the client and family

ANS: C All actions are appropriate for this client. However, for safety, the nurse should place priority on proper handling and disposal of chemotherapeutic agents.

The nurse knows that hematopoiesis occurs in what part of the musculoskeletal system? a. Cancellous tissue b. Collagen matrix c. Red marrow d. Yellow marrow

ANS: C Hematopoiesis occurs in the red marrow, which is part of the cancellous tissues containing both types of bone marrow.

A nurse cares for a client who had a bronchoscopy 2 hours ago. The client asks for a drink of water. Which action should the nurse take next? a. Call the physician and request a prescription for food and water. b. Provide the client with ice chips instead of a drink of water. c. Assess the client's gag reflex before giving any food or water. d. Let the client have a small sip to see whether he or she can swallow.

ANS: C The topical anesthetic used during the procedure will have affected the client's gag reflex. Before allowing the client anything to eat or drink, the nurse must check for the return of this reflex.

The nurse studying osteoporosis learns that which drugs can cause this disorder? (Select all that apply.) a. Antianxiety agents b. Antibiotics c. Barbiturates d. Corticosteroids e. Loop diuretics

ANS: C, D, E Several classes of drugs can cause secondary osteoporosis, including barbiturates, corticosteroids, and loop diuretics. Antianxiety agents and antibiotics are not associated with the formation of osteoporosis.

15. A patient is experiencing an asthma attack and shows an increased respiratory effort. Which arterial blood gas value is more associated with the early phase of the attack? A. PaCO2 of 60 mmHg B. PaCO2 of 30 mmHg C. pH of 7.40 D. PaCO2 of 98 mmHg

B

16. A patient who has well-controlled asthma has what kind of airway changes? A. Chronic, leading to hyperplasia B. Temporary and reversible C. Open alveoli D. Permanent and irreversible

B

Which symptom of pneumonia may present differently in the older adult than in the younger adult? A) Crackles on auscultation B) Fever C) Headache D) Wheezing (Chp. 31; elsevier resources)

B) Fever (Chp. 31; elsevier resources)

13. For a patient that is a nonsmoker, which classic assessment finding of chronic airflow limitation is particularly important in diagnosing asthma? A. Cyanosis B. Dyspnea C. Audible wheezing D. Tachypnea

C

62. A patient is fearful that she might develop lung cancer because her father and grandfather died of cancer. She seeks advice about how to modify lifestyle factors that contribute to cancer. How does the nurse advise the patient? A. Not to worry about air pollution unless there is hydrocarbon exposure. B. Quit her job if she has continuous exposure to lead or other heavy metals C. Avoid situations where she would be exposed to "secondhand" smoke D. Not to be concerned because there are no genetic factors associated with lung cancer

C

A local hunter is admitted to the intensive care unit with a diagnosis of inhalation anthrax. Which medications does the RN anticipate the health care provider will order?

Ciprofloxacin (Cipro) 400 mg IV every 12 hours Intravenous ciprofloxacin (Cipro) is a first-line drug for treatment of inhaled anthrax. A dose of 400 mg IV every 12 hours is typically used for treatment of anthrax, while a dose of 500 mg orally twice daily is usually prescribed for anthrax prophylaxis. Oral doses of amoxicillin are used only as prophylaxis, not as treatment, for inhaled anthrax. Cephalosporins such as ceftriaxone are not used for treatment of anthrax. Pyrazinamide (Zinamide) is used for treatment of tuberculosis.

The nurse is assessing a client with chronic bronchitis who smoked 3 packs of cigarettes daily for 32 years. How does the nurse document pack-year history of smoking in the medical record? Client has a 32 pack-year history Client has a 96 pack-year history Client smoked 3 packs for years Client was a passive smoker for 32 years

Client has a 96 pack-year history This client has a 96-year pack history. Pack-year history refers to the number of packs per day multiplied by the number of years the client smoked.Pack-year history refers to the number of packs per day multiplied by the number of years the client smoked.

A 56-year-old client admitted with a diagnosis of acute myelogenous leukemia is prescribed IV cytosine arabinoside for 7 days and an infusion of daunorubicin for the first 3 days. What is the major side effect of this therapy? a) Bone marrow suppression b) Liver toxicity c) Nausea d) Stomatitis

Correct Answer: a Intravenous cytosine arabinoside and daunorubicin are a commonly prescribed course of aggressive chemotherapy, and bone marrow suppression is a major side effect. The client is even more at risk for infection than before treatment began. Liver toxicity, nausea, and stomatitis are not the major problems with this therapy.

A client with multiple myeloma reports bone pain that is unrelieved by analgesics. How does the nurse respond to this client's problem? a) "Ask your doctor to prescribe more medication." b) "It is too soon for additional medication to be given." c) "I'll turn on some soothing classical music for you." d) "Would you like to try some relaxation techniques?"

Correct Answer: d Because most clients with multiple myeloma have local or generalized bone pain, analgesics and alternative approaches for pain management, such as relaxation techniques, are used for pain relief. This also offers the client a choice. Before prescribing additional medication, other avenues should be explored to relieve this client's pain. Even if it is too soon to give additional medication, telling that to the client is not helpful because it dismisses the client's pain concerns. Although music therapy can be helpful, this response does not give the client a choice.

A client who has been newly diagnosed with leukemia is admitted to the hospital. Avoiding which potential problem takes priority in the client's nursing care plan? a) Fluid overload (overhydration) b) Hemorrhage c) Hypoxia d) Infection

Correct Answer: d The main objective in caring for a newly diagnosed client with leukemia is protection from infection. Fluid overload, hemorrhage, and hypoxia are not priority problems for the client with leukemia.

50. A patient with CF is admitted to the medical-surgical unit for an elective surgery. Which infection control measure is best for this patient? A. It is best to put two patients with CF in the same room. B. Standard precautions including handwashing are sufficient. C. The patient is to be placed on contact isolation. D. Measures that limit close contact between people with CF are needed.

D

A client is taking isoniazid, rifampin, pyrazinamide, and ethambutol for tuberculosis. The client calls to report visual changes, including blurred vision and reduced visual fields. Which medication may be causing these changes?

Ethambutol Ethambutol can cause optic neuritis, leading to blindness at high doses. When discovered early and the drug is stopped, problems can usually be reversed. Both isoniazid and pyrazinamide may cause liver failure; side effects of major concern include jaundice, bleeding, and abdominal pain. Rifampin will cause the urine and all other secretions to have a yellowish-orange color; this is harmless. Contact lenses will also be stained and oral contraceptives will be less effective.

When caring for a client with polycystic kidney disease, which goal is most important? Preventing progression of the disease Performing genetic testing Assessing for related causes Consulting with the dialysis unit

Preventing progression of the disease Correct Preventing complications and progression of the disease is the goal. Genetic testing should be done, but this is not a priority. Assessment for related causes is an intervention, not a goal. Not all clients with polycystic kidney disease require dialysis.

The nurse is providing pre-operative teaching for a pt who will have a malignant sinus tumor surgically removed. Which KEY teaching point would the nurse be sure to include? a. pts often have changes in the sensations of taste and smell b. problems of speech rarely occur with this type of surgery c. often pts gain weight and need dietary consults for weight loss d. after the surgery, you will need to have irrigations with an alcohol based solution

a

Which factors contribute to sleep apnea? SELECT ALL THAT APPLY a. smoking b. a short neck c. athletic lifestyle d. small uvula e. enlarged tonsil/adenoids f. underweight for height and gender

a b e

Question 20 of 29 The nurse is administering the intake assessment for a newly admitted client with a history of seizures. The client suddenly begins to seize. What does the nurse do next? a. Documents the length and time of the seizure. b. Forces a tongue blade in the mouth. c. Restrains the client. d. Positions the client on the side.

d Positions the client on the side. Turning the client on the side during a generalized tonic-clonic or complex partial seizure is indicated because he or she may lose consciousness. Documenting the length and time of seizures is important, but not the first priority intervention. Forcing a tongue blade in the mouth can cause damage. Restraining the client can cause injury.

A nurse notes crepitation when performing range-of-motion exercises on a client with a fractured left humerus. Which action should the nurse take next? a. Immobilize the left arm. b. Assess the client's distal pulse. c. Monitor for signs of infection. d. Administer prescribed steroids.

ANS: A A grating sound heard when the affected part is moved is known as crepitation. This sound is created by bone fragments. Because bone fragments may be present, the nurse should immobilize the client's arm and tell the client not to move the arm. The grating sound does not indicate circulation impairment or infection. Steroids would not be indicated.

A preoperative nurse assesses a client who has type 1 diabetes mellitus prior to a surgical procedure. The client's blood glucose level is 160 mg/dL. Which action should the nurse take? a. Document the finding in the client's chart. b. Administer a bolus of regular insulin IV. c. Call the surgeon to cancel the procedure. d. Draw blood gases to assess the metabolic state.

ANS: A Clients who have type 1 diabetes and are having surgery have been found to have fewer complications, lower rates of infection, and better wound healing if blood glucose levels are maintained at between 140 and 180 mg/dL throughout the perioperative period. The nurse should document the finding and proceed with other operative care. The need for a bolus of insulin, canceling the procedure, or drawing arterial blood gases is not required.

A nurse reviews the allergy list of a client who is scheduled for an intravenous urography. Which client allergy should alert the nurse to urgently contact the health care provider? a. Seafood b. Penicillin c. Bee stings d. Red food dye

ANS: A Clients with seafood allergies often have severe allergic reactions to the standard dyes used during intravenous urography. The other allergies have no impact on the client's safety during an intravenous urography.

A client in a nursing home refuses to take medications. She is at high risk for osteomalacia. What action by the nurse is best? a. Ensure the client gets 15 minutes of sun exposure daily. b. Give the client daily vitamin D injections. c. Hide vitamin D supplements in favorite foods. d. Plan to serve foods naturally high in vitamin D.

ANS: A Sunlight is a good source of vitamin D, and the nursing staff can ensure some sun exposure each day. Vitamin D is not given by injection. Hiding the supplement in food is unethical. Very few foods are naturally high in vitamin D, but some are supplemented.

The nurse instructs a client on the steps needed to obtain a peak expiratory flow rate. In which order should these steps occur? 1. "Take as deep a breath as possible." 2. "Stand up (unless you have a physical disability)." 3. "Place the meter in your mouth, and close your lips around the mouthpiece." 4. "Make sure the device reads zero or is at base level." 5. "Blow out as hard and as fast as possible for 1 to 2 seconds." 6. "Write down the value obtained." 7. "Repeat the process two additional times, and record the highest number in your chart." a. 4, 2, 1, 3, 5, 6, 7 b. 3, 4, 1, 2, 5, 7, 6 c. 2, 1, 3, 4, 5, 6, 7 d. 1, 3, 2, 5, 6, 7, 4

ANS: A The proper order for obtaining a peak expiratory flow rate is as follows. Make sure the device reads zero or is at base level. The client should stand up (unless he or she has a physical disability). The client should take as deep a breath as possible, place the meter in the mouth, and close the lips around the mouthpiece. The client should blow out as hard and as fast as possible for 1 to 2 seconds. The value obtained should be written down. The process should be repeated two more times, and the highest of the three numbers should be recorded in the client's chart.

A nurse is assessing a client who is recovering from a lung biopsy. Which assessment finding requires immediate action? a. Increased temperature b. Absent breath sounds c. Productive cough d. Incisional discomfort

ANS: B Absent breath sounds may indicate that the client has a pneumothorax, a serious complication after a needle biopsy or open lung biopsy. The other manifestations are not life threatening.

A client has been prescribed denosumab (Prolia). What instruction about this drug is most appropriate? a. "Drink at least 8 ounces of water with it." b. "Make appointments to come get your shot." c. "Sit upright for 30 to 60 minutes after taking it." d. "Take the drug on an empty stomach."

ANS: B Denosumab is given by subcutaneous injection twice a year. The client does not need to drink 8 ounces of water with this medication as it is not taken orally. The client does not need to remain upright for 30 to 60 minutes after taking this medication, nor does the client need to take the drug on an empty stomach.

An older adult is brought to the emergency department by a family member, who reports a moderate change in mental status and mild cough. The client is afebrile. The health care provider orders a chest x-ray. The family member questions why this is needed since the manifestations seem so vague. What response by the nurse is best? a. "Chest x-rays are always ordered when we suspect pneumonia." b. "Older people often have vague symptoms, so an x-ray is essential." c. "The x-ray can be done and read before laboratory work is reported." d. "We are testing for any possible source of infection in the client."

ANS: B It is essential to obtain an early chest x-ray in older adults suspected of having pneumonia because symptoms are often vague. Waiting until definitive manifestations are present to obtain the x-ray leads to a costly delay in treatment. Stating that chest x-rays are always ordered does not give the family definitive information. The x-ray can be done while laboratory values are still pending, but this also does not provide specific information about the importance of a chest x-ray in this client. The client has manifestations of pneumonia, so the staff is not testing for any possible source of infection but rather is testing for a suspected disorder.

A nurse reviews a female client's laboratory results. Which results from the client's urinalysis should the nurse recognize as abnormal? a. pH 5.6 b. Ketone bodies present c. Specific gravity of 1.020 d. Clear and yellow color

ANS: B Ketone bodies are by-products of incomplete metabolism of fatty acids. Normally no ketones are present in urine. Ketone bodies are produced when fat sources are used instead of glucose to provide cellular energy. A pH between 4.6 and 8, specific gravity between 1.005 and 1.030, and clear yellow urine are normal findings for a female client's urinalysis.

20. An emergency department nurse assesses a client with a history of urinary incontinence who presents with extreme dry mouth, constipation, and an inability to void. Which question should the nurse ask first? a. Are you drinking plenty of water? b. What medications are you taking? c. Have you tried laxatives or enemas? d. Has this type of thing ever happened before?

ANS: B Some types of incontinence are treated with anticholinergic medications such as propantheline (Pro-Banthine). Anticholinergic side effects include dry mouth, constipation, and urinary retention. The nurse needs to assess the clients medication list to determine whether the client is taking an anticholinergic medication. If he or she is taking anticholinergics, the nurse should further assess the clients manifestations to determine if they are related to a simple side effect or an overdose. The other questions are not as helpful to understanding the current situation. DIF: Applying/Application REF: 1355 KEY: Urinary incontinence| medication safety MSC: Integrated Process: Nursing Process: Assessment NOT: Client Needs Category: Physiological Integrity: Pharmacological and Parenteral Therapies

A pulmonary nurse cares for clients who have chronic obstructive pulmonary disease (COPD). Which client should the nurse assess first? a. A 46-year-old with a 30-pack-year history of smoking b. A 52-year-old in a tripod position using accessory muscles to breathe c. A 68-year-old who has dependent edema and clubbed fingers d. A 74-year-old with a chronic cough and thick, tenacious secretions

ANS: B The client who is in a tripod position and using accessory muscles is working to breathe. This client must be assessed first to establish how well the client is breathing and provide interventions to minimize respiratory failure. The other clients are not in acute distress.

24. A nurse assesses a male client who is recovering from a urologic procedure. Which assessment finding indicates an obstruction of urine flow? a. Severe pain b. Overflow incontinence c. Hypotension d. Blood-tinged urine

ANS: B The most common manifestation of urethral stricture after a urologic procedure is obstruction of urine flow. This rarely causes pain and has no impact on blood pressure. The client may experience overflow incontinence with the involuntary loss of urine when the bladder is distended. Blood in the urine is not a manifestation of the obstruction of urine flow. DIF: Applying/Application REF: 1350 KEY: Urethral strictures| urinary incontinence| postoperative nursing MSC: Integrated Process: Nursing Process: Assessment NOT: Client Needs Category: Physiological Integrity: Physiological Adaptation

12. A nurse assesses a client who is recovering from extracorporeal shock wave lithotripsy for renal calculi. The nurse notes an ecchymotic area on the clients right lower back. Which action should the nurse take? a. Administer fresh-frozen plasma. b. Apply an ice pack to the site. c. Place the client in the prone position. d. Obtain serum coagulation test results.

ANS: B The shock waves from lithotripsy can cause bleeding into the tissues through which the waves pass. Application of ice can reduce the extent and discomfort of the bruising. Although coagulation test results and fresh-frozen plasma are used to assess and treat bleeding disorders, ecchymosis after this procedure is not unusual and does not warrant a higher level of intervention. Changing the clients position will not decrease bleeding. DIF: Applying/Application REF: 1363 KEY: Urolithiasis| postoperative nursing MSC: Integrated Process: Nursing Process: Implementation NOT: Client Needs Category: Physiological Integrity: Reduction of Risk Potential

A nurse cares for a client who is prescribed pioglitazone (Actos). After 6 months of therapy, the client reports that his urine has become darker since starting the medication. Which action should the nurse take? a. Assess for pain or burning with urination. b. Review the client's liver function study results. c. Instruct the client to increase water intake. d. Test a sample of urine for occult blood.

ANS: B Thiazolidinediones (including pioglitazone) can affect liver function, liver function should be assessed at the start of therapy and at regular intervals while the client continues to take these drugs. Dark urine is one indicator of liver impairment because bilirubin is increased in the blood and is excreted in the urine. The nurse should check the client's most recent liver function studies. The nurse does not need to assess for pain or burning with urination and does not need to check the urine for occult blood. The client does not need to be told to increase water intake.

A nurse cares for a client with a urine specific gravity of 1.018. Which action should the nurse take? a. Evaluate the client's intake and output for the past 24 hours. b. Document the finding in the chart and continue to monitor. c. Obtain a specimen for a urine culture and sensitivity. d. Encourage the client to drink more fluids, especially water.

ANS: B This specific gravity is within the normal range for urine. There is no need to evaluate the client's intake and output, obtain a urine specimen, or increase fluid intake.

After administering a medication that stimulates the sympathetic division of the autonomic nervous system, the nurse assesses the client. For which clinical manifestations should the nurse assess? (Select all that apply.) a. Decreased respiratory rate b. Increased heart rate c. Decreased level of consciousness d. Increased force of contraction e. Decreased blood pressure

ANS: B, D Stimulation of the sympathetic nervous system initiates the fight-or-flight response, increasing both the heart rate and the force of contraction. A medication that stimulates the sympathetic nervous system would also increase the clients respiratory rate, blood pressure, and level of consciousness.

4. A nurse assesses a client who is recovering from a nephrostomy. Which assessment findings should alert the nurse to urgently contact the health care provider? (Select all that apply.) a. Clear drainage b. Bloody drainage at site c. Client reports headache d. Foul-smelling drainage e. Urine draining from site

ANS: B, D, E After a nephrostomy, the nurse should assess the client for complications and urgently notify the provider if drainage decreases or stops, drainage is cloudy or foul-smelling, the nephrostomy sites leaks blood or urine, or the client has back pain. Clear drainage is normal. A headache would be an unrelated finding.

The nurse is teaching a client with chronic obstructive pulmonary disease who has been prescribed continuous oxygen therapy at home. Which statement indicates the client correctly understands the teaching? a. "I plan to wear my oxygen when I exercise and feel short of breath." b. "I will use my portable oxygen when grilling burgers in the backyard." c. "I plan to use cotton balls to cushion the oxygen tubing on my ears." d. "I will only smoke while I am wearing my oxygen via nasal cannula."

ANS: C Cotton balls can decrease pressure ulcers from the oxygen tubing. Continuous oxygen orders mean the client should wear the oxygen at all times. Oxygen fuels a fire. Wearing oxygen while grilling and smoking increases the risk for fire.

A nurse assesses a client after an open lung biopsy. Which assessment finding is matched with the correct intervention? a. Client states he is dizzy. - Nurse applies oxygen and pulse oximetry. b. Client's heart rate is 55 beats/min. - Nurse withholds pain medication. c. Client has reduced breath sounds. - Nurse calls physician immediately. d. Client's respiratory rate is 18 breaths/min. - Nurse decreases oxygen flow rate.

ANS: C A potentially serious complication after biopsy is pneumothorax, which is indicated by decreased or absent breath sounds. The physician needs to be notified immediately. Dizziness after the procedure is not an expected finding. If the client's heart rate is 55 beats/min, no reason is known to withhold pain medication. A respiratory rate of 18 breaths/min is a normal finding and would not warrant changing the oxygen flow rate.

A nurse teaches a client who is prescribed nicotine replacement therapy. Which statement should the nurse include in this client's teaching? a. "Make a list of reasons why smoking is a bad habit." b. "Rise slowly when getting out of bed in the morning." c. "Smoking while taking this medication will increase your risk of a stroke." d. "Stopping this medication suddenly increases your risk for a heart attack."

ANS: C Clients who smoke while using drugs for nicotine replacement therapy increase the risk of stroke and heart attack. Nurses should teach clients not to smoke while taking this drug. The other responses are inappropriate.

The nurse is teaching a client with chronic obstructive pulmonary disease who has been prescribed continuous oxygen therapy at home. Which statement indicates the client correctly understands the teaching? a. "I plan to wear my oxygen when I exercise and feel short of breath." b. "I will use my portable oxygen when grilling burgers in the backyard." c. "I plan to use cotton balls to cushion the oxygen tubing on my ears." d. "I will only smoke while I am wearing my oxygen via nasal cannula."

ANS: C Cotton balls can decrease pressure ulcers from the oxygen tubing. Continuous oxygen orders mean the client should wear the oxygen at all times. Oxygen fuels a fire. Wearing oxygen while grilling and smoking increases the risk for fire.

A nurse teaches a client who is recovering from a urography. Which instruction should the nurse include in this client's discharge teaching? a. "Avoid direct contact with your urine for 24 hours until the radioisotope clears." b. "You may have some dribbling of urine for several weeks after this procedure." c. "Be sure to drink at least 3 liters of fluids today to help eliminate the dye faster." d. "Your skin may become slightly yellow from the dye used in this procedure."

ANS: C Dyes used in urography are potentially nephrotoxic. A large fluid intake will help the client eliminate the dye rapidly. Dyes used in urography are not radioactive, the client should not experience any dribbling of urine, and the dye should not change the color of the client's skin.

1. A nurse assesses clients on the medical-surgical unit. Which client is at greatest risk for the development of bacterial cystitis? a. A 36-year-old female who has never been pregnant b. A 42-year-old male who is prescribed cyclophosphamide c. A 58-year-old female who is not taking estrogen replacement d. A 77-year-old male with mild congestive heart failure

ANS: C Females at any age are more susceptible to cystitis than men because of the shorter urethra in women. Postmenopausal women who are not on hormone replacement therapy are at increased risk for bacterial cystitis because of changes in the cells of the urethra and vagina. The middle-aged woman who has never been pregnant would not have a risk potential as high as the older woman who is not using hormone replacement therapy. DIF: Understanding/Comprehension REF: 1344 KEY: Cystitis| health screening MSC: Integrated Process: Nursing Process: Assessment NOT: Client Needs Category: Safe and Effective Care Environment: Management of Care

An older client with diabetes is admitted with a heavily draining leg wound. The client's white blood cell count is 38,000/mm3 but the client is afebrile. What action does the nurse take first? a. Administer acetaminophen (Tylenol). b. Educate the client on amputation. c. Place the client on contact isolation. d. Refer the client to the wound care nurse.

ANS: C In the presence of a heavily draining wound, the nurse should place the client on contact isolation. If the client has discomfort, acetaminophen can be used, but this client has not reported pain and is afebrile. The client may or may not need an amputation in the future. The wound care nurse may be consulted, but not as the first action.

After teaching a client who is prescribed a long-acting beta2 agonist medication, a nurse assesses the client's understanding. Which statement indicates the client comprehends the teaching? a. "I will carry this medication with me at all times in case I need it." b. "I will take this medication when I start to experience an asthma attack." c. "I will take this medication every morning to help prevent an acute attack." d. "I will be weaned off this medication when I no longer need it."

ANS: C Long-acting beta2 agonist medications will help prevent an acute asthma attack because they are long acting. The client will take this medication every day for best effect. The client does not have to always keep this medication with him or her because it is not used as a rescue medication. This is not the medication the client will use during an acute asthma attack because it does not have an immediate onset of action. The client will not be weaned off this medication because this is likely to be one of his or her daily medications.

A nurse cares for a client with chronic obstructive pulmonary disease (COPD). The client states that he no longer enjoys going out with his friends. How should the nurse respond? a. "There are a variety of support groups for people who have COPD." b. "I will ask your provider to prescribe you with an antianxiety agent." c. "Share any thoughts and feelings that cause you to limit social activities." d. "Friends can be a good support system for clients with chronic disorders."

ANS: C Many clients with moderate to severe COPD become socially isolated because they are embarrassed by frequent coughing and mucus production. They also can experience fatigue, which limits their activities. The nurse needs to encourage the client to verbalize thoughts and feelings so that appropriate interventions can be selected. Joining a support group would not decrease feelings of social isolation if the client does not verbalize feelings. Antianxiety agents will not help the client with social isolation. Encouraging a client to participate in activities without verbalizing concerns also would not be an effective strategy for decreasing social isolation.

A nurse cares for an older adult client who is recovering from a leg amputation surgery. The client states, "I don't want to live with only one leg. I should have died during the surgery." How should the nurse respond? a. "Your vital signs are good, and you are doing just fine right now." b. "Your children are waiting outside. Do you want them to grow up without a father?" c. "This is a big change for you. What support system do you have to help you cope?" d. "You will be able to do some of the same things as before you became disabled."

ANS: C The client feels like less of a person following the amputation. The nurse should help the client to identify coping mechanisms that have worked in the past and current support systems to assist the client with coping. The nurse should not ignore the client's feelings by focusing on vital signs. The nurse should not try to make the client feel guilty by alluding to family members. The nurse should not refer to the client as being "disabled" as this labels the client and may fuel the client's poor body image.

25. A nurse cares for a client with urinary incontinence. The client states, I am so embarrassed. My bladder leaks like a young childs bladder. How should the nurse respond? a. I understand how you feel. I would be mortified. b. Incontinence pads will minimize leaks in public. c. I can teach you strategies to help control your incontinence. d. More women experience incontinence than you might think.

ANS: C The nurse should accept and acknowledge the clients concerns, and assist the client to learn techniques that will allow control of urinary incontinence. The nurse should not diminish the clients concerns with the use of pads or stating statistics about the occurrence of incontinence. DIF: Applying/Application REF: 1353 KEY: Urinary incontinence| psychosocial response| coping MSC: Integrated Process: Caring NOT: Client Needs Category: Psychosocial Integrity

A nurse is providing care after auscultating clients' breath sounds. Which assessment finding is correctly matched to the nurse's primary intervention? a. Hollow sounds are heard over the trachea. - The nurse increases the oxygen flow rate. b. Crackles are heard in bases. - The nurse encourages the client to cough forcefully. c. Wheezes are heard in central areas. - The nurse administers an inhaled bronchodilator. d. Vesicular sounds are heard over the periphery. - The nurse has the client breathe deeply.

ANS: C Wheezes are indicative of narrowed airways, and bronchodilators help to open the air passages. Hollow sounds are typically heard over the trachea, and no intervention is necessary. If crackles are heard, the client may need a diuretic. Crackles represent a deep interstitial process, and coughing forcefully will not help the client expectorate secretions. Vesicular sounds heard in the periphery are normal and require no intervention.

A nurse is caring for four clients. After reviewing todays laboratory results, which client should the nurse see first? a. Client with an international normalized ratio of 2.8 b. Client with a platelet count of 128,000/mm3 c. Client with a prothrombin time (PT) of 28 seconds d. Client with a red blood cell count of 5.1 million/L

ANS: C A normal PT is 11 to 12.5 seconds. This client is at high risk of bleeding. The other values are within normal limits.

A nurse is teaching a client with cerebellar function impairment. Which statement should the nurse include in this clients discharge teaching? a. Connect a light to flash when your door bell rings. b. Label your faucet knobs with hot and cold signs. c. Ask a friend to drive you to your follow-up appointments. d. Use a natural gas detector with an audible alarm.

ANS: C Cerebellar function enables the client to predict distance or gauge the speed with which one is approaching an object, control voluntary movement, maintain equilibrium, and shift from one skilled movement to another in an orderly sequence. A client who has cerebellar function impairment should not be driving. The client would not have difficulty hearing, distinguishing between hot and cold, or smelling.

A nurse assesses a client with a brain tumor. The client opens his eyes when the nurse calls his name, mumbles in response to questions, and follows simple commands. How should the nurse document this clients assessment using the Glasgow Coma Scale shown below? a. 8 b. 10 c. 12 d. 14

ANS: C The client opens his eyes to speech (Eye opening: To sound = 3), mumbles in response to questions (Verbal response: Inappropriate words = 3), and follows simple commands (Motor response: Obeys commands = 6). Therefore, the clients Glasgow Coma Scale score is: 3 + 3 + 6 = 12.

A nurse prepares to teach a client who has experienced damage to the left temporal lobe of the brain. Which action should the nurse take when providing education about newly prescribed medications to this client? a. Help the client identify each medication by its color. b. Provide written materials with large print size. c. Sit on the clients right side and speak into the right ear. d. Allow the client to use a white board to ask questions.

ANS: C The temporal lobe contains the auditory center for sound interpretation. The clients hearing will be impaired in the left ear. The nurse should sit on the clients right side and speak into the right ear. The other interventions do not address the clients left temporal lobe damage.

3. A nurse reviews laboratory results for a client with glomerulonephritis. The client's glomerular filtration rate (GFR) is 40 mL/min as measured by a 24-hour creatinine clearance. How should the nurse interpret this finding? (Select all that apply.) a. Excessive GFR b. Normal GFR c. Reduced GFR d. Potential for fluid overload e. Potential for dehydration

ANS: C, D The GFR refers to the initial amount of urine that the kidneys filter from the blood. In the healthy adult, the normal GFR ranges between 100 and 120 mL/min, most of which is reabsorbed in the kidney tubules. A GFR of 40 mL/min is drastically reduced, with the client experiencing fluid retention and risks for hypertension and pulmonary edema as a result of excess vascular fluid.

2. A nurse cares for a client with autosomal dominant polycystic kidney disease (ADPKD). The client asks, "Will my children develop this disease?" How should the nurse respond? a. "No genetic link is known, so your children are not at increased risk." b. "Your sons will develop this disease because it has a sex-linked gene." c. "Only if both you and your spouse are carriers of this disease." d. "Each of your children has a 50% risk of having ADPKD."

ANS: D Children whose parent has the autosomal dominant form of PKD have a 50% chance of inheriting the gene that causes the disease. ADPKD is transmitted as an autosomal dominant trait and therefore is not gender specific. Both parents do not need to have this disorder.

After teaching a young adult client who is newly diagnosed with type 1 diabetes mellitus, the nurse assesses the client's understanding. Which statement made by the client indicates a correct understanding of the need for eye examinations? a. "At my age, I should continue seeing the ophthalmologist as I usually do." b. "I will see the eye doctor when I have a vision problem and yearly after age 40." c. "My vision will change quickly. I should see the ophthalmologist twice a year." d. "Diabetes can cause blindness, so I should see the ophthalmologist yearly."

ANS: D Diabetic retinopathy is a leading cause of blindness in North America. All clients with diabetes, regardless of age, should be examined by an ophthalmologist (rather than an optometrist or optician) at diagnosis and at least yearly thereafter.

A nurse reviews the medication list of a client recovering from a computed tomography (CT) scan with IV contrast to rule out small bowel obstruction. Which medication should alert the nurse to contact the provider and withhold the prescribed dose? a. Pioglitazone (Actos) b. Glimepiride (Amaryl) c. Glipizide (Glucotrol) d. Metformin (Glucophage)

ANS: D Glucophage should not be administered when the kidneys are attempting to excrete IV contrast from the body. This combination would place the client at high risk for kidney failure. The nurse should hold the metformin dose and contact the provider. The other medications are safe to administer after receiving IV contrast.

A nurse assesses a client's respiratory status. Which information is of highest priority for the nurse to obtain? a. Average daily fluid intake b. Neck circumference c. Height and weight d. Occupation and hobbies

ANS: D Many respiratory problems occur as a result of chronic exposure to inhalation irritants used in a client's occupation and hobbies. Although it will be important for the nurse to assess the client's fluid intake, height, and weight, these will not be as important as determining his occupation and hobbies. Determining the client's neck circumference will not be an important part of a respiratory assessment.

A nurse reviews the urinalysis results of a client and notes a urine osmolality of 1200 mOsm/L. Which action should the nurse take? a. Contact the provider and recommend a low-sodium diet. b. Prepare to administer an intravenous diuretic. c. Obtain a suction device and implement seizure precautions. d. Encourage the client to drink more fluids.

ANS: D Normal urine osmolality ranges from 300 to 900 mOsm/L. This client's urine is more concentrated, indicating dehydration. The nurse should encourage the client to drink more water. Dehydration can be associated with elevated serum sodium levels. Although a low-sodium diet may be appropriate for this client, this diet change will not have a significant impact on urine osmolality. A diuretic would increase urine output and decrease urine osmolality further. Low serum sodium levels, not elevated serum levels, place the client at risk for seizure activity. These options would further contribute to the client's dehydration or elevate the osmolality.

A phone triage nurse speaks with a client who has an arm cast. The client states, "My arm feels really tight and puffy." How should the nurse respond? a. "Elevate your arm on two pillows and get ice to apply to the cast." b. "Continue to take ibuprofen (Motrin) until the swelling subsides." c. "This is normal. A new cast will often feel a little tight for the first few days." d. "Please come to the clinic today to have your arm checked by the provider."

ANS: D Puffy fingers and a feeling of tightness from the cast may indicate the development of compartment syndrome. The client should come to the clinic that day to be evaluated by the provider because delay of treatment can cause permanent damage to the extremity. Ice and ibuprofen are acceptable actions, but checking the cast is the priority because it ensures client safety. The nurse should not reassure the client that this is normal.

11. A nurse cares for a client who has kidney stones from secondary hyperoxaluria. Which medication should the nurse anticipate administering? a. Phenazopyridine (Pyridium) b. Propantheline (Pro-Banthine) c. Tolterodine (Detrol LA) d. Allopurinol (Zyloprim)

ANS: D Stones caused by secondary hyperoxaluria respond to allopurinol (Zyloprim). Phenazopyridine is given to clients with urinary tract infections. Propantheline is an anticholinergic. Tolterodine is an anticholinergic with smooth muscle relaxant properties. DIF: Applying/Application REF: 1363 KEY: Urolithiasis| medications MSC: Integrated Process: Nursing Process: Planning NOT: Client Needs Category: Physiological Integrity: Pharmacological and Parenteral Therapies

After teaching a client with a fractured humerus, the nurse assesses the client's understanding. Which dietary choice demonstrates that the client correctly understands the nutrition needed to assist in healing the fracture? a. Baked fish with orange juice and a vitamin D supplement b. Bacon, lettuce, and tomato sandwich with a vitamin B supplement c. Vegetable lasagna with a green salad and a vitamin A supplement d. Roast beef with low-fat milk and a vitamin C supplement

ANS: D The client with a healing fracture needs supplements of vitamins B and C and a high-protein, high-calorie diet. Milk for calcium supplementation and vitamin C supplementation are appropriate. Meat would increase protein in the diet that is necessary for bone healing. Fish, a sandwich, and vegetable lasagna would provide less protein.

10. A nurse cares for a client who is recovering after a nephrostomy tube was placed 6 hours ago. The nurse notes drainage in the tube has decreased from 40 mL/hr to 12 mL over the last hour. Which action should the nurse take? a. Document the finding in the client's record. b. Evaluate the tube as working in the hand-off report. c. Clamp the tube in preparation for removing it. d. Assess the client's abdomen and vital signs.

ANS: D The nephrostomy tube should continue to have a consistent amount of drainage. If the drainage slows or stops, it may be obstructed. The nurse must notify the provider, but first should carefully assess the client's abdomen for pain and distention and check vital signs so that this information can be reported as well. The other interventions are not appropriate.

A nurse is caring for a client who is recovering from an above-the-knee amputation. The client reports pain in the limb that was removed. How should the nurse respond? a. "The pain you are feeling does not actually exist." b. "This type of pain is common and will eventually go away." c. "Would you like to learn how to use imagery to minimize your pain?" d. "How would you describe the pain that you are feeling?"

ANS: D The nurse should ask the client to rate the pain on a scale of 0 to 10 and describe how the pain feels. Although phantom limb pain is common, the nurse should not minimize the pain that the client is experiencing by stating that it does not exist or will eventually go away. Antiepileptic drugs and antispasmodics are used to treat neurologic pain and muscle spasms after amputation. Although imagery may assist the client, the nurse must assess the client's pain before determining the best action.

A nurse assesses a clients recent memory. Which client statement confirms that the clients remote memory is intact? a. A young girl wrapped in a shroud fell asleep on a bed of clouds. b. I was born on April 3, 1967, in Johnstown Community Hospital. c. Apple, chair, and pencil are the words you just stated. d. I ate oatmeal with wheat toast and orange juice for breakfast.

ANS: D Asking clients about recent events that can be verified, such as what the client ate for breakfast, assesses the clients recent memory. The clients ability to make up a rhyme tests not memory, but rather a higher level of cognition. Asking clients about certain facts from the past that can be verified assesses remote or long-term memory. Asking the client to repeat words assesses the clients immediate memory.

The nurse receives report on a client with hydronephrosis. Which laboratory study does the nurse monitor? Hemoglobin and hematocrit (H&H) White blood cell (WBC) count Blood urea nitrogen (BUN) and creatinine Lipid levels

Blood urea nitrogen (BUN) and creatinine BUN and creatinine are kidney function tests. With back-pressure on the kidney, glomerular filtration is reduced or absent, resulting in permanent kidney damage. Hydronephrosis results from the backup of urine secondary to obstruction; H&H monitors for anemia and blood loss, while WBC count indicates infection. Elevated lipid levels are associated with nephrotic syndrome, not with obstruction and hydronephrosis.

The medical-surgical unit has one negative-airflow room. Which of these four clients who have just arrived on the unit should the charge nurse admit to this room? A) Client with bacterial pneumonia and a cough productive of green sputum B) Client with neutropenia and pneumonia caused by Candida albicans C) Client with possible pulmonary tuberculosis who currently has hemoptysis D) Client with right empyema who has a chest tube and a fever of 103.2° F (Chp. 31; elsevier resources)

C) Client with possible pulmonary tuberculosis who currently has hemoptysis (Chp. 31; elsevier resources)

When caring for a client 24 hours after a nephrectomy, the nurse notes that the client's abdomen is distended. Which action does the nurse perform next? Check vital signs. Notify the surgeon. Continue to monitor. Insert a nasogastric (NG) tube.

Check vital signs. The client's abdomen may be distended from bleeding; hemorrhage or adrenal insufficiency causes hypotension, so vital signs should be taken to see if a change in blood pressure has occurred. The surgeon should be notified after vital signs are assessed. An NG tube is not indicated for this client.

The nurse is caring for a client with neutropenia. Which clinical manifestation indicates that an infection is present or should be ruled out? a) Coughing and deep breathing b) Evidence of pus c) Fever of 102° F or higher d) Wheezes or crackles

Correct Answer: d Wheezes or crackles in the neutropenic client may be the first symptom of infection in the lungs. Coughing and deep breathing are not indications of infection, but can help prevent it. The client with leukopenia, not neutropenia, may have a severe infection without pus or with only a low-grade fever.

When assessing a client with acute glomerulonephritis, which finding causes the nurse to notify the provider? Purulent wound on the leg Crackles throughout the lung fields History of diabetes Cola-colored urine

Crackles throughout the lung fields Crackles indicate fluid overload resulting from kidney damage; shortness of breath and dyspnea are typically associated. The provider should be notified of this finding. Glomerulonephritis may result from infection (e.g., purulent wound); it is not an emergency about which to notify the provider. The history of diabetes would have been obtained on admission. Dark urine is expected in glomerulonephritis.

Which of these clients will the charge nurse on the medical unit assign to an RN who has floated from the postanesthesia care unit (PACU)? A. Client with allergic rhinitis scheduled for skin testing B. Client with emphysema who needs teaching about pulmonary function testing C. Client with pancreatitis who needs a preoperative chest x-ray D. Client with pleural effusion who has had 1200 mL removed by thoracentesis

D. Client with pleural effusion who has had 1200 mL removed by thoracentesis Correct: A nurse working in the PACU would be familiar with assessing vital signs and respiratory status after procedures such as thoracentesis.

When assessing a client with acute pyelonephritis, which findings does the nurse anticipate will be present? (Select all that apply.) Suprapubic pain Vomiting Chills Dysuria Oliguria

Suprapubic pain Vomiting Correct Chills Correct Dysuria Correct Oliguria Nausea and vomiting are symptoms of acute pyelonephritis. Chills along with fever may also occur, as well as burning (dysuria), urgency, and frequency. Suprapubic pain is indicative of cystitis, not kidney infection (pyelonephritis). Oliguria is related to kidney impairment from severe or long-standing pyelonephritis.

A client is being admitted for pneumonia. The sputum culture is positive for streptococcus, and the client asks about the length of the treatment. On what does the nurse base the answer?

The client will be treated for 5 to 7 days. Anti-infectives usually are used for 5 to 7 days in uncomplicated community-acquired pneumonia, and for up to 21 days in an immunocompromised client or one with hospital-acquired pneumonia. A client may become afebrile early in the course of treatment with anti-infective medications; this may cause many clients to fail to complete their course of treatment.

The nurse is caring for a client with severe acute respiratory syndrome. What is the most important precaution the nurse should take when preparing to suction this client?

Wearing a disposable particulate mask respirator and protective eyewear To protect health care workers during procedures that induce coughing or promote aerosolization of particles, nurses should wear a particulate mask respirator and protective eyewear to prevent the spread of infectious organisms. Keeping the head of the bed elevated 30 to 45 degrees is not the most important precaution. Performing oral care is a comfort measure. Washing hands and donning gloves is necessary, but not the most important measure.

Question 24 of 29 The nurse has received report on a group of clients. Which client requires the nurse's attention first? a. Adult who is lethargic after a generalized tonic-clonic seizure b. Young adult who has experienced four tonic-clonic seizures within the past 30 minutes c. Middle-aged adult with absence seizures who is staring at a wall and does not respond to questions d. Older adult with a seizure disorder who has a temperature of 101.9° F (38.8° C)

b Young adult who has experienced four tonic-clonic seizures within the past 30 minutes The young adult client who is experiencing repeated seizures over the course of 30 minutes is in status epilepticus, which is a medical emergency and requires immediate intervention. The adult client who is lethargic and the middle-aged adult client with absence seizures do not require immediate attention; these are not medical emergencies. A fever of 101.9° F (38.8° C) is not a medical emergency and does not require immediate attention.

Question 17 of 29 The nurse is providing medication instructions for a client for whom phenytoin (Dilantin) has been ordered for treatment of epilepsy. The nurse instructs the client to avoid which beverage? a. Apple juice b. Grape juice c. Grapefruit juice d. Milk

c Grapefruit juice Grapefruit juice can interfere with the metabolism of phenytoin. Apple juice, grape juice, and milk do not interact with phenytoin.


संबंधित स्टडी सेट्स

Unit 13: THE civil war Required reading: BRINKLEY, CH. 14

View Set

Psych 305 Exam 2 Part 2: Chapter 6 & 7

View Set

THE ULTIMATE TEAS COMBINATION EAT YOUR HEART OUT (ONLY SOME SCIENCE)llu

View Set

Chapter 3: Medical Expenses Insurance

View Set

Chapter 14: Nursing Management During Labor and Birth

View Set

Chapter 10- Complete 20 hours of Training

View Set